You are on page 1of 360

Unit 3 Table of Contents

MA
TERNAL & CHILD NURSING
Section I. Anatomy & !ysiolo"y
#. Re$%o&'cti(e System
a. )emale Re$%o&'cti(e System
#* E+te%nal Genitalia
,* Inte%nal Genitalia
3* Ty$es of el(ic Li"aments
b. Male Re$%o&'cti(e System
#* E+te%nal & Inte%nal )eat'%es
,. Mamma%y Glan&s
3. Re$%o&'cti(e Ho%mones
a. )emale Re$%o&'cti(e Ho%mones
b. -t!e% Re$%o&'cti(e Ho%mones
.. Menst%'ation
a. Menst%'al C!an"es
b. Menst%'al Cycle
c. -(a%ian Cycle
&. En&omet%ial / Ute%ine Cycle
e. Menst%'al Diso%&e%s
0. )amily lannin"
a. Nat'%al Conce$tion
b. 1a%%ie% Met!o&s
c. !a%macolo"ical Met!o&s
&. 1i%t! Cont%ol S'mma%y

Section II. Ante$a%tal e%io&
#. Assessment of %enatal Ris2 )acto%s
,. !ysiolo"ical C!an"es in %e"nancy
a. !ysiolo"ical C!an"es
b. Ante$a%t'm Healt! %omotion
3. )e%tili3ation to Conce$tion
a. )e%tili3ation
1
b. -%i"in of 1o&y Tiss'es
.. )etal De(elo$ment
a. Meas'%in" A"e of Gestation
0. Mate%nal & )etal Dia"nostic Tests

4. Elect%onic )etal Monito%in"
5. Labo%ato%y St'&ies

6. -t!e% Gynecolo"ical %oce&'%es
7. T!%ee Common %e"nancy Si"ns
#8. Discomfo%t Si"ns of %e"nancy
##. syc!olo"ical C!an"es in %e"nancy
a. Mate%nal C!an"es in %e"nancy
b. ate%nal A&a$tations in %e"nancy
Section III. Ante$a%tal Com$lications
#. Abo%tion
,. Ecto$ic %e"nancy
3. H9mole
.. Incom$etent Ce%(i+
0. Diabetes Mellit's of %e"nancy
4. IH :%e"nancy In&'ce& Hy$e%tension*
5. 1lee&in" Diso%&e%s in %e"nancy : Table of Com$a%ison*
a. lacenta %e(ia
b. Ab%'$tio lacenta
6. ;ena Ca(a Syn&%ome
7. Diseminate& Int%a(asc'la% Coa"'lation
#8. Hy$e%emesis G%a(i&a%'m
Section I;. Int%a$a%t'm Ca%e
#. )i(e )acto%s Affectin" Labo% :Table of Mec!anics of Labo%*
2
a. assa"e<ay
#. Ty$es of el(is
,. el(ic Meas'%ements
b. assen"e%
#. )etal Attit'&e
,. )etal Lie
3. )etal $%esentation
.. )etal osition
c. o<e%
#. T!%ee !ases of Cont%action
,. C!a%acte%istics of Cont%actions
&. lacental )acto%s
e. syc!e
,. Labo%
a. Si"ns of Im$en&in" Labo%
b. Com$a%ison of T%'e & )alse Labo%
c. Sta"es of Labo%
#. Stations of %esentin" a%t
&. N'%sin" Consi&e%ations &'%in" Labo% & Deli(e%y
e. N'%sin" Ca%e &'%in" labo%
f. Assessin" )etal Hea%t Rate
". Ca%&inal Mec!anisms / Mo(ements of Labo%
.. Anest!e3ia
0. -bstet%ical %oce&'%es
a. %ete%m Labo%
b. R-M :%emat'%e R'$t'%e of t!e Memb%anes*
c. %ola$se Co%&
&. Dystocia
e. Infection
3
f. %eci$itate Deli(e%y
". Ute%ine R'$t'%e
!. Amniotic )l'i& Embolism
Section ;. Com$lications of Labo% & Deli(e%y
a. %ete%m Labo%
b. R-M : %emat'%e R'$t'%e of t!e Memb%anes*
c. %ola$se& Umbilical Co%&
&. Dystocia
e. Infection
f. %eci$itate Deli(e%y
". Ute%ine %'$t'%e
!. Amniotic )l'i& embolism
Section ;I. ost$a%t'm
#. ost$a%t'm 1io$!ysical c!an"es
a. Loc!ia
b. Ute%'s
c. Ute%ine In(ol'tion
&. 1%east
e. GI T%act
,. ost a%t'm Discomfo%ts
a. e%ineal &iscomfo%ts
b. E$isiotomy
c. 1%east Discomfo%ts
4
3. ost $a%t'm Disc!a%"e Teac!in"s
a. 1%east fee&in"s
b. 1'%$in" & )ee&in"
c. syc!olo"ical A&a$tations
SECTI-N ;II. Neonatal Ca%e
#. Initial !ysical E+amination & Ca%e of t!e Ne<bo%n
a. Assessment
b. Im$lementation
c. ;ital Si"ns
&. 1o&y Meas'%ement
,. Hea& to Toe Ne<bo%n Assessment
3. Gestational Assessment
.. Ne<bo%n Refle+es
0. 1asic Teac!in" Nee&s of Ne< a%ents
4. %ete%m Neonates
5. ost te%m Neonates
6. -t!e% Ne<bo%n Abno%malities
a. RDS :Res$i%ato%y Dist%ess Syn&%ome*
b. Hemolytic Disease
c. Hy$e%bili%'binemia
&. E%yt!%oblastosis )etalis
e. T!e Ne<bo%n of A&&icte& Mot!e%s
f. SGA :Small Gestational A"e*
". Ne%(o's System Anomalies
#. S$ina 1ifi&a
,. Menin"ocele
3. Myelomenin"ocele
5
Unit 3
MATERNAL AND CHILD HEALTH NURSING
Section I
ANAT-M= AND H=SI-L-G= -) THE )EMALE RER-DUCTI;E S=STEM
6
I.a E+te%nal Genitalia :;'l(a/'&en&'m*
M-NS U1IS
-Soft fatty tissue, lies directly over symphysis pubis & becomes covered w/ hair ust before puberty
!t is where the pubic hair "rows#
$
LA1IA MA>-RA
-%/ hair outside but smooth i&side
fatty s'i& folds from ()*S +,-!S to
+./!*.,( a&d protects the labia
mi&ora , uri&ary meatus & va"i&a
LA1IA MIN-RA
-0hi&, pi&', smooth, hairless, e1tremely
se&sitive to pressure, touch a&d
temperature# 0he "la&ds of labia mi&ora
lubricate the vulva. It is formed by the
frenulum and the prepuce of the clitoris
which is also very se&sitive because it has
rich &erve supply#
URETHRAL MEATUS
;AGINAL INTR-ITUS
CLIT-RIS
-.&tra&ce of urethra,
ope&s appro1imately
1cm below clitoris
T?- GLANDS THAT LU1RICATE DURING SE@
1. SKENES GLANDS (Paraurethral Glands*A lubricates the
e1ter&al "e&italia
2. Bartholins Gland (Vulvovaginal Glands*A al'ali&e i& ph,
helps improve sperm survival
Doderleins BacillusA causes the va"i&al ph to be acidic, which
forms lactic acid
HymenA the elastic tissue, symboli2es vir"i&ity# 0hor& &
bloody duri&" forced se1ual act
!GAE" thic' folds of membra&ous stratified epitheliums o&
the i&ter&al wall of the va"i&a, capable of stretchi&" duri&" the
birth process, to accommodate the delivery of the fetus#
-3omposed of "la&s &
shaft that is partially
covered by prepuce
-456*S is small a&d
rou&d a&d is filled w/
ma&y &erve e&di&"s a&d
rich blood supply
-S7680 is a cord
co&&ecti&" the "la&s to
the pubic bo&e9 w/i& it is
the maor blood supply
of clitoris
Co(e%s an& $%otects ;ESTI1ULE
)i"'%e #9a Inte%nal St%'ct'%e
Ib. Inte%nal Genitalia
:8i"ure 1-a;
)/46* 8,*30!)*S S0/,30,/. *)0.S
,terus
+ear shape muscular
or"a& which has
three:3; mai&
fu&ctio&s
1# receive the ova
from the fallopia&
tube
2# provide a place for
impla&tatio& of the
ova
3# *ourishme&t for
fetal "rowth#
Di(isions of t!e 'te%'s
!# 3ervi1 < lowest portio& , 1/3 of the
total uterus
.1ter&al )s< where the &urse obtai&
the Pap Smear to the
SQUAMOCOLUMNA !UNC"ION
cells# 0his is where the cercla"e is
do&e for i&compete&t cervi1# *amely<
6# Shirod'ar -arter Suture- perma&e&t
closure of the internal cer#ical os,
u&til the 3=
th
wee' after which is
separated > "$A"M$N" %O
INCOMP$"$N" C$&I' and
P$&IOUS A(O"ION#
-# (c ?o&alds or +urse Stri&"
3ercla"e of the e1ter&al os< usually
*ormal spo&ta&eous delivery will be
do&e for the patie&t#
!!# !sthmus< shortest portion of the
uterus, the portio& that is cut )hen the
fetus is deli#ered durin* cesarean
birth#
!!!# 8u&dus< ,pper se"me&t, this is the
most vascular, the portio& also where
palpatio& is do&e# 6lso touchi&" it by
the tip of the fi&"ers duri&" co&tractio&
is the best method to determi&e the
i&te&sity of co&tractio&s duri&" labor#
Laye%s of t!e Ute%'sA
#. En&omet%i'mA
i&&er layer, most
vascular, S+$,
,UIN-
M$NS"UA"ION."+
$ NON.P$-NAN"
U"$US
,. Myomet%i'mA
LA-$S" PO"ION
$'P$LS "+$ %$"US
,UIN- "+$ (I"+
POC$SS. "he part
that contracts durin*
hemorrha*e. Pre#ents
hemorrha*e.
1# e%imet%i'm<
)uter most layer#
6ids for support &
added stre&"th#
=
(andl/s in* 0 Patholo*ical etraction
in*;< see& i& +rolo&"ed 5abor or
?ystocia
8allopia&
tubes
Site of fertili2atio& of
the ovum with perm
4 +arts of the 8allopia& tubes
1# !&terstitial < lies withi& the uteri&e
wall
2# Isthmus1 the portion that is cut or
sealed in "U(AL LI-A"ION 0 site for
sterili2ation;
3# Ampulla1 )here fertili2ation
occurs 3 this is also the LON-$S"
portion3 fre4uent site for ectopic
pre*nancy#
4# Infundibular1 co#ered by the
%imbriae cells that help *uide the o#a
to the %allopian "ube#
8allopia& tubes
tra&sport the ova from
the ovaries to the
uterus#
)varies )vulatio& :the release
of a& ovum;9 Steroid
hormo&e productio&
+air of follicle co&tai&i&" or"a&s o&
the other side of the uterus
-(a%iesA 4 by 2 cm i& diameter, 1#5
cm thic'# /espo&sible for
the productio&,
(aturatio&, a&d
dischar"e of ova
Secretion of estro*en
and pro*esterone
#orte$ o% the &varies' de#elopin*
and *raafian follicles are found here.
0he ovaries lie i& the
upper pelvic cavity#
@a"i&a )r"a& for coitus9
-irth ca&al9 3o&duit
for me&strual flow#
0ube e1te&di&" from the i&troitus to
cervi1
8ibromuscular or"a&
li&ed with mucus
membra&e
I c. Ty$es of el(ic Li"aments
1# Ro'n&A remai& la1 duri&" &o&-pre"&a&cy & become +5P$"OP+I$, & elo&"ated duri&"
pre"&a&cy#
2# Ca%&inalA chief uteri&e supports
3# 1%oa& li"amentsA drapes over the fallopia& tubes, uterus & ovaries
A
1B
I. 1 MALE RER-DUCTI;E S=STEM
E$ternal (eaturesA
, E%ectile Tiss'es in t!e $enisA
a# Corpus ca#ernosa
b# corpus spon*iosum
)nternal (eaturesA
E$i&i&ymisA totals 2B ft# 6+$$ SP$MS A$ S"O$,
;as / D'ct's Defe%ensA carries the sperm to the i&"ui&al ca&al
Seminal Glan& / ;esicle< Secretes S.(.*
%ost%ate Glan&< secretes S.(.* also#
Co<$e%s Glan&/ 1'lbo9'%et!%alA secretes also seme&
SEMEN so'%cesA ## prostrate "la&d < 6BC
2# Semi&al vesicles < 3BC
3# .pididymis < 5C
4# 3owpers < 5C
Accesso%y St%'ct'%es
)i"'%e #9b Mamma%y Glan&s
III. Mamma%y Glan&s
11
MAMMAR= GLANDS
-2 mammary "la&ds located o& each side of chest wall
-.ach breast 15-2B lobes co&tai&i&" clusters of 65@.)5!
ACINI
-Sacli'e e&d of
the "la&dular
system
-5i&ed both w/
epithelial cells
that secrete
colostrum0 )hic
h is rich in I*A7
& mil' & w/
muscles that
e1pel mil'
DUCTULES
-.1it alveoli & oi&
to form lar"er ca&als
5630!8./),S
?,30S
-?uri&" lactatio&,
mil' flows to the
alveoli a&d the& thru
the duct system
further "oi&" to the
balloo& li'e stora"e
sacs called
5630!8./),S
S!*,S.S
NILES
-Si&uses mer"e i&to
ope&i&"s o& &ipple
I;. )emale Re$%o&'cti(e Ho%mones
H-RM-NES
LUTENIBING H-RM-NE AND ESTR-GEN $ea2 imme&iately befo%e o('lation
(ost wome& ovulate two wee's before the be"i&&i&" of the &e1t period#
12
)ollicle Stim'latin"
Ho%mone
DStimulates
4raafia& follicle to
mature a&d resulti&"
i& i&crease levels of
estro"e&
L'teni3in"
Ho%mone
-%he& follicle is
ripe a&d mature,
tri""ers follicular
rupture a&d release
of ovum
-Pea8s at 9:.9;
hours before
ovulatio&#
-stimulates
o#ulation <
de#elopment of
corpus luteum
Est%o"en
-+roduce from ovaries,
adre&al corte1, a&d
place&ta
-6ssists i& maturatio& of
4raafia& follicle
-Stimulates thic'e&i&" of
e&dometrium#
-t!e% f'nctions
a. 3o&tracts smooth
muscles !&hibits the
secretio& of 8S7
b. /espo&sible for the
i&crease #a*inal
secretion in the #a*ina
0L$U=O+$A;
c. 0hic'e&s the
e&dometrium
d. SUPP$SS$S "+$
%S+ < Prolactin
e. /espo&sible for the
devEt of 2&dary se1
characteristics i&
females
f. Stimulates uterine
contractions <
smuscular peristalsis
of the fallopian tubes
for the passa*e of the
o#um to the uterus#
*. (ildly i&creases *a &
water reabsorptio&
h. Stimulates L+
secretion <
responsible for the
production of cer#ical
mucus associated in
fernin* < spinnbar8eit
%o"este%one
D+roduce from corpus
luteum, place&ta
-Secretes thic'/viscous
cervical secretio&s#
A.Preparation of the
uterus to recei#e a
fertili2ed o#um
(. ,ecrease uterine
motility>
contractility durin*
pre*nancy
C. Increases basal
metabolism
,. $nhances
placental *ro)th
$. Stimulates the
de#/t of acini cells
in the
breast0ma?or cells
for breast mil87
Increase the
endometriums
supply of
*lyco*en3 o@y*en
< amino acids for
maintainin*
pre*nancy
I; a. -t!e% Re$%o&'cti(e Ho%mones
#. Lactogenic *or+one (Prolactin*
-Stimulates lactatio&
,. Melanocyte Stim'latin" Ho%mone
-/espo&sible for the li&ea &i"ra & chloasma i& pre"&a&cy
-Secreted by the a&terior pituitary hormo&e (.56*)0/)+!*
-%ill e&d o& the 2
&d
mo&th of pre"&a&cy
3. *u+an #horionic Gonadotro,in
-!&creases i& &ausea a&d vomiti&"
esponsible for +yperemesis -ra#idarum
;. MENSTRUATI-N
Mena%c!eA 1
st
me&strual period, usually a"e 12, but may be"i& as early as A#
Meno$a'seA cessatio& of me&strual cycle that occurs &ormally from 4B & 55 y#o#
Menst%'al CycleA
1# (e&strual +hase : 1 > 14 days;
-3orpus luteum dies#
-+ro"estero&e & .stro"e& va&ishes- tri""ers/stimulate the productio& of 8S7#
-.&dometrium de"e&erated/ sheds- me&struatio& occurs#
Se@ual intercourse durin* menstruation is not harmful.
2# +roliferative +hase- .stro"e& +hase : 6 > 14 days; 4raafia& 8ollicle< .stro"e&
6&terior +ituitary 4la&d secretes 8S7 stimulates the developme&t of the -raafian
follicle 0secretes $stro*en7 suppresses %S+ < stimulates L+ L+ stimulates
o#ulation !&crease .stro"e& 'ills/decreases 8S7
3# Secretory +hase :15 to 21 days; +ro"estero&e +hase :3orpus 5uteum< +ro"estero&e;
)ther -oo's it is called< 5uteal +hase
6fter )vulatio&-----release of mature ovum from the 4raafia& follicle-----4raafia&
8ollicles die a&d replaced by 3orpus 5uteum-----secretes pro"estero&e 8u&ctio&s of
+ro"estero&e<
4# +re-(e&strual +hase :22 days to 2= days;
-!f fertili2atio& does &ot occur, corpus luteum be"i&s to die
-+ro"estero&e & .stro"e& decreases
-.&dometrium de"e&erates
-Menstruation stops durin* pre*nancy because there is decrease secretion of hormones by the
o#ary.
13
-;ARIAN C=CLE
:63)/?!*4 0) 7)/()*65 630!@!0F;
B $ 14 21 2=
)-LLICULAR HASE LUTEAL HASE
)varia& follicles mature u&der i&flue&ce -mittelshmer2
of 8S7 a&d estro"e& -cervical cha&"es
57 sur"e causes ovulatio& -i&crease --0
END-METRIAL/UTERINE C=CLE
:?escribed by varyi&" thic'&ess of the e&dometrium;
:)i"'%e #9c*
14
DE;EL-ING )-LLICLES -;ULATI-N C-RUS LUTEUM LUTEAL
REGRESSI-N
(.*S0/,65
+76S.
-(e&struatio&
-?ecrease estro"e&
-?ecrease
pro"estero&e
+/)5.8./60!@.
+76S.
-7ypothalamus
secretes 8S7
-6+4 :a&terior
pituitary "la&d;
secretes 8S7
-(aturatio& of
4raafia& follicle
-!&creased estro"e&
-7ypothalamus stops
8S7 & starts 57
-6+4 stops 8S7 &
starts 57 secretio&
S.3/.0)/F
+76S.
-8ormatio& of corpus
luteum
-!&crease
pro"estero&e
-*)
8./0!5!G60!)*9
corpus luteum
de"e&erates 1B days
after ovulatio&
-%!07
8./0!5!G60!)*9
co&cepts produces
734 that sustai&s
life corpus luteum9
pro"estero&e level is
mai&tai&ed at hi"h
level
-+ro"estero&e level
decreases
-3orpus albica&s
Slou"hi&" off of
e&dometrial li&i&"
+/.-
(.*S0/,65
+76S.
-e&dometrium
de"e&erates
)i"'%e #9c Menst%'al Cycle
;. a Menst%'al Diso%&e%s
?ysme&orrheal+reme&strual Sy&drome6me&orrhea(e&orrha"ia(etrorrha"ia- +rimary-
*o '&ow& cause
- Seco&dary-
(ay be caused by tumor/i&flammatory co&ditio&s-.dema of lower e1tremities
- 6bdomi&al bloati&"
- %ei"ht "ai&
- 7eadache
--reast te&der&ess
- ?epressio&
- 3ryi&"
- 5oss of co&ce&tratio& +rimary-
*ever me&struated9 structural/co&"e&ital ab&ormality
Seco&dary>3essatio& of me&struatio&-.1cessive or prolo&"ed bleedi&"- !rre"ular bleedi&" i& betwee& periods
;I. )AMIL= LANNING AND C-NTRACETI-N
)amily lannin" Met!o&s
0he most importa&t topic i& a +re&atal @isit is the ,AN-$ SI-NS IN P$-NANC5AA
?ischar"e pla&&i&" should start Bn the admission to the facility< to i&troduce to the commu&ity &
support servicesHHH
I1 !&itial /espo&sibility of a *urse i& +re"&a&t 6dolesce&ts is to impress the importa&ce of +re&atal
care< cause they are ofte& PON$ to PI+ devEt factors such as -: a"e, diet & lac' of pre&atal care;
-efore cou&seli&" a patie&t about co&traceptive methods, the &urse must< $&ALUA"$ +$ O6N
($LI$%S < &ALU$S $-A,IN- %AMIL5 PLANNIN-AAAA
15
Nat'%al o% )e%tility A<a%eness Met!o&s
A. Nat'%al Cont%ace$ti(es
1# (illin*s Method 0Cer#ical Mucus;< with ovulatio& :pea' day; the mucus becomes thi&
a&d watery, tra&spare&t, CL$A3 "+IN < $LAS"IC- avoid havi&" se1 i& this phase;#
S+!**-6/J.!0# -reatest %actor for (asal (ody "emperature ,IS"U(ANC$...)ill
be the presence of stress#
2# Calendar Method< to determi&e her 8./0!5!0F, subtract 1= days from the
S7)/0.S0 (.*S0/,65 3F35. & 11 days from her lo&"est cycle#
3# ,aily (asal (ody "emperature< will drop from B#2 > B#= de"rees 8ahre&heit duri&"
ovulatio& i& respo&se to +/)4.S0./)*.# ,on/t ha#e se@ on the 9
st
day of menses
unt:il C
rd
day of temperature ele#ation. (o&itor for at least 3 mo&ths before a&aly2i&" the
resultsHHHH
Most accurate readin*3 immediately after a)a8enin*3 before arisin*HHHH
4# Sympto thermal< mi1ture of 3ervical (ucus & -asal -ody 0emperature
5. Coitus Interruptus < oldest & least effective method#
Natural methods of birth control *enerally ha#e a hi*her failure rate because it depends on 8no)in* )hen the
o#ulation occurs3 since this is difficult to accurately determine3 the chance of miscalculation is hi*h#
"he determination of infertility is based on a*e. In a couple youn*er than CB years old3 infertility is defined as
failure to concei#e after 9 year of unprotected intercourse. In a couple a*e CB or older3 the time period is
reduced to : months of unprotected intercourse.
365.*?6/
(.07)?
-6S65 -)?F
0.(+./60,/.
3./@!365
(,3,S
(.07)?
SF(+)07.
/(65
(.07)?
(!00.5S37(./G 3)!0,S
!*0.//,+0S
D

/
e
K
u
i
r
e
s
w
i
t
h
d
r
a
w
a
l
o
f
t
/elies o& absti&e&ce from i&tercourse duri&" fertile period
16
h
e
p
e
&
i
s
f
r
o
m

t
h
e
v
a
"
i
&
a
b
e
f
o
r
e
D

3
o
u
p
l
e
m
a
'
e
s
u
s
e
o
f
c
1$
o
m
b
i
&
a
t
i
o
&

o
f
c
a
l
e
&
d
a
r
,
-
-
0
,
a
&
d

c
e
r
v
i
c
a
l
m
u
c
u
s
m
e
t
1=
h
o
d

t
o

d
e
t
e
r
m
i
&
e
f
e
r
t
i
l
e
p
e
r
i
o
d
D

-
e
t
w
e
e
&

m
e
&
s
t
r
u
1A
a
l
c
y
c
l
e
s
,
s
o
m
e
w
o
m
e
&

e
1
p
e
r
i
e
&
c
e
p
a
i
&

w
h
e
&

t
h
e
o
v
a
r
2B
y

r
e
l
e
a
s
e
s
e
"
"
D

,
s
e
s
t
h
e
a
p
p
e
a
r
a
&
c
e
,
c
h
a
r
a
c
t
e
r
i
s
t
i
21
c
s
a
&
d

a
m
o
u
&
t
o
f
c
e
r
v
i
c
a
l
m
u
c
u
s
t
o

i
d
e
&
t
i
f
y

o
v
u
l
a
t
i
22
o
&

e

a
c
u
l
a
t
i
o
&
D
/
a
r
e
l
y
a
c
c
o
m
p
a
&
i
e
d
b
y
s
c
a
&
t
v
a
"
i
&
a
l
s
p
o
t
t
i
23
&
"
D
S
o
m
e
c
o
u
p
l
e
u
s
e
s
t
h
i
s
a
s
s
i
"
&
a
l
o
f
t
h
e
b
e
"
i
&
&
i
&
"
p
e
r
i
o
d
a
&
d
t
o
a
v
24
o
i
d
s
e
1
u
a
l
i
&
t
e
r
c
o
u
r
s
e
u
&
t
i
l
t
h
e
f
e
r
t
i
l
e
p
e
r
i
o
d
p
a
s
s
e
s
)v
u
l
a
t
o
r
y
<
25
c
e
r
v
i
c
a
l
m
u
c
u
s
i
s
c
l
e
a
r
a
&
d
a
b
u
&
d
a
&
t
+re
-
o
v
u
l
a
t
o
r
y
/
p
o
s
t
o
v
u
l
a
t
o
r
y
26
<
c
e
r
v
i
c
a
l
m
u
c
u
s
i
s
y
e
l
l
o
w
i
s
h
,
l
e
s
s
a
b
u
&
d
a
&
t,
a
&
d
s
t
i
c
'
y
:
i
&
h
i
b
i
t
s
2$
p
e
r
m

m
o
t
i
l
i
t
y
;
.
D (easured by ta'i&" & recordi&" e temperature rally rectally each mor&i&" before wa'i&"
after at least 3 hours of sleep
1. 1a%%ie% Met!o&s
IUD)EMALE C-ND-M :;AGINAL -UCH*
(65. 3)*?)(

8i"ure 1-d 3o&dom

NCLE@ TISCC
T!e female con&om &'%in" se+
)i"'%e #9&
?uri&" se1 the pe&is is i&serted i&to the ce&ter of the ope& ri&" at the ope&i&" of the va"i&a# ,&til both part&ers
are familiar with the /eality co&dom, the pe&is should be "uided by ha&d i&to the ope& ri&"# )therwise there is
the cha&ce that the pe&is will be i&serted outside the co&dom i&to the va"i&a, thus defeati&" the co&domLs
purpose# ,se of the male co&dom with the female co&dom is &ot recomme&ded, because rubbi&" the late1 male
co&dom a"ai&st the polyuretha&e female co&dom creates frictio& that may ma'e i&tercourse difficult#

Remo(in" t!e female con&om
2=
0he female co&dom should be removed followi&" i&tercourse a&d before sta&di&" up# 0o remove, sKuee2e a&d
twist the outer ri&" to e&sure that seme& remai&s i&side the co&dom# 4e&tly pull the co&dom from the va"i&a#
?iscard i& the trash# ?o &ot attempt to flush the co&dom dow& the toilet, as it may clo" the toilet or sewer li&es#
?o &ot reuse#

Im$o%tant $oints to %emembe% <!en 'sin" t!e female con&om
- "he female condom )or8s only if you use it e#ery time you ha#e se@.
. Use a ne) condom each time you ha#e se@ual intercourse. Do not reuse the %e+ale condo+.
. 5ou can still become pre*nant and transmit or ac4uire a se@ually transmitted disease )hile usin* the female
condom. "he ris8 is less than if you do not use the condom3 but there still is a sli*ht ris8.
. Althou*h the eality condom is prelubricated3 it also comes )ith a tube of lubricant in the pac8a*e. 5ou may
)ish to add a fe) drops of lubricant to the openin* of the condom or to the penis. Lubricants reduce friction
and noise those results from friction.
. emo#e tampons before insertin* the female condom.
- !se caution to avoid tearing the %e+ale condo+ .ith a shar, %ingernail/ ring/ or other 0e.elr1 .hen
inserting and re+oving the condo+.
CER;ICAL CA ;S DIAHRAGM
?
!
6
+
7
/
6
4
(

:
8
i
"
u
r
e
1
-
e
;
CHARACTERISTICS
)
l
e
+
i
b
l
e
%
i
n
"
?.S3/!+0!)*
2A
c
o
(
e
%
e
&
<
i
t
!
&
o
m
e
s
!
a
$
e
%
'
b
b
e
%
c
a
$
3
.
/
@
!
3
6
5

3
6
+
Small rubber plastic that
fits s&u"ly over cervi1
.88.30!@!0F
*,55!+6/6M=BC
(50!+6/6M6BC
=BC with typical use
)
&
t
w
o
h
o
u
r
s
p
,S64.
3B
r
i
o
r
t
o
s
e
1
u
a
l
i
&
t
e
r
c
o
u
r
s
e
a
&
d
i
&
p
l
a
c
e
f
o
r
6
h
o
u
r
s
a
f
t
e
r
3o&ti&uous protectio& 24
hours re"ardless of the
&umber of times of se1ual
i&tercourse
S+./(!3!?.
*ot &ecessary for repeated
coitus
,se every coitus
S!?. .88.30S
3ervicitis 3ystitis, cramps, rectal prolapsed
"o@ic Shoc8 syndrome 0"SS7
31
7)% 0) !*S./0
8itted by health provider Same, refitted after birth a&d wei"ht loss of
15lbs
?,/60!)*
*ot lo&"er tha& 4= hours *ot lo&"er tha& 24 hours
A diaphra*m should be left in the #a*ina
:.; hours after se@ual intercourse.

Dia,hrag+A should remai& i& place 6-= hours after se1 & maybe left for 24 hours#
AL6A5S C+$C= %O "$AS < +OL$SAAA
#ontraindicated %or" %re4uent U"I3 Prolapsed Cord < etro#erted Uterus3 cystocele < rectocele3 acute
cer#icitis

)i"'%e 1-e ?iaphra"m
C. !a%macolo"ic met!o&s
&ral #ontrace,tive Pill A sy&thetic estro"e& combi&ed with small amou&ts of sy&thetic pro"estero&e-
preve&ti&" ovulatio& by stoppi&" 8S7 & 57#
- Stops L+ < %S+
ST- I) ?ITH THE ))< :637.S;
- A. abdominal pain3 #- Chest pain3 *. +eadaches3 E. eye problems < S.se#ere le* cramps
- A""N1 Se#ere +eadaches maybe an indication of +ypertensionAAAA
C-NTRAINDICATEDA
9 "hromboembolism
D C&A3 +PN3 smo8in* < diabetics3,IC3 hyper#iscosity
#ontraindicated %or D)ABE2)#S. "he best for diabetics are (arrier Contracepti#es..Condom <
,iaphra*m
E$a+,les" ?emule& :.thi&yl .stradiol .thyl&odiol ; a mo&ophasic oral co&traceptive a"e&t.
If the patient for*ets to ta8e D tablets for the ne@t D days3 she should ta8e D tablets N$'" D ,A5SAAA
And use another contracepti#e method for the rest of the cycle.
If she misses C or more3 she should discard the remainin* tablets < use another contracepti#e
method for the rest of the cycle.
)/65 3)*0/3.+0!@.S (!*!+!55S S,-?./(65
!(+56*0S
S,-3,06*.),S
!*N.30!)*S
(edro1ypro"estero&e :?(+6 or ?.+)@./6;Si1 soft sillastic rods filled with sy&thetic pro"estero&e
impla&ted i&to the woma&Es arm,se to preve&t co&ceptio& by i&hibiti&" ovulatio& :i&hibits release of 8S7 a&d
57;
+ro"estero&e lea's i&to the blood stream, i&hibiti&" impla&tatio& i&to e&dometrium
*orpla&t
!&serted subdermally i&to the midportio& of the upper arm about =-1Bcm above the elbow crease# 6 impla&table
capsules are i&serted at o&e time
3auses atrophic cha&"es i& the e&dometrium to preve&t impla&tatio& of e""
3auses thic'e&i&" of cervical mucus to i&hibit sperm travel
32
Under ideal conditions the sperm can reach the o#um 9 to E minutes after e?aculation.
3ombi&ed estro"e& a&d pro"estero&e preparatio& i& tablet form a&d are ta'e& daily with combi&atio&s of
hormo&es
Oral contracepti#es pre#ent pre*nancy by suppressin* %S+ 0follicle stimulatin* hormone7 and L+ 0leuteni2in*
hormone7 release from the pituitary *land thereby bloc8in* o#ulation.
+ills co&tai& pro"esti& but &o estro"e&
Birth #ontrol Su++ar1 2a3le
-!/07 3)*0/)5 (.07)?6?@6*064./!SJS )/ +)SS!-5. +/)-5.(S
O )&ly partially effective a"ai&st se1ually tra&smitted disease :S0?; tra&smissio&Spermicides< chemicals i&
the form of foams, creams, ellies, films, or suppositories that are i&serted i&to the va"i&a to 'ill sperm before
they ca& e&ter the uterus9 typical use effective&ess< $BC
O +ossible aller"ies or irritatio&
O +ossible aller"ies to late1 or spermicide3o&dom< male co&dom is a sheath of late1 or a&imal tissue placed
o& erect pe&is9 female co&dom is a plastic sac with a ri&" o& each e&d i&serted i&to the va"i&a9 both may be
used with a spermicide9 typical use effective&ess< =4C :male; $AC :female;O 6vailable over the cou&ter
O 5esse&s se&satio&
O (ay brea' duri&" i&tercourse
#A#oid usin* petroleum ?elly of oil base productsF it can cause INC$AS$ %IC"ION )hich )ill lead to
"$AIN- O% "+$ LA"$' CON,OM#
O 3a& be used with other methods to improve effective&ess
O
.
ff
e
ct
iv
e
a
"
ai
&
st
S
0
?
tr
a
&
s
m
is
si
o
&
O
6
v
?iaphra"m< shallow late1 cup with fle1ible rim i&serted i&to
va"i&a over cervi1 to preve&t sperm from e&teri&" uterus9 used
with spermicide9 typical use effective&ess< =2C
O /eusable
O 3a& last for
o&e to two
years
O *ot effective a"ai&st
S0? tra&smissio&
O *eeds to be fitted by a
health care professio&al
O !&creased ris' of
bladder i&fectio&
O +ossible aller"ies to
late1 or spermicide
33
ai
la
bl
e
o
v
er
th
e
c
o
u
&t
er
O
3
a
&
b
e
u
s
e
d
w
it
h
ot
h
er
m
et
h
o
d
s
to
f
u
rt
h
er
p
r
ot
e
34
ct
a
"
ai
&
st
S
0
?
3ervical 3ap< thimble-shaped late1 cap i&serted i&to va"i&a over
cervi1 to preve&t sperm from e&teri&" uterus9 used with
spermicide9 typical use effective&ess< =2C
C$&ICAL CAP< ca& be retai&ed upto 4= hours# !t does &ot
lea'# 3a&&ot be re-applied a"ai& after use# (ay use spermicide
before use#
O /eusable
O 3a& last for
o&e to two
years
O *ot effective a"ai&st
S0? tra&smissio&
O *eeds to be fitted by a
health care professio&al
O ?ifficult to fit wome&
with a& u&usual cervi1
si2e
O ?ifficult for some
wome& to i&sert
-irth 3o&trol +ill< prescriptio& dru" co&tai&i&" female
hormo&es9 o&e pill ta'e& daily preve&ts ovaries from releasi&"
e""s a&d/or thic'e&s cervical mucus to preve&t sperm from
reachi&" e""9 typical use effective&ess< A4C
O (ore
re"ular
periods
O *o actio&
reKuired
prior to
se1ual
i&tercourse,
permits
se1ual
spo&ta&eity
O Some
protection
a*ainst
o#arian
and
endometria
l cancer3
noncancero
us breast
tumors3
o#arian
cysts
O *ot effective a"ai&st
S0? tra&smissio&
O are but dan*erous
complications3
includin* blood
clottin* and
hypertension3
particularly in )omen
o#er CE years )ho
smo8e
O (ust be ta'e& daily
O *ot effective a"ai&st S0? tra&smissio&7ormo&al !mpla&t :*orpla&t;< si1 small capsules
i&serted by a health care professio&al u&der the s'i& of upper arm that deliver small
amou&ts of hormo&e to preve&t ovaries from releasi&" e""9 typical use effective&ess< AAC
O +ossible scarri&" or, rarely, i&fectio& at i&sertio& site
O Side effects i&clude irre"ular bleedi&", headaches, &ausea, depressio&
O *ot effective a"ai&st S0? tra&smissio&7ormo&al !&ectio& :?epo-+rovera;< in?ection *i#en by a health care
professional in the arm or buttoc8 e#ery 9D )ee8s to preve&t ovaries from releasi&" a& e"" a&d/or thic'e&
cervical mucus to 'eep sperm from reachi&" a& e""9 typical use effective&ess< AACO +rotects a"ai&st
pre"&a&cy for up to five years
O Side effects i&clude irre"ular bleedi&", wei"ht "ai&, headaches, depressio&, abdomi&al
35
pai&
O Side effects
do &ot reverse
u&til
medicatio&
wears off
O (ay cause
delay i&
becomi&"
pre"&a&t after
i&ectio&s are
stopped
O *o actio&
reKuired prior to
se1ual
i&tercourse,
permits se1ual
spo&ta&eity
O Can be used
)hile breast.
feedin*
be*innin* si@
)ee8s after
deli#erin* baby
O +rotects
a"ai&st
pre"&a&cy for
12 wee's
O *o actio&
reKuired prior to
se1ual
i&tercourse,
permits se1ual
spo&ta&eity
O Can be used
)hile breast.
feedin*
be*innin* si@
)ee8s after
deli#erin* baby
O +rotects
a"ai&st ca&cer
of the uteri&e
li&i&" a&d iro&
deficie&cy
a&emia
O *ot effective a"ai&st S0? tra&smissio&O +erma&e&t
protectio& from pre"&a&cy0ubal 5i"atio&< sur"ical
O +erma&e&t
protectio&
O *ot effective a"ai&st
S0? tra&smissio&
!&trauteri&e ?evice :!,?;< small device i&serted by a health care professio&al i&to the uterus9 preve&ts e""s from
bei&" fertili2ed a&d/or impla&ti&" i& uterus9 typical use effective&ess< A6C
Intra.uterine ,e#ices 0IU,;- a small plastic obect is i&serted i&to the uterus where it remai&s i& place# !t
i&terferes with the ability of the ovum to develop as it tra&sverses the fallopia& tube#
Most %re4uent Side $ffect<
a# $@cessi#e Menstrual flo) 0menorrha*ia7 b. Spontaneous $@pulsion of the de#ice1 Myometrium irritability
c. Crampin* < fe#er
Contraindications1
1# +istory of PI,1 a )oman usin* IU, has EBG chance of *ettin* PI,.
D. $ctopic Pre*nancy3 AI,S
Ne#er use > *i#e IU, to NULLIPAOUS 6OM$NAAA
eturn to the clinic for e#aluation after her 9
st
mensesAAA

8i"ure !&tra uteri&e device :!,?;
O .ffective o&e to si1 years, depe&di&" o& type used
O *o actio& reKuired prior to se1ual i&tercourse, permits se1ual spo&ta&eityO *ot effective a"ai&st S0?
tra&smissio&
O (ay cause spotti&" betwee& periods a&d lo&"er, heavier periods
O Increased ris8 of pel#ic inflammatory disorder0PI,7 )ithin first four months after insertionO /are ris' of uteri&e
perforatio&
36
procedure to perma&e&tly bloc' woma&Ls 8allopia&
tubes to preve&t e""s from reachi&" sperm9 typical
use effective&ess< AAC
O /eactio&s to sur"ery may i&clude i&fectio&, blood
clot &ear testes, bruisi&", swelli&", or te&der&ess of
scrotum
O !rreversible
O *o actio& reKuired prior to se1ual i&tercourse, permits se1ual
spo&ta&eity
"ubal li*ation1 isthmus part in the fallopian tube is the usual
part bein* li*hted#
from
pre"&a&cy
O *o actio&
reKuired
prior to
se1ual
i&tercourse,
permits
se1ual
spo&ta&eity
O /eactio&s to sur"ery
may i&clude i&fectio&,
bleedi&", i&ury to
i&testi&e, reactio& to
a&esthesia
O !&creased cha&ce of
ectopic pre"&a&cy
O !rreversible
@asectomy< sur"ical procedure to perma&e&tly bloc'
the maleLs vas defere&s to preve&t sperm from
reachi&" e""s9 typical use effective&ess< AAC
Sur*ical sterili2ation of the male in#ol#es cuttin* the ductus
deferens.
&asectomy1 &as ,eferens is cut. "he man can resume se@
after one )ee8 or )hen the sperm count indicates B count or
D ne*ati#e sperm count ha#e been e@amined.
4e&erally it reKuires 6 > 36 eaculatio&s to re&der &e"#
sperm cou&t

In order to *et for semen analysis3 collect them in a clean


*lass not plastic3 because it may affect the spermato2oa. No
se@ for C days before the semen collection < no drin8in* of
alcohol for 9 day. "he first portion of the semen has a hi*h
ration of sperm.


Section II
Ante$a%t'm e%io&
I. Assessment of Ris2 )acto%s in t!e %enatal e%io&
Age o% Pregnant 4o+en 91$ below< 7ave a hi"her i&cide&ce of
1# +rematurity
2# +re"&a&cy !&duced 7yperte&sio&
3# 3ephalopelvic ?isproportio&
4o+en over 56 1ears old are at is7 %or"
1# 3hromosomal ?isorders i& i&fa&ts
2# +!7
*atural 8amily +la&&i&"< tech&iKues, i&cludi&" chec'i&" body
temperature or cervical mucus daily or recordi&" me&strual
cycles o& a cale&dar, to determi&e the days whe& body is most
fertile9 typical use effective&ess< =1C
O *o medical
or hormo&al
side effects
O !&e1pe&sive
O 6ccepted by
most
reli"io&s
O *ot effective a"ai&st
S0? tra&smissio&
O /eKuires strict
record'eepi&"
O !ll&ess or lac' of sleep
may affect body
temperature
O @a"i&al i&fectio&s a&d
douches may affect
cervical mucus
O /eKuires absti&e&ce
from se1ual i&tercourse
or alter&ative
co&traceptio& duri&"
fertile days
3$
3# 3esarea& ?elivery
%imi"%a(i&a 9 1st time +re"&a&cy
%imi$a%a 9 1
st
delivery of a live i&fa&t,
N'lli"%a(i&a 9 &ever bee& pre"&a&t

)n%ections" !se 2&#*
T - 0o1oplasmosis
- - )ther i&fectio&s
R - /ubella
C - 3ytome"alovirus
H - 7erpes
6# 0o1oplasmosis :proto2oa;
+roduces symptoms of acute, flu-li'e i&fectio& i& mother
0ra&smitted throu"h raw meat or ha&dli&" cat litter of i&fected cats
S$ontaneo's abo%tion li2ely to occ'% ea%ly in $%e"nancy
-# /ubella
E$tre+el1 teratogenic in %irst tri+ester
#auses congenital de%ects o% e1es/ heart/ ears/ and 3rain
4o+en .ith lo. ru3ella titers should 3e vaccinated at least 5 +onths 3e%ore 3eco+ing ,regnant or %ollo.ing
a deliver1
N&2E" Any )oman in the first trimester of pre*nancy is at ris8 if e@posed to rubella. Con*enital %etal defects
often results from such an infection.
3# 3ytome"alovirus :3(@;
#+roduces flu-li'e or mo&o&ucleosis-li'e symptoms i& the mother
0ra&smitted throu"h the respiratory or se1ual route
(ay cause fetal death, retardatio&, heart defects, deaf&ess
*o effective treatme&t available
?# 7erpes Simples
6ffects the e1ter&al "e&italia, va"i&a, a&d cervi1
3auses drai&i&", ,ain%ul vesicles
?elivery of the fetus is usually by cesarea& sectio& active lesio&s are prese&t i& the va"i&a9 delivery may be
performed va"i&ally if the lesio&s are i& the a&al, peri&eal, or i&&er thi"h area :strict precautio&s are
&ecessary to protect the fetus duri&" delivery;
*o va"i&al e1ami&atio&s are do&e i& the prese&ce of active va"i&al herpetic lesio&s
8aintain #&N2A#2 isolation ,rocedures during hos,itali9ation i% the disease is active
*eo&ate a&d mother may be se$a%ate& &'%in" t!e acti(e $e%io&, or other special precautio&ary measures
may be used to avoid tra&smissio& to &eo&ate
2eratogenic Drugs" BASA-&(code:
1 9 -arbiturates
3=
A 9 6&ti-malarial
S 9 Salicylates
A 9 6&esthetic
- 9 -ral hypo"lycemics
Su3stance A3useA
Alcohol" causes lear&i&" disabilities, (o&"olism, fetal alcohol sy&drome
NicotineA i&creases vasoco&strictio&, retardatio&, S46 :small "estatio&al a"e;, low birth wei"ht
*eroin addictA babies are bor& with an $'A--$A"$,> +5P$AC"I&$ CNS > $%L$'$S or
CNS II"A(ILI"5.
#occaine" "he effect of cocaine in a labor and the fetus is preterm labor thus increased uterine
contractions3 intrauterine *ro)th retardation and the potential for a sic83 addicted infant


II. !ysiolo"ical #hanges in Pregnanc1
Inc%eases &'%in" $%e"nancy
!&crease 7eart /ate for 1B-15 beats/mi&ute
!&crease 3ardiac )utput for 2BC - 3BC duri&" 1
st
> 2
&d
trimester to meet i&crease tissue
dema&d
!&crease secretio& of su"ar :4lycosuria;
INC$AS$ PLAMA &OLUM$
!&crease ,ri&ary 8reKue&cy due to pressure to bladder#
!&crease &ormal depe&de&t .dema :bilateral or a&'le edema; &ormal for 36 wee's "estatio&#
Dec%eases &'%in" $%e"nancy
?ecrease :sli"htly of blood pressure; i& the 2
&d
trimester due to decrease peripheral resista&ce
?ecrease 7emo"lobi& & 7ematocrit because of !ro& ?eficie&cy 0Pseudo. AN$MIA;
?ecrease "astroi&testi&al motility & peristalsis due to displaceme&t of the i&testi&e & compressio&
of the stomach# ---leadi&" to 3)*S0!+60!)*#
?ecrease ,ri&e Specific "ravity< a result of i&crease ,ri&ary )utput#
-t!e%sA
#hloas+a " Mas8 of pre*nancy
Leu7orrhea" )hitish #a*inal dischar*e )ithout si*ns of inflammation < itchin*.
&,erculu+" formation of mucus plu* in C$&I' to seal out bacteria.
Lordosis" the Pride of Pre*nancy
ela$in" responsible hormone for the softenin* of the pel#ic cartila*es. Produce by the corpus luteum3
contributes to the )addlin* *ait typically noted in pre*nancy.
Nor+al deliver1 3lood loss" CBB H IBB ml of blood
#esarean Section" ;BB H 9BBB ml
II a. Ante$a%t'm Healt! %omotion
%enatal ;isit
Sc!e&'le of (isit if <it! no com$licationsA
a. E(e%y . <ee2sD '$ to 3, <ee2s
b. E(e%y , <ee2sD f%om 3,934 <ee2s :mo%e f%eE'ently if $%oblems e+ist*
c. E(e%y <ee2 f%om 349.8 <ee2s
3A
Classifications of %e"nancy
GRA;IDA > &umber of times pre"&a&t, re"ardless of duratio&, i&cludi&" prese&t pre"&a&cy#
RIMIGRA;IDA > pre"&a&t for the first time#
ItJs important for the nurse to distin*uish bet)een a client )hoJs ha#in* her first baby and one )ho has already
had a baby. %or the client )hoJs pre*nant for the first time3 4uic8enin* occurs around DB to DD )ee8s. 6omen
)ho ha#e had children )ill feel 4uic8enin* earlier3 usually around 9; to DB )ee8s3 because they reco*ni2e the
sensations.
MULTIGRA;IDA > pre"&a&t for seco&d or subseKue&t time#
ARA > &umber of pre"&a&cies that lasted more tha& 2B wee's#
NULLIARA > a woma& who has &ot "ive& birth to a baby beyo&d 2B wee's "estatio&#
RIMIARA > a woma& who has "ive& birth to o&e baby more tha& 2B wee's "estatio&#
MULTIARA > a woma& who has had two or more births at more tha& 2B wee's "estatio&#
Note1 ")ins or triplets counted as 9 para#
PE2E8 H ne)born born before CK )ee8s of *estation.
TERM > &ewbor& bor& after 3$ wee's to 4B wee's of "estatio&#
-ST9TERM > &ewbor& bor& after 4B wee's of "estatio&#
a%ity :TAL*
T 9 *umber of terms births,
9 *umber of premature births,
A 9 *umber of 6bortio&s,
L 9 *umber of livi&" childre&
NUTRITI-N
1
st
2ri+ester" D HI lbs *ain > CB.CE calories>8*>day
2
nd
tri+ester" 9 lb per )ee8 > DBB calories>8*>day
5
rd
tri+ester" 9 lb per )ee8> DBB calories>8*>day
%e"nant ?omen nee&s 5;; e$tra calories PE DA< fo% a&eE'ate n't%ition.
A &iet of 26;; calories ,er da1
An inc%ease of abo't 6;; calories ,er da1 is nee&e& &'%in" LA#2A2)&N.
)ron De%icienc1 Ane+ia is a %es'lt of P)#A.
Diffe%ent ty$es of E+e%cises
Pelvic (loor #ontractions (Kegel=s E$ercise*A +romotes peri&eal heali&", i&crease se1ual
respo&sive&ess, press stress i&co&ti&e&ce# ?o&e 5B-1BB times# .1amples< 0i"hte&i&" &
stre&"the&i&" the muscles of the @a"i&a, rectum, peri&eum & the& rela1 after# .fficie&t for
,ri&ary 8reKue&cy & 7emorrhoids# !&crease elasticity of the Pubococcy*eus muscle#
A3do+inal +uscle #ontractionsA pre#ent constipation i& pre"&a&cy, do&e i& sta&di&" or lyi&" positio&,
stre&"the&i&" the abdomi&al muscles#
Pelvic oc7ingA elie#es bac8ache duri&" pre"&a&cy, do&e by ti"hte&i&" the buttoc's & flatte&s the
lower bac' a"ai&st the floor for o&e mi&ute#
DI))ERENT T=ES -) 1REATHING TECHNIFUES
6# 6bdomi&al breathi&" : duri&" late&t phase of Sta"e 1 5abor;
1# ,sed u&til labor is more adva&ced
4B
2# 0he abdome& moves outward duri&" i&halatio& a&d dow&ward duri&" e1halatio&
3# 0he rate remai&s slow, with appro1imately si1 to &i&e breaths per mi&ute
-# +a&t-pa&t-blow: duri&" 0ra&sitio&al +hase of Sta"e 1 5abor;
1# ,sed i& adva&ced labor
2# 6 more rapid patter&, co&sisti&" of two short blows from the mouth followed by a lo&"er blow
3# 6ll e1halatio&s are a blowi&" motio&
III. )e%tili3ation to Conce$tion
)e%tili3ationA the u&io& of the ovum & sperm# 0he start of (itotic cell divisio& < fetal se@
determination.
P +rimary oocyte :immature ovum; co&tai&s ?iploid &umber of chromosomes :46;#
P )&e oocyte co&tai&s a haploid :23; &umber of chromosomes after divisio&#
P 4amete :mature ovum;< is a cell or ovum that has u&der"o&e (aturatio& & will be ready for
fertili2atio&#
P )&e "amete carries 23 chromosomes#
P 6 sperm carries 2 types of se1 chromosomes# Q & F#
P 4BB millio& sperm cells i& o&e eaculatio&#
P 8u&ctio&al 5ife of spermato2oa is 4= hours
P QQM female, QFM male#

)i"'%e #9) Mo%'la
%ocess of )e%tili3ationA
6fter ovulatio& ovum will be e1pelled from the 4raafia& follicles ovum will be surrou&ded by Lona
Pellucida :mucopolysaccharide fluid; & a circle of cells 0Corona adiata; which i&creases the bul' of the
)vum e1pelled from the 8allopia& 0ube by the 8imbriae :i&fu&dibulum;# Sperms move by fla"ella &
+e&etrate the & dissolve the cell wall of the ovum by releasi&" a proteolytic e&2yme
:+yaluronidase7 6fter pe&etratio& 8usio& will result to Ly*ote# Gy"ote mi"rate for 4 days i& the
body of the uterus :(itosis will ta'e place-3leava"e formatio& will be"i&; 6fter 16-5B cell formatio& from
mitosis, a mulberry & -umpy appeara&ce will follow morula 0%igure 1-(7 ---after 3-4 days, the structure will
be ball li'e i& appeara&ce which will be called (lastocyst# 3ells i& the outer ri&" are called "rophoblast :later it
forms the place&ta, respo&sible for the devEt of place&ta & fetal membra&e9 3ells i& the i&&er ri&" are called
$rythroblas t cells :which will be the embryo;#
Te%ms to %emembe%A
&vu+A 8rom ovulatio& to fertili2atio&
>1goteA 8rom fertili2atio& to impla&tatio&
E+3r1oA 8rom impla&tatio& to 5-= wee's#
(etus" 8rom 5-= wee's u&til term
"he o#um is said to be #iable for DI.36 hours#
Sodium (icarbonate. the freKue&t medicatio& to alter the va"i&al ph, decrease the acidity of the
va"i&a so as to !*3/.6S. 07. ()0!5!0F )8 07. S+./(#

)i"'%e #9G )etal Memb%anes
)etal Memb%anes< membra&es that surrou&d the fetus, & "ive the place&ta the shi&y appeara&ce#
41
:)i"'%e #-4;
2 5ayers<
a. A+nion< shi&y membra&e o& the 2
&d
wee' of .mbryo&ic ?evelopme&t & e&closes the 6m&iotic
3avity
b. #horion1 )uter membra&e that supports the sac of the am&iotic fluid#
#horionic Villi1 fin*er li8e pro?ections from the chorion. "his is the place )here *ases3 nutrients and
)aste products bet)een the maternal < fetal blood ta8es place.
Amniotic )l'i&< surrou&ds the embryo, co&tai&s fetal uri&e, la&u"o from fetal s'i& & epithelial cells#
+h is $# 2# Specific 4ravity< 1#BB5 > 1#B25
Normal Amount1 EBB H 9BBB ml.
Oli*ohydramnios. less than CBB ml.
Polyhydramnios. more than DBBB ml. obser#e for ,o)n syndrome < con*enital defects
8u&ctio&s of 6m&iotic 8luid<
a# +rotects the fetus from cha&"es i& the temperature & cushio& a"ai&st i&ury#
b# +rotects the umbilical cord from pressure, the fetus dri&'s & breaths the fluid
i&to the lu&"s#
Amniotic )l'i& Colo%s< *ormal color< tra&spare&t, clear, with white ti&y spec's
,ar8 amber or yello)< )mi&ous si"& of prese&ce of -ilirubi&, hemolytic disease
Port 6ine Colored< 6bruptio +lace&ta
-reenish1 (eco&ium Stai&ed / 8.065 ?!S0/.SS< always "o for Cesarian SectionH 6lso if ph is
less than K.D
If )ith odor< deliver withi& 24 hours, may i&dicate i&fectio&#
Umbilical Co%&A 21 i&ches i& le&"th & 2 cm i& thic' &ess, circulatory commu&icatio& of the fetus to the
mother# 3)*06!*S 2 6/0./!.S & 1 @.!*# 3overed by a "elati&ous mucopolysaccharide called
6hartons ?elly.
!mpla&tatio& occurs at the e&d of the 1st wee' after fertili2atio&, whe& the blastocyst attaches to the
e&dometrium# ?uri&" the 2&d wee' :14 days after impla&tatio&;, impla&tatio& pro"resses a&d two "erm layers,
cavities, a&d cell layers develop# ?uri&" the 3rd wee' of developme&t :21 days after impla&tatio&;, the
embryo&ic dis' evolves i&to three layers, a&d three &ew structures R the primitive strea', &otochord, a&d
alla&tois R form# .arly duri&" the 4th wee' :2= days after impla&tatio&;, cellular differe&tiatio& a&d
or"a&i2atio& occur#

)i"'%e #9H )e%tili3ation Cycle
0able Summary from 8ertili2atio& to !mpla&tatio& :8i"ure 1-7;
42
+/.-8./0!5!G60!)*
630!@!0!.S
)vum moves to amulla of
fallopia& tubes
3apacitatio&
6crosome reactio&
3)*3.+0!)*
Go&a reactio&
Gy"ote :fertili2ed ovum9
about 24-4= hrs, divides9
cleava"e divides, travels to
the uterus
!(+56*060!)*
(orula :after 3-4
days impla&tatio&;
-lastocyst
:trophoblast9
embryolast;
!mpla&ts complete
w/& $-1B days
III.a -RIGIN -) 1-D= TISSUE
2issue La1er Bod1 Portion (or+ed
.30)?./( *ervous system, mucus membra&es, a&us & mouth
(esoderm 3o&&ective 0issue, /eproductive, circulatory & upper
,ri&ary system, bo&es, cartilla"e
.&doderm li&i&" of the 4! tract, /espiratory 0ract, bladder & urethra
MULTILE REGNANCIES
Dou3le ovu+ Single &vu+
?i2y"otic/frater&al twi&s (o&o2y"otic/ide&tical twi&s
)va from same or differe&t ovaries u&io& of a si&"le ovum & a si&"le sperm
Same or differe&t se1 same se1 o&e place&ta
2 place&tas but maybe fused
2 chorio&s & 2 am&io&s o&e chorio& & 2 am&io&s
Genetics"
!enoty$eA !&dividualEs outward appeara&ce
Genoty$e< !&dividuals 4e&etic (a'e up
Ga%yoty$eA +ictorial a&alysis of i&dividualEs chromosomes
Se%oty$eA a&ti"e&ic character S6-)T
Genetic Disorders"
Autoso+al ecessive DisordersA both me& & wome& are at eKual ris' because the ?.8.30!@. 4.*.
is a& 6,0)S)(.< o&e of 22 pairs of &o&-se1 chromosomes# )ffspri&" of each pre"&a&cy
has a 25C cha&ce of bei&" affected a&d 5BC cha&ce of bei&" a carrier#
E$a+,les are" P=U 0 phenyl8etenuria7 3 "ay . Sachs ,isease3 Cystic %ibrosis3 "hallasemia3
and Sic8le Cell Anemia
Autoso+al Do+inantA a& affected offspri&" has a& affected pare&t#
E$a+,les are" +untinton/s Chorea and Marfan/s Syndrome 0Arachnodactyly7
?-lin7ed do+inant@ecessive Diso%&e%s< ab&ormal "e&e is fou&d o& the Q chromosome because me&
have o&ly o&e Q chromosome, they always e1press the disorder#
E$a+,les are1 +emophillia and ,uchenne Muscular ,ystrophy
I;. )ETAL DE;EL-MENT

)i"'%e #9 H, )etal De(elo$ment
6
mos# /
.mbryo is 4-5 mm le&"th
0rophoblasts embedded i& deciduas
43
21-25
wee's
21-25
%..
JSU
)5?
(6*
Es
863.
5
mos# /
1$-2B
wee's
8etus
is
15B-
1AB
mm#
!&
le&"th
a&d
wei"h
s
appro
1imat
ely
26B-
46B
"ms#4
mos# /
13-16
wee's
A4-
14B
mm
le&"th
a&d
wei"h
s A$-
2BB
"ms#3
mos#/
A-12
w'sC
&S
done
8ou&datio&s for &ervous system, "e&itouri&ary system, s'i&, bo&es, a&d
lu&"s are formed
/udime&ts of eyes, ears, &ose appear
Cardio#ascular system functionin*3 heart be*innin* to beat3 be*innin* of heart circulation.
Placenta de#/t.
44
0; 9D
)ee8s
7
e#ery
or*an
prese
nt3
+ead
*reatl
y
enlar
*ed
+lace
&tal
tra&sp
ort of
substa
&ces
: 5
wee's
;
5e&"th 2BB-24B
mm# %t# 4A5-A1B
"ms#
S'i& appears
wri&'led a&d pi&'
to red#
/.( be"i&s
.yebrows a&d
fi&"er&ails
develop#
&$NI' CO&$S "+$ $N"I$
(O,5. +as the ability to hear.
Production of lun* surfactants.
Passi#e Antibody transfer
0 placental immuno*lobulin -7
Su
stained
)ei*ht
*ain
occurs.
5a&u"
o covers e&tire
body#
.yebro
ws a&d scalp hair
is prese&t#
+eart
sounds are
perceptible by
45
auscultation.
&erni@
caseosa co#ers
s8in#
+eartbeat can be
heard in the fetoscope 0 9; )ee8sM
DB )ee8s7. Li#er is already
pancreas functionin*.
Quic8enin*
mother. S8eleton be*ins to de#elop.
(ro)n %ats be*in to
form. +eart sounds in the
stethoscope
Can be
heard 0 9K. DB
)ee8s7

NO"$1
2here is a
,lacental 3arrier
to s1,hilis until
the 1A
th
.ee7 o%
,regnanc1. )% the
+other is treated
3e%ore 1A
th
.ee7/
the 3a31 .ill +ost
li7el1 not 3e
a%%ected.

7ead is erected,
lower limbs are
well developed#
7eartbeat is
prese&t
*asal septum a&d
palate close
8i&"erpri&ts are
set

LANU
-O
APP$
AS
IN
"+$
(O,5
6vera"e le&"th is
5B-55 mm a&d
wei"hs 45 "ms#
8i&"ers a&d toes
are disti&ct#
+lace&ta is
complete#
46
/udime&tary
'id&eys secrete
uri&e#
8etal circulatio& is
complete#
.1ter&al "e&italia
show defi&ite
characteristics#
4a&"lio&ic cells
S$' IS
&ISUALL5
$CO-NILA(L$#
+eart is audible
in a ,oppler 0 99
th
)ee87 8etus
swallows# %ith
&ails# Jid&eys
able to secrete#
0he
fetus is 2$-31 mm
a&d wei"hs 2-4
"rams
8etus s
mar'edly be&t
7ead
is
disproportio&ately
lar"e due to brai&
developme&t
3e&ter
s of bo&e be"i& to
ossify
4a&"li
o&ic cells :5
th
to
12
th
wee's;
+lace&ta a&d meco&ium
are prese&t, with facial features
1 mo/ 4 wee's
$ mos# /26-2A wee's 5e&"th 25B-2$59 wei"ht A1B-15BB "ms#
S'i& red
/hythmic breathi&" occurs
+upillary membra&e disappears from
eyes#
8etus ofte& survives if bor& prematurely
(rain de#elops rapidly. Lecithin. Sphin*omyelin 0L>S
ratio is already D197
(rains fully de#eloped. If born3 neonate may sur#i#e.
4$
4=
4A
5B
51
52
53
!os
$
!
a
t
i
&
y
l
G
l
y
c
e
%
o
l
:

G
;
54
<
w
h
e
&
p
r
e
s
e
&
t
i
&
t
h
e
a
m
&
i
o
t
i
c
f
l
u
i
d
,
i
t
c
a
&
b
e
p
r
e
d
i
c
t
e
d
t
h
a
t
r
e
s
p
i
55
r
a
t
o
r
y
d
i
s
t
r
e
s
s
s
w
i
l
l
&
o
t
o
c
c
u
r
,
o
r
/
?
S

w
i
l
l
&
o
t
o
c
c
u
r#

Al$!a
)e
to
$%
ot
ei
nA
56

5$
P
(
at
er
&a
l
bl
oo
d
sa
m
pli
&"
be
tw
ee
&
16
-
2B
w
ee
's#
5
)
%
<
ch
ro
m
os
o
m
al
de
fe
cts
:?
o
w
&s
sy
&d
ro
m
e;

5=

I.&.
a
n
e
s
t
h
e
s
i
a

i
s
n
J
t
*
i
#
e
5A
n

f
o
r
a
m
n
i
o
c
e
n
t
e
s
i
s
.
"
h
e
c
l
i
e
n
t
s
h
o
u
l
d

b
e
s
u
p
i
n
e
d
u
r
i
n
*

t
h
e
p
r
o
6B
c
e
d
u
r
e
F
a
f
t
e
r
)
a
r
d
3
s
h
e
s
h
o
u
l
d

b
e
p
l
a
c
e
d

o
n

h
e
r
l
e
f
t
s
i
d
e
t
o

a
#
o
61
i
d

s
u
p
i
n
e
h
y
p
o
t
e
n
s
i
o
n
3
p
r
o
m
o
t
e
#
e
n
o
u
s
r
e
t
u
r
n
3
a
n
d

e
n
s
u
r
e
a
d
e
4
u
62
a
t
e
c
a
r
d
i
a
c
o
u
t
p
u
t
.
A
+
n
i
o
c
e
n
t
e
s
i
s
A
i
&
v
a
s
i
v
e
p
r
o
c
e
d
u
r
e
f
o
r
a
m
&
i
o
63
t
i
c
f
l
u
i
d
a
&
a
l
y
s
i
s
,
&

f
e
t
a
l
l
u
&
"
m
a
t
u
r
i
t
y
#

%
o
c
e
&
'
%
e
A
,
l
t
r
a
s
o
u
&
64
d
1
st
<
t
h
e
r
a
t
i
o
&
a
l
e
<
t
o
l
o
c
a
t
e
t
h
e
+
l
a
c
e
&
t
a
#
"
h
e
p
a
t
i
e
n
t
M
U
S
"

$
M
P
"
65
5

"
+
$

(
L
A
,
,
$


"
O

$
,
U
C
$

"
+
$

S
I
L
$

O
%

"
+
$

(
L
A
,
,
$

#
@
i
t
a
l
s
i
"
66
&
s
a
r
e
a
s
s
e
s
s
e
d
e
v
e
r
y
1
5
m
i
&
u
t
e
s
#

0
y
p
i
c
a
l
l
y
p
e
r
f
o
r
m
e
d
o
&
t
h
e
C
r
d
6$
t
r
i
m
e
s
t
e
r
t
o
a
s
s
e
s
s
L
$
C
I
"
+
I
N
.
S
P
+
I
N
-
O
M
5
$
L
I
N

/
6
0
!
)

!
*

0
7
.

6
(
*
6=
!
)
0
!
3

8
5
,
!
?

:
t
h
i
s
r
a
t
i
o
i
&
d
i
c
a
t
e
s
f
e
t
a
l
l
u
&
"
m
a
t
u
r
i
t
y
;
,
w
h
i
c
h
i
6A
s
c
o
m
m
o
&
l
y
d
e
l
a
y
e
d
i
&
a

d
i
a
b
e
t
i
c
c
l
i
e
n
t
3
3
e
s
a
r
e
a
&
?
e
l
i
v
e
r
y

s
h
o
u
$B
l
d

n
o
t
b
e
d
o
n
e
,
u
&
l
e
s
s
t
h
e
f
e
t
a
l
l
u
&
"
s
a
r
e
m
a
t
u
r
e
d
#

o
s
i
t
i
o
n
A
S
'
$
$1
i
n
e
.

P
L
A
C
$

A

%
O
L
,
$
,

"
O
6
$
L

O
N

+
$

I
-
+
"

(
U
"
"
O
C
=
S

"
O

"
I
P

+
$2
$


S
L
I
-
+
"
L
5

"
O

"
+
$

L
$
%
"

<

M
O
&
$

"
+
$

U
"
$

U
S

O
%
%

"
+
$

&
$
N
A

C
$3
A
&
A

"
O

P

$
&
$
N
"

S
U
P
I
N
$

+
5
P
O
"
$
N
S
I
O
N

S
5
N
,

O
M
$
.

A
(
,
O
M
I
N
A
L

P

$4
$
P

I
S

,
O
N
$
,
t
h
e
&
,
&
e
e
d
l
e
i
&
s
e
r
t
i
o
&
i
&
a
2
B
-
2
2
"
a
u
"
e
s
p
i
&
a
l
&
e
e
d
l
e
$5
,
w
i
t
h
d
r
a
w
i
&
"
a
m
&
i
o
t
i
c
f
l
u
i
d
#

*
)
/
(
6
5

5
/
S

/
6
0
!
)

:
l
e
c
i
t
h
i
&
/
s
p
$6
h
i
&
"
o
m
y
e
l
i
&
;
<
2
<
1
M

&
o
r
m
a
l
f
e
t
a
l
l
u
&
"
m
a
t
u
r
i
t
y
r
a
t
i
o

8
o
s
t
i
+
,
o
$$
r
t
a
n
t
%
a
c
t
o
r
a
%
%
e
c
t
i
n
g

A
+
n
i
o
c
e
n
t
e
s
i
s
A
*
.
.
?
5
.

!
*
S
.
/
0
!
)
*
-
b
e
c
a
$=
u
s
e
o
f
t
h
e
r
i
s
'
o
f
p
u
&
c
t
u
r
e
o
r
d
a
m
a
"
e
t
o
t
h
e
p
l
a
c
e
&
t
a
,
f
e
t
u
s
,
u
m
b
i
l
i
$A
c
a
l
c
o
r
d
,
b
l
a
d
d
e
r
&

u
t
e
r
i
&
e
a
r
t
e
r
i
e
s
#

D
i
s
a
&
(
a
n
t
a
"
e
s
A

i
s
7

%
o
r
=B
"
n.
o.
p.
4.
r.
=1
s.

#
A
L
L
2
*
E
P
*
<
S)
#)
A
N
(
&

2
*
E
(
(1
C
hil
ls3
fe
#e
r3
le
a8
a*
e
of
flu
id3
de
cr
ea
se
fet
al
m
o#
e
m
en
=2
t
or
ut
er
in
e
co
nt
ra
cti
on
s.
=3
=4

Ult%a
s
o
'
n
&
A
d
o
&
e
1
=
-
4
B
w
e
e
'
s
f
o
r
f
e
t
a
l
a
b
&
o
r
m
a
l
i
t
i
e
s
3
2*
E

B
=5
E
S
2

2
E
S
2

(
&


E
#
2
&
P
)
#

P

E
G
N
A
N
#
)
E
S
*
o
&
-
i
&
v
a
s
i
v
e
p
r
o
c
e
d
u
r
e
w
i
t
=6
h
h
i
"
h
f
r
e
K
u
e
&
c
y
s
o
u
&
d
w
a
v
e
s
t
o
o
b
t
a
i
&
o
u
t
l
i
&
e
o
f
t
h
e
f
e
t
u
s
,
p
l
a
c
e
&
=$
t
a
&

u
t
e
r
i
&
e
c
a
v
i
t
i
e
s
a
&
d
t
o
c
o
&
f
i
r
m

"
e
s
t
a
t
i
o
&
a
l
a
"
e
&

.
?
?
#

N
$
$
==
,
S

A

%
U
L
L

(
L
A
,
,
$


"
O

O
(
"
A
I
N

A

(
$
"
"
$


I
M
A
-
$

0
d
r
i
n
8
a

f
u
l
l
*
=A
l
a
s
s
e
#
e
r
y
9
E

m
i
n
u
t
e
s
b
e
*
i
n
n
i
n
*

a
n

h
o
u
r
<

h
a
l
f
t
h
e
p
r
o
c
e
d
u
r
e
7

AB
C
O
M
M
O
N

M
$
"
+
O
,

I
N

L
O
C
A
"
I
N
-

"
+
$

P

$
C
I
S
$

P
O
S
I
"
I
O
N

O
%

"
+
$

%
$
A1
"
U
S

<

P
L
A
C
$
N
"
A

(
$
%
O

$

A
M
N
I
O
C
$
N
"
$
S
I
S
.
-
*o '&ow&
-
-
-
-
-
-
-
-

A2

#
h
o
r
i
o
n
i
c
V
i
l
l
i
S
a
+
,
l
i
n
g
<
r
e
m
o
v
a
l
o
f
a
s
m
a
l
l
p
i
e
A3
c
e
o
f
3
h
o
r
i
o
&
i
c
v
i
l
l
i
s
a
m
p
l
i
&
"
t
o
d
e
t
e
c
t
t
h
e
f
f
<
f
e
t
a
l
c
h
r
o
m
o
s
o
m
e
,
A4
e
&
2
y
m
e
,
?
*
6

&

b
i
o
c
h
e
m
i
c
a
l
a
b
&
o
r
m
a
l
i
t
i
e
s
#
+
e
r
f
o
r
m
e
d
b
e
t
w
e
e
&
t
h
A5
e
=
th
>
1
1
th
w
e
e
'
s
o
f
"
e
s
t
a
t
i
o
&
#
3
a
&
d
e
t
e
c
t
t
h
e
f
f
9
4
e
&
e
t
i
c
?
e
f
e
c
t
s
<
).
c
A6
1
s
t
i
c

%
i
3
r
o
s
i
s
/
t
r
i
s
o
+
1

2
1
/
2
a
1

S
a
c
h
s
/
s
i
c
7
l
e

c
e
l
l
a
n
e
+
i
a
/
t
h
A$
a
l
l
a
s
e
+
i
a
/
D
u
c
h
e
n
n
e

+
u
s
c
u
l
a
r

d
1
s
t
r
o
,
h
1

B

h
e
+
o
,
h
i
l
i
a
.
(
o
s
t
c
A=
o
m
m
o
&

i
&
d
i
c
a
t
i
o
&
<
a
d
v
a
&
c
e

m
a
t
e
r
&
a
l
a
"
e
<
i
&
c
r
e
a
s
e
s
r
i
s
'

o
f
c
h
r
AA
o
m
o
s
o
m
a
l
d
a
m
a
"
e

f
r
o
m

a
"
i
&
"

o
f
o
o
c
y
t
e
#
4
r
e
a
t
e
s
t
6
d
v
a
&
t
a
"
e

o
v
e
1BB
r
6
m
&
i
o
c
e
&
t
e
s
i
s
<
+
.
/
8
)
/
(
.
?

?
,
/
!
*
4

0
7
.

8
!
/
S
0

0
/
!
(
.
S
0
.
/
#
:
1
6
t
1B1
h
-
2
B
t
h

w
e
e
'

o
f
"
e
s
t
a
t
i
o
&
;
#
#
5
a
b
o
r
a
t
o
r
y

r
e
s
u
l
t
s
a
r
e

o
b
t
a
i
&
e
d
1B2

i
&

1

-
$

d
a
y
s
c
o
m
p
a
r
e
d

t
o

2
B
-
2
=

d
a
y
s
f
o
r
a
&

a
m
&
i
o
c
e
&
t
e
s
i
s
#
D
1B3
i
s
a
&
(
a
n
t
a
"
e
s
A

1#

2#

3#
1B4

4#

5#
1B5

A
f
t
e
%

a
n

R
!
9
n
e
"
a
t
i
(
e

$
a
t
i
e
n
t
'
n
&
1B6
e
%
"
o
e
s
a
m
n
i
o
c
e
n
t
e
s
i
s
o
%

C
;
S
D
t
h
e

n
u
r
s
e

s
h
o
u
l
d

a
d
+
i
n
i
s
t
e
r

h
1B$

(
D
:

i
+
+
u
n
e

g
l
o
3
u
l
i
n

(

h
o
G
A
8
:
/
t
o

,
r
e
v
e
n
t

h

s
e
s
n
s
i
t
i
9
a
t
i
o
1B=
n
D
a
&

a
&
t
i
"
e
&

a
&
t
i
b
o
d
y

i
m
m
u
&
o
l
o
"
i
c

r
e
a
c
t
i
o
&

t
h
a
t
s
o
m
e
t
i
m
e
s
1BA
o
c
c
u
r
s
w
h
e
&

a
&

/
h

&
e
"
a
t
i
v
e

m
o
t
h
e
r
c
a
r
r
i
e
s
a
&

/
h

V

f
e
t
u
s
#

T
!
11B
e

$
a
t
i
e
n
t
&
o
e
s
n
o
t
%
e
E
'
i
%
e

c
o
m
$
l
e
t
e

b
e
&

%
e
s
t
a
f
t
e
%

C
;
S
-
-
-
S
*
E
111

S
*
&
!
L
D

E
(

A
)
N

(

&
8

S
E
?
!
A
L

)
N
2
E

#
&
!

S
E

A
N
D

P
*
<
S
)
#
A
L

A
#
2
112
)
V
)
2
<

(
&


C
A

h
o
u
r
s
.
6

s
m
a
l
l
a
m
o
u
&
t
o
f
s
p
o
t
t
i
&
"

i
s
&
o
r
m
a
l
f
o
r
t
h
113
e

9
s
t
D
I
.
I
;

h
o
u
r
s
.

114

;
.
)
e
t
a
l
C
i
%
c
'
l
a
ti
o
n
6s
e
a
r
l
y

a
s

3
r
d

w
e
e
'

o
f

i
&
115
t
r
a
-
u
t
e
r
i
&
e

l
i
f
e
,

f
e
t
a
l

b
l
o
o
d

i
s

a
l
r
e
a
d
y

i
s

c
i
r
c
u
l
a
t
i
&
"
,
116

s
p
e
c
i
f
i
c
a
l
l
y

t
h
e
r
e

i
s

a
l
r
e
a
d
y

e
1
c
h
a
&
"
e

o
f

&
u
t
r
i
e
&
t
s

w
i
t
11$
h

t
h
e

m
a
t
e
r
&
a
l

c
i
r
c
u
l
a
t
i
o
&

i
&

t
h
e

c
h
o
r
i
o
&
i
c

v
i
l
l
i
#

6
r
t
11=
e
r
i
e
s

c
a
r
r
y

,
*
)
Q
F
4
.
*
6
0
.
?

-
5
)
)
?
#

@
.
!
*
S

c
a
r
r
y

)
Q
F
4
.
*
6
0
.
?

-
11A
5
)
)
?
#

8
e
t
a
l

3
i
r
c
u
l
a
t
i
o
&

-
y
p
a
s
s
<

%
h
y
<

?,
.

0
)

*
)
*
-
8
,
*
3
12B
0
!
)
*
!
*
4

5
,
*
4
S
<
-----
?uc
tus
arter
ious
us
:bet
wee
&
pul
mo&
ary
arter
y &
6ort
a,
)+.
*S
60
-!/
07
&
35
)S.
S 24
>4=
hour
s
after
deli
very
#;
C
N"A
INS
a
mi@t
ure
of
arte
rial
121
<
#en
ous
bloo
d
-----
8ora
me&
)va
le <
bet
wee
&
ri"ht
&
left
atriu
m
?,. 0)
*)
*-
8,
*3
0!)
*!*
4
5!@
./<
-----
?uc
tus
@e&
osus
:by
pass
the
liver
,
clos
es at
birth
9 a&
umb
ilica
l
vei&
that
carri
es
7i"
h
122
o1y
"e&
fro
m
the
plac
e&ta#

)
i
"
'
%
e
I
.
i
C
!
a
n
"
e
s
i
n

)
e
t
a
l
c
i
%
c
'
l
a
t
i
o
n

:
123

%
e
n
a
t
a
l
t
o

o
s
t
n
a
t
a
l
C
i
%
c
'
l
a
t
i
o
n
*

124
Estimate& Date of
Confinement
:EDC*
(arch
2B,
-3 mo&ths
V$ days

ember
2$,
McDonal&s
)o%m'la :a"e of
"estation*
i& cm 1 2 divided
by $ M 6)4 i&
mo&ths
1 2 $ M
24 wee's
Len"t! :HaaseHs
R'le*
mo&ths -
Mmo&ths :sKuared;
6
-
5
6
7
;I.
#8
##
12
37
13
14
125
15

S'i&
i
s
s
m
o
o
t
h
,
c
h
e
s
t
i
s
p
r
o
m
i
&
e
&
t
.yes
a
r
e
u
&
i
f
o
r
m
l
y
s
l
a
t
e
c
o
l
o
r
e
d
126
-o&e
s
o
f
s
'
u
l
l
a
r
e
o
s
s
i
f
i
e
d
a
&
d
a
r
e
&
e
a
r
l
y
t
o
"
e
t
h
e
r
a
t
s
u
t
u
r
e
s
#

12$

8ace
a
&
d
b
o
d
y
h
a
s
a
l
o
o
s
e
w
r
i
&
'
l
e
d
a
p
p
e
a
r
12=
a
&
c
e
b
e
c
a
u
s
e
o
f
s
u
b
c
u
t
a
&
e
o
u
s
f
a
t
d
e
p
o
s
i
t#
-ody
i
s
u
s
u
a
l
l
y
l
u
m
p
a
&
d
l
a
&
12A
u
"
o
d
i
s
a
p
p
e
a
r
s
*ails
r
e
a
c
h
f
i
&
"
e
r
t
i
p
e
d
"
e
6m&i
o
t
i
c
f
l
u
i
d
d
e
c
r
e
a
s
e
s
#

13B

0
o
e
&
a
i
l
s
b
e
c
o
m
e
v
i
s
i
b
l
e
S
t
e
a
d
y
w
e
i
"
h
t
"
a
i
&
o
c
c
u
r
131
s
@
i
"
o
r
o
u
s
f
e
t
a
l
m
o
v
e
m
e
&
t
o
c
c
u
r
s
#

132
2
m
o
/
5
-
=
w
e
e
'
s
8
.
0
6
5

(
)
@
.
(
.
*
0

3
)
,
*
0
0
e
a
c
h
m
o
t
h
e
r
t
o
c
o
u
&
t
2
-
3/.60!*!*. 5.@.5 .stimates fetal re&al maturity a&d
fu&ctio&, uses am&iotic fluid#

133
3
t
i
m
e
s
d
a
i
l
y
,
3
B
-
6
B
m
i
&
u
t
e
s
e
a
c
h
t
i
m
e
,
s
h
o
u
l
d
f
e
e
l
5
-
6
m
o
v
e
m
e
&
t
s
p
134
e
r
c
o
u
&
t
i
&
"
t
i
m
e
9
m
o
t
h
e
r
s
h
o
u
l
d
&
o
t
i
f
y
c
a
r
e
"
i
v
e
r
i
m
m
e
d
i
a
t
e
l
y
o
f
a
135
b
r
u
p
t
c
h
a
&
"
e
o
r
&
o
m
o
v
e
m
e
&
t#
+
.
/
3
,
0
6
*
.
)
,
S

,
(
-
!
5
!
3
6
5

-
5
)
)
?

S
6
(
136
+
5
!
*
4
,
s
e
s
u
l
t
r
a
s
o
u
&
d
t
o
l
o
c
a
t
e
u
m
b
i
l
i
c
a
l
c
o
r
d
#
3
o
r
d
b
l
o
o
d
a
s
p
i
r
a
13$
t
e
d
a
&
d
t
e
s
t
e
d
#
,
s
e
d
i
&
s
e
c
o
&
d
a
&
d
t
h
i
r
d
t
r
i
m
e
s
t
e
r
s
#
-
!
)
+
7
F
S
!
3
6
5

13=
+
/
)
8
!
5
.
6

c
o
l
l
e
c
t
i
o
&
o
f
d
a
t
a
o
&
f
e
t
a
l
b
r
e
a
t
h
i
&
"
m
o
v
e
m
e
&
t
s
,
b
o
d
y
m
13A
o
v
e
m
e
&
t
s
,
m
u
s
c
l
e
t
o
&
e
,
r
e
a
c
t
i
v
e
h
e
a
r
t
r
a
t
e
a
&
d
a
m
&
i
o
t
i
c
f
l
u
i
d
v
o
l
14B
u
m
e
#
5
e
v
e
l
-
h
i
"
h
e
a
r
l
y
i
&
p
r
e
"
&
a
&
c
y
9
&
%
o
$
s
a
f
t
e
%
3
4
<
e
e
2
s
"
e
s
t
a
t
i
o
141
n
9
u
s
e
s
a
m
&
i
o
t
i
c
f
l
u
i
d
#
=
m
o
s
#
/
3
B
-
3
4
w
e
e
'
s
"
h
e
y
e
l
l
o
)

c
o
l
o
r
i
s
t
h
e
142
r
e
s
u
l
t
o
f
f
e
t
a
l
a
n
e
m
i
a

a
n
d

b
i
l
i
r
u
b
i
n
.
-!5!/,-!*
ELECTR-NIC M-NIT-RING
A. Non9St%ess Test > acceleratio&s i& heart rate accompa&y &ormal fetal moveme&t9 &o&-i&vasive
0ocody&amometer records fetal moveme&ts a&d ?oppler ultrasou&d measures
9 )bservatio& of fetal heart rate related to fetal moveme&t# %etal )ell.bein*#
!&dicated for< assess place&tal fu&ctio& & o1y"e&atio&, fetal well bei&", evaluates fetal heart rate i&
respo&se to fetal moveme&t especially for< Maternal Problems such as chronic hypertension3 diabetes
and Pre.eclampsia3 *i#en after the CD
nd
)ee8#
REARATI-NA
+atie&t should eat s&ac's#
143
PositionA Semi.%o)lers or left lateral positions the mother may as' tom press the butto& every time
she feels fetal moveme&ts9 the mo&itor records a mar' at each poi&t of fetal moveme&t#
RESULTSA
9. eactive (nor+al*A i&dicates a fetal fetus
-reater than 9E beats per minute. occur )ith fetal mo#ement in a 9B or DB minute period.
86@)/6-5. /.S,50S<
- 2 or more 87/ acceleratio&s of 15 seco&ds over a 2B mi&utes i&terval a&d retur& of 87/ to
&ormal baseli&e#
D. Non-eactive (A3nor+al;< No fetal mo#ement occurs or there is short.term fetal heart rate
#ariability 0less than : beats per minute7. 0he doctor will order a& )1ytoci& 0est 680./ the
patie&t has &o&-reactive test#
*)0.< COMMONL5 P$%OM$, ON ,IA($"IC PA"I$N"S ($CAUS$ O% "+$ INC$AS$ IS=
%O S"ILL (I"+.
(. Cont%action St%ess Test :CST* > based o& the pri&ciple that healthy fetus ca& withsta&d
decreased o1y"e& duri&" co&tractio& but compromised fetus ca&&ot# /espo&se of the fetus to i&duced
uteri&e co&tractio&s as a& IN,ICA"O O% U"$OPLAC$N"AL < %$"AL P+5SIOLO-ICAL
IN"$-I"5.
REARATI-NA
%oma& i& semi-8owlerEs or side-lyi&" positio&#
(o&itor for post-test labor o&set#
0F+.S<
a# (ammary stimulatio& 0est or -reast Stimulatio& .1am or
*ipple Stimulated 3S0 > &o&-i&vasive
b# )1ytoci& 3halle&"e test
)ndications" ALL PEGNAN#)ES A(2E 2A 4EEKS 4)2* *)G* )SK #L)EN2S.
#ontraindicated %or histor1 o% PE-2E8 LAB&.
Inte%$%etationsA
-SITI;E RESULT< 5ate deceleratio&s with at least 5BC of co&tractio&s# +ote&tial ris's to the fetus, which
may &ecessitate to 3-sectio&#
Abno%mal an& 2no<n as Iositi(e <in&o<D. Abnormal1 NPositi#e 6indo)O1 0P7 LA"$
,$C$L$A"IONS O% %+ )ith three contractions a 9B minute inter#al. Indicates Uteroplacental
Insufficiency.
NEGATI;E RESULTS< *o late deceleratio&s with a mi&imum of 3 co&tractio&s lasti&" 4B-6B seco&ds i& 1B
mi&utes period# No%mal1 NNe*ati#e 6indo)O1 0.7 LA"$ ,$C$L$A"IONS O% %+ )ith three
contractions a 9Bm minute inter#al
No%mal an& 2no<n as INe"ati(e <in&o<
Labo%ato%y St'&ies
#. .striol e1cretio&< measures place&tal fu&ctio&i&" throu"h uri&e test#
3ollect a 24-hour uri&e specime& or serum blood levels#
7i"h .striol< 4ood place&tal fu&ctio&
5ow .striol< 8etal hypo1ia
144
Est%iolA estro"e&ic hormo&e, sy&thesi2ed by the place&ta & adre&al "la&d of the fetus which secreted
by the ovaries
h )nco+,ati3ilit1 2estA
'%$oseA a# to discover prese&ce of a&tibodies prese&t i& /h-&e"ative motherEs blood#
P 0est will co&firm the dia"&osis for 7emolytic ?isease i& the *ewbor&#
Ty$esA
1# In&i%ect CoombHs TestA wome& who have /h &e"ative have this test do&e to determi&e
if they have a&tibodies to the factor prese&t# /epeated 2= wee's pre"&a&cy#
(others reveal a&tibodies as a result of previous tra&sfusio& or pre"&a&cy#
2# Di%ect CoombHs testA tests for &ewbor&s cord blood- determi&es prese&ce of mater&al
a&tibodies attached to the babyEs cell#
/h :?; & ? &e"ative who has&Et formed a&tibodies should receive
/ho"am at 2= wee's "estatio& or after $2 hours after delivery#
2he Bet7e-Kleihauer test is a test that deter+ines i% a greater than usual %etal E +aternal 3lood +i$
occurred. )t is also used in h inco+,ati3ilit1 cases to deter+ine i% another dose o% hoga+ is
needed
(ern 2est" determi&e the prese&ce of 6m&iotic 8luid lea'a"e# ,si&" a sterile tech&iKue, a specime& is
obtai&ed from the e1ter&al os of the cervi1 & va"i&al pool#
Position< ?orsal 5ithotomy, !&struct the clie&t to cou"h to cause the fluid to lea' from the uterus if the
membra&es are ruptured#
Nitra9ine 2estA use of &itra2i& strip to detect the prese&ce of am&iotic fluid#
@a"i&al Secretio&s< +7< 4#5- 5#5
6m&iotic fluid< +7< $#2 > $#5 :tur&s the yellow *itra2i&e blue "ray, blue "ree& > /uptured
(embra&es;
Kic7s count< fetal moveme&t cou&ti&" mother sits Kuietly o& the 5.80 S!?. for 1 hour after meals &
cou&t fetal 'ic's for 3B mi&utes# *otify the physicia& or health care provider if 8.%./ 076* 3
J!3JS#
Bio,h1sical Pro%ile " surveilla&ce of fetal well bei&" base o& 5 cate"ories<
9. %etal breath mo#/t
D. %etal tone
C. Amniotic fluid
I. %etal heart reacti#ity
E. Placental -rade
Inte%$%etationA
%etal score of ; H 9B1 normal fetal )ell.bein*
%etal score of I H :1 fetal distress
;II. -t!e% Gynecolo"ical %oce&'%es
a. Schiller 0est< i&dicated for ca&cer, ca&didates are wome& of 2B years old & above & se1ually active
wome&#
P 3ervi1 is tai&ted with ti&cture of iodi&e9 color cha&"e i& the cervi1 is &oted#
/esult<
Ne"ati(eA maho"a&y brow& stai&
ositi(eA &o stai&i&"
145
b. a$anicola' TestA cytolo"ic test for ca&cer
P ?etect preca&cerous lesio&s &, detect the recurre&ce of 3a&cer#
c. Hyste%osal$!in"o"%am A COMPL$"$ $&ALUA"ION O% ALL P$L&IC O-ANS IN %$MAL$S
P .@65,60.S 0,-65 +60.*3F & +/)-5.(S !* 8./0!5!0F#
P !f the tubes are pate&t, the dye ca& be visuali2ed passi&" out the fimbtriated e&d & of the fallopia&
tubes#
&. R'bins Test9 determi&es tubal pate&cy of the fallopia& tubes# 3)2 is passed throu"h the cervi1 to the
uterus#
P !f pate&t, "as will pass throu"h the fimbriated e&d of the fallopia& tubes, will "ive a se&satio& of
full&ess & spasmodic pai&s due to irritatio& from the "as#
P 6 test to detect i&fertility caused by a defect i& the tube, which is usually related to Past Infection.
e. Sims H'!ne% Test :ost Coital Test*A withi& 1 >2 days, a specime& of semi&al fluid from the posterior
for&i1 & cervical ca&al is aspirated 2 >4 hours after coitus#
'%$oseA test for i&compatibility of sperms with cervical mucus#
9.D days is the best time to e#aluate fertility because there is increase estro*en#
. A(UN,AN" C$&ICAL MUCUS. increases sperm sur#i#al#
;III. THREE REGNANC= SIGNS & S=MT-MS
146
S=
I@.
Disc
o+%o
rts in
Preg
nanc
1
D
i
s
c
o
m
f
o
%
t

T
%
i
m
e
s
t
e
%
N
'
%
s
i
n
"
M
e
a
s
'
%
e
N
a
'
RESUMTI;E R-1A1LE -SITI;E
14$
s
e
a
&

;
o
m
i
t
i
n
"
e
n
&
s
o
n

t
!
e
#
st
t
%
i
m
e
s
t
e
%
J

&
%
y
c
%
a
c
2
e
%
s
&

e
a
t
s
m
14=
a
l
l
f
%
e
E
'
e
n
t
U
%
i
n
a
%
y
)
%
e
E
'
e
n
c
y
e
n
&
s
o
n

t
!
e
#
st
t
%
i
m
e
s
t
e
%
G
e
"
e
14A
l
H
s
e
+
e
%
c
i
s
e
D

M
a
y
s
t
a
%
t
o
n

t
!
e
3
%
&
t
%
i
m
e
s
t
e
%
a
l
s
o

1
%
e
a
s
t
t
e
n
15B
&
e
%
n
e
s
s
a
l
l
t
%
i
m
e
s
t
e
%
$
!
a
s
e
s

J

<
e
a
%
s
'
$
$
o
%
t
i
(
e
<
e
l
l
f
i
t
t
i
n
"
151
b
%
a
&

E
n
"
o
%
"
e
m
e
n
t
J

a
(
o
i
&

s
o
a
$
A
t
o
$
%
e
(
e
n
t
&
%
y
i
n
"
H
e
a
%
t
152
b
'
%
n

,
n
&
9
3
%
&

J

s
m
a
l
l
e
%
m
e
a
l
s
D
s
!
o
%
t
e
%
i
n
t
e
%
(
a
l
s
153
J

A
(
o
i
&

f
a
t
t
y
f
o
o
&
s
&

N
a
1
i
c
a
%
b
o
n
a
t
e
J

D
%
i
n
2

154
m
i
l
2

b
e
t
<
e
e
n

m
e
a
l
s
J

I
n
c
%
e
a
s
e
<
a
t
e
%
:
6
9
#
,
"
l
a
s
s
e
s
*
9
t
155
o

m
i
n
i
m
i
3
e
%
e
"
'
%
"
i
t
a
t
i
o
n
M'scle & Le" c%am$s
3
%
&
J
l
e
"
e
+
t
e
n
s
i
o
n

&

f
e
e
156
t
&
o
%
s
i
f
l
e
+
i
o
n

%
e
l
i
e
f
9
n
'
%
s
i
n
"
i
n
t
e
%
(
e
n
t
i
o
n
J

E
l
e
(
a
t
15$
e
t
!
e
l
e
"
s
<
i
t
!

a
$
i
l
l
o
<

;a%ic
ositie
s
3
%&

?ea%
s'$$
o%ti(
e
!ose.
N-T
GNE
E
HIG
H
H-S
E
3
%&
J
?ea%
lo<
!eele
&
s!oes
.
Sittin
" &
el(i
c
Roc2
15=
in"
3
%&
J
Left
Late
%al
si&e
lyin"
$ositi
onA
Syn&%ome
%elie
(e
infe%i
o%
(ena
ca(a
syn&
%ome


Le'2
o%%!
ea
3%&
J
A(oi
&
Do'c
!e.
Hy"i
ene.

(atig
ue
2
nd

J
due
to
)ron
De%ic
ienc1
Ane
+ia
2he diagnosis o% iron-de%icienc1 ane+ia is +ade on the 3asis o% a
he+oglo3in concentration value o% 1; g@dl 3lood or less and a
he+atocrit value o% 5;F or less.
(re
Guen
t rest
,erio
15A
ds B
give
dar7
#
onsti
,atio
n

2
n-
-
5
rd

Prev
entio
n"
*igh
%i3er
)nter
venti
on"
)ncre
ase
(lui
d
N&2E" Bul7 and %luid hel, increase ,eristalsis. La$atives and
su,,ositories should not 3e used routinel1 in ,regnanc1.
Prevention is +ore desira3le than treat+ent.
@.
S=
CH-
L-G
ICA
L
CHA
NGE
S IN
RE
GNA
NC=
0/!
(.
S0.
/
,ltra
sou&
d
ima"i
&" of
16B
fetal
heart
motio
& by
wee'
$
,ltra
sou&
d
co&fi
rmati
o& of
"estat
io&al
sac
by
wee'
6
Ultra
soun
d1 :
)ee8
s can
ausc
ultat
e the
fetal
heart
.
8etal
move
me&t
s
palpa
ted
by
the
provi
der
by
wee'
2B#
"he
most
ob?ec
ti#e
si*n
of
pre*
nanc
y is
fetal
mo#e
161
ment
felt
by
the
e@am
iner.
-ood
ell/s
si*n
is a
softe
nin*
of
the
cer#i
@3
)hic
h
occu
rs in
pre*
nanc
y
+alpa
ti&"
fetal
co&to
urs
-ra1t
o&-
7ic's
co&tr
actio
&s
(allotment1 bouncin*
of the fetus in the
amniotic fluid a*ainst
the e@aminers hand.
,urin* the 9:
)ee8.

(ra
@
t
o
n

+
i
c
8
s

162
C
o
n
t
r
a
c
t
i
o
n
s
1

p
a
i
n
l
e
s
s

c
o
n
t
r
a
c
t
i
o
n
s

f
e
l
t

f
o
r

D
B
.
C
B

m
i
n
u
163
t
e
s

o
c
c
u
r
s

o
n

t
h
e

9
:
t
h

)
e
e
8
#

Chad)ic8/s si*n is a
Increase si2e of the uterus
Q P Pre*nancy "est

R Secretion of +C-
164

. "he fetal heartbeat


2#
!&evitable0
hreate&ed
loss that
ca& be
preve&ted9
abortive
process is
"oi&"
o&-leedi&"
a&d
cervical
dilatio&Sav
e tissue
fra"me&ts?
.8!*!0!)
*S/S*,/S
!*4
!*0./@.
*0!)*1#
0hreate&ed
0he
co&ti&uatio
& of the
pre"&a&cy
is i&
doubt-leed
6mbivale&ce
8ear
8a&tasies about
motherhood a&d about
havi&" a Wdream childE#
+ossible decrease i& se1
drive#
6ccepti&" the
pre"&a&cy
II AM
REGNAN
TK
,urin* the first
trimester3 the
mother copes )ith
the common
discomforts and
chan*es
165
i&" or
spotti&"
closed
cervi1-edr
est,
/estrictive
activity,
Sedatio&,
6void
coitus for 2
wee's
followi&"
last
evide&ce of
bleedi&"+re
pari&" for
pare&thood#
0hird8eeli&
"s of
aw'ward&e
ss a&d
clumsi&ess#
6ccepti&"
the baby
S6 -6-F
!S
4/)%!*
4 !*S!?.
(.T8irst6
me&orrhea
*ausea/@o
miti&"
ho*am
indicated
)hen a youn*
patient has a
threatened
abortion in
the first
trimester and
a laboratory
studies re#eal
an h
ne*ati#e and
the husband
is h positi#e
166
A.
MATERNAL
ADATATI
-NS
DURING
REGNANC
= / <it!
1I-L-GIC
AL TASGS
-)
REGNANC
=
)i%st
T%imeste%A
6(-!@65.
*3.- about
pre"&a&cy<
pre"&a&t
woma& focus
o&ly to self#
I am
pre*nant. S
Accept the
biolo*ical
fact of
pre*nancy
Secon&
T%imeste%A
633.+06*
3.---of the
ide&tificatio&
of
motherhood
& aware&ess
& i&terest i&
the fetus#
I am
*oin* to ha#e
a babyS
Accept the
*ro)in* fetus
as distinct
from self <
as person to
care for
T!i%& T%imeste%A .()0!)*65
56-!5!0F-
assumi&"
already the
mother, fears
& fa&tasies &
dreams about
labor
I am
*oin* to be a
16$
motherS
Prepare
realistically
for birth <
parentin*

1.
ATERNAL
ADAATATI
-NS /
REACTI-N
S T-
REGNANC
=
#&!VADE
S<ND&8E
1
identification
of the
motherF
ambi#alence
< an@iety
about the role
chan*e
HEAL&!S<
S2AGE"
increase
interest in
mothers care.
SEL(-
#&N#EP2
#*ANGE"
acti#e
in#ol#ement
in the fears <
death of the
fetus.
SECTI-N
III
6*0.+6/06
5
3)(+5!360
!)*S
A. Abo%tion
-termination
of pre*nancy
before the
fetus is #iable
(2; .ee7s or
16=
a .eight o%
6;; *7
)i". ...
0yp e s o f
6b o r t i o &
0F+.S
II AM A
M-THERK
During the
third
tri+ester/ a
7e1
,s1chosocial
tas7 is to
overco+e
%ears the
.o+an +a1
have a3out
the
un7no.n/
la3or ,ain/
loss o% sel%-
estee+/ loss
o% control/
and death.
2he e+otions
and %ears
that are
usuall1 %elt
during the
third
tri+ester are
%eelings o%
IuglinessD/
alterations in
16A
(issed !&complete 3omplete 7abitual
0hreate&ed !&evitable
0herapeutic Spo&ta&eous
6-)/0!)*
3od1 sel%-
i+age and
an$iet1 a3out
the co+ing
la3or and
deliver1.
Rene<e&
fea%s an&
tensions
abo't labo%.
Spurt of
e&er"y duri&"
last mo&th#
.
. During the
second
tri+ester/
,s1chosocial
tas7s include
+other-
i+age
develo,+ent/
co,ing .ith
3od1 i+age
and se$ualit1
changes/ and
,renatal
3onding.
"he
benefits of
dru*
therapy
out)ei*h
the ris8s to
the
patient/s
nausea is
to control
in a first
trimester
patient/s
nausea.
-reast
se&sitivity
a&d i&creased
si2e 8ati"ue
6bdomi&al
e&lar"eme&t
S'i&
pi"me&tatio&
1$B
cha&"es
:(elasma
chloasma,
li&ea &i"ra- a
brow& li&e
ru&&i&" from
the umbilicus
to the
symphysis
pubis
Stretch mar8s
)ill
e#entually
fade to a
sil#ery )hite
color3 but it is
hi*hly
unli8ely that
they )ill
completely
disappear.
-reast
cha&"es-
i&crease i&
full&ess,
dar'er areola#
DXuic'e&i&"<
first fetal
movEt#
D,ri&ary
8reKue&cy
D (elasma #
3#
3omplete6lte
r&ate feeli&"s
of emotio&al
well-bei&"
a&d
liability#Seco&
d
6ccepta&ce of
pre"&a&cy#
+ossible
i&crease i&
se1#
6dustme&t to
cha&"e i&
body ima"e#
"he
second
trimester
of
pre*nancy3
+roducts of co&ceptio&
are totally e1pelled
(i&imal bleedi&" 3o&ti&uous mo&itori&"
1$1
)omen
*enerally
feels their
best.
4# !&complete Some fra"me&ts are
retai&ed i&side the uteri&e
cavity
+rofuse bleedi&" ?ilatatio& & 3uretta"e9
,se of o1ytoci&<
O@ytocin nasal spray should be administered
)hile the client is sittin* )ith her head in a
#ertical position. A nasal preparation must
not be administered )ith the client lyin* do)n
or the head tilted bac8 because this could
cause aspiration.
.vacuatio&
5# (issed /ete&tio& of the products
of co&ceptio& after fetal
death
!&termitte&t
bleedi&"9 abse&ce of
uteri&e "rowth
.vacuatio&, ? & 3
6#7abitual /
/ecurre&t
3 spo&ta&eous abortio&s
occurri&" successively
+rovide !@, (o&itor bleedi&", 3ou&t peri&eal
pads, psycholo"ical support
NO"$1Because s,ontaneous a3ortion is
threatening/ all ,erineal ,ads +ust 3e
ins,ected %or the ,roducts o% conce,tion.
%luid replacement is necessary because of
blood loss
1. Ecto$ic %e"nancy
6# ?escriptio&< 6 pre"&a&cy that occurs i& a&other tha& uteri&e site, with impla&tatio& usually occurri&" i&
fallopia& tubes
6 ruptured ectopic pre"&a&cy is a medical emer"e&cy due to the lar"e Kua&tity of blood that may be lost i& the
pelvic a&d abdomi&al cavities# Shoc' may develop from blood loss, a&d lar"e Kua&tities of !#@# fluids are &eeded
to restore i&travascular volume u&til the bleedi&" is sur"ically co&trolled# +ai& may be caused by a ruptured or
diste&ded fallopia& tube or blood i& the perito&eal cavity#
(o&itor amou&t Si"&s a&d Symptoms ?ia"&ostic 0ests (a&a"eme&t
1$2
of bleedi&"-
3uldoce&tesis-
@a"i&al
-leedi&"3auses
6ssess vital
si"&s
6ssess
abdomi&al
pai&
-lood
tra&sfusio&
Sur"ery<
Salpi&"osto
my
6dmi&ister
/ho"am for
/h :-; clie&t
-3uldoscopy
-
/adioimmu&oas
say of elevated
serum
Kualitative
--eta-734
-6bdomi&al
,ltrasou&d
--lood samples
of 7"b a&d
7ct9 blood type
a&d "roup
-=nife.li8e
abdominal pain
-eferred pain
on the ri*ht
shoulder
-Symptoms of
Shoc8<
decreased -+
i&creased //,
fast but thready
pulse# "his is
the number 9
complication.
-+elvic pressure
of pelvic
full&ess
-3ulle&Es si"&
-Pain
unilaterally3
)ith crampin*
1$3
and tenderness
. Mass in the
adne@al or cul.
de.sac
- Sli"ht, dar'
va"i&al bleedi&"
- +rofou&d
shoc' if rupture
occurs
-*arrowi&" of tube 2he J1 #o+,lication o% Ecto,ic Pregnanc1 is *e+orrhagic Shoc7.
C. Hy&ati&ifo%m mole / T%o$!oblastic Disease / Mola% Disease
- 4estatio&al trophoblastic &eoplasm that arise from the chorio&9 characteri2ed
by the proliferatio& a&d de"e&eratio& of the chorio&ic or trophoblastic villi#
A patient )ith +ydatidiform mole has a positi#e si*ns of pre*nancy but is not pre*nant.
"he T9 Complication is #horiocarcino+a
"he "hree + of +.mole
9.*yper . emesis *ra#idarum
D. increase *c*
C. increase incidence for pi*
(6*!8.S060!)*S?!64*)S0!3 0.S0S(6*64.(.*0+/.?!S+)S!*4 8630)/S
1# 5ow
socioeco&omic
status0F+.S
2# %ome& below
1= or above 35
3# !&ta'e of
3lomid
:3lomiphe&e
1# 3omplete/
classical
parts of the
villi are
affected
2# !&complete/
partial- some
parts are
1# @a"i&al bleedi&"
2# .1cessive */@
C. apid enlar*ement of
the uterus
I. 0P7 Pre*nancy test
5# +ossible +!7
6# 6bdomi&al cramps
$# 6bse&t 87/
1# 734 titer
determi&atio&
D. Ultrasound
3# Q-ray of the
abdome&
1# (olar evacuatio& /
?&3
2# 3hemotherapy
3# (o&itor 734 levels
4# ?elay childbeari&"
pla&s for a year
5# +eri&eal pad cou&ts
6# !&struct the couple to
1$4
3itrate;
4# %ome& of
asia& herita"e
&ormal =# .levated 734 titer< 1-2
millio& !,9 *ormal
level< 4BB,BBB !,
have @64!*65
/.S0 : &o se1; for 1
year#
2he J1 #o+,lication o% *-+ole is choriocarcino+a
D. Incom$etent ce%(i+
- +ai&less premature dilatatio& of the cervi1 :usually i& the 9:
th
to DB
th
)ee87
INC-METENT CER;I@

8i"ure 1A
!*3)(+.0.*0 3./@!Q
1# 0he cervi1 dilates pai&lessly i& the seco&d
trimester of pre"&a&cy# -loody
show+ressure or heavi&ess o& the lower
abdome&#3ardi&al/+atho"&omo&ic/maor
si"&<5ate si"&s<!&itial Si"&sSho) 0a pin8.
stained #a*inal dischar*e7?ysfu&ctio&al
cervi1+redisposi&"/3o&tributi&"
8actors</epeated dilatatio& of the cervi1,
maternal ,$S 0 ,iethylstilbestrol7
$@posure, 0raumatic i&uries to the cervi1#
3o&"e&ital a&omaly
2#+/)(
3#+ai&less dilatatio&
9. -irth of dead/&o&-viable fetus
D.
C.
I. J1 Sign1 upture of membranes and dischar*e
of amniotic fluid
E. 0rauma to the cervi1 :sur"ery / birth;
:. ,teri&e a&omaly
K. 7abitual abortio&
;. +re-term labor

U. 4.S060!)*65 ?!6-.0.SDefinitionA
t1,e o% Dia3etes .here onl1 ,regnant
.o+en gets .here her 3lood sugar rate
elevates 3ut never had a high 3lood sugar
rate 3e%ore ,regnanc1.SynonymsDiabetes
&'%in" %e"nancy
%e&is$osin"/Cont%ib'tin"
)acto%sHy$e%"lycemia &e(elo$s &'%in"
$%e"nancy beca'se of t!e sec%etion of
$lacenta !o%mones s'c! as %olactinD
%o"este%one& Co%ticoste%oi&s-est side
eKuipme&tSuctio&*ursi&"
?ia"&osis3ervical !&compete&ce*ursi&"
!&terve&tio& Pre.op1 $ncoura*e patient to
maintain bed restUltrasono*raphy(est
ma?or sur*ery1Cer#ical Cercla*e3
Mc,onald Cercla*ePossible sur*ical
99. ,ltrasou&d
1$5
complication1Sterility3 rupture of the cer#i@
premature deli#ery3 pel#ic bleedin* and
infection.,isease complicationT9
+emorrha*e3 $ctopic pre*nancy3 birth
defects3 #iruses and pre*nancy diseases3
diabetes in pre*nancy3 +PN(est position
before and after sur*erySide lyin*
positionScreenin* or initial dia*nostic
test1Synonyms
(ater&al a"e more tha& 35
+revious macrosomic i&fa&t
+revious u&e1plai&ed stillbirth
+revious pre"&a&cy with 4?(
8amily history of ?(
)besity
7yperte&sio&
9B. 8-S more tha& 14B m"/dl
(Pillitteri/ 8aternal and #hild Nursing/
,.5K1-K5:

E. DIA1ETES MELLITUS
4estatio&al diabetes mellitus :pre"&a&cy
i&duced;
A pre*nant3 insulin.dependent diabetic is at
ris8 for sudden h1,ogl1ce+ia because insulin
needs and metabolism are affected b pre*nancy3
ma8in* sudden hypo*lycemic episodes more
common for diabetics.
3ha&"es i& the "lucose-i&suli& mecha&ism<
o .arly i& pre"&a&cy<
6# !&crease productio& of
i&suli&
-# (ater&al "lucose is
co&sumed by fetus
o 5ate i& pre"&a&cy<
6# (other develops
i&suli& resista&ce
-# 0he prese&ce of
place&tal i&suli&ase
brea's dow& i&suli&
rapidly

-# ?escriptio& of
?iabetes i& +re"&a&cy

1. (ater&al "lucose crosses the place&ta but


i&suli& does &ot
2. ?uri&" the first trimester, mater&al i&suli&
&eeds decrease
5. 0he fetus produces its ow& i&suli& a&d pulls
"lucose from the mother, which predisposes
1$6
the mother to hypo"lycemic reactio&s
C. ?uri&" the seco&d a&d third trimesters,
i&creases i& place&tal hormo&es cause a&
i&suli&-resista&t state, reKuiri&" a& i&crease
i& the clie&tLs i&suli& dose
6. ?iabetes mellitus is more difficult to co&trol
duri&" pre"&a&cy & occurs duri&" the
seco&d or third trimester# +remature
delivery is more freKue&t# 0he &ewbor&
i&fa&t of a diabetic mother may be lar"e i&
si2e but will have fu&ctio&s related to
"estatio&al a"e rather tha& si2e# 0he
&ewbor& i&fa&t of a diabetic mother is
subect to h1,ogl1ce+ia/
h1,er3iliru3ine+ia/ res,irator1 distress
s1ndro+e/ and congenital ano+alies.
Still3orn and neonatal +ortalit1 rates are
higher in ,regnancies o% a dia3etic .o+an
L.
M. N&2E" 2he greatest incidence o% insulin
co+a during ,regnanc1 occurs during the
second and the third +onths/ the incidence o%
the dia3etic co+a during ,regnanc1 occurs
around the si$th +onths.
A.
K.
GESTATI-NAL DIA1ETES
Post.op1 Chec8 for e@cessi#e #a*inal dischar*e
and se#ere pain.
9. (ed rest in trendelenbur* position
D. Administer tocolytic medications as ordered $*F
itodrine +ydrochloride 05utopar71 "erbutaline
sulfate 0(rethine71 Ma*nesium Sulfate
I. +ydro@y2ine hydrochloride 0&istaril7 is a
common dru* ordered to counteract the effect of
terbutaline 0(rethine7
:. Sur*ery1 Cer#ical Cercla*e
a. Shirod8ar.(arter "echni4ue
0 internal os7 permanent suture1 subse4uent
deli#ery by C>S.
b. Mc ,onald Procedure 0 e@ternal
os7.suture remo#ed at term )ith #a*inal
deli#ery
c.
Usually I.: )ee8s after #a*inal deli#ery is the
safe period for a patient to resume se@ual
acti#ity3 )hen the episiotomy has healed
and the lochia had stopped
1$$
. Monitor &>S and report +PN (o&itor
87/
5imit activities
)bserve for /uptured -)%
6void va"i&al douche
6void coitus
+ro&e positio&
9D. %ei"ht loss, fati"ue, &ausea, a&d
vomiti&" e1cessive thirst, decrease
uri&atio&8ati"ue, wea'&ess, sudde& visio&
cha&"es, ti&"li&" or &umb&ess i&
ha&ds3ardi&al/+atho"&ome&ic/maor si"&!&itial
Si"&s3o&formity test<
9C.
9I. 3-+Es< +olyuria, +olydipsia a&d +olypha"ia
(60./*65 S!4*S & SF(+0)(S<
1#.1cessive thirst
2# 7u&"er
3# %ei"htless
4# -lurred visio&
5# 8reKue&t uri&atio&
6# /ecurre&t uri&ary tract i&fectio&s a&d
va"i&al yeast i&fectio&s
$# 4lycosuria a&d 'eto&uria
=# Si"&s of pre"&a&cy-i&duced hyperte&sio&
A# +olyhydram&ios
1B# 8etus lar"e for "estatio&al a"e
22# 3- hour "lucose tolera&ce test will be performed
to co&firm diabetes mellitusScree&i&" or i&itial
dia"&ostic test5ate si"&s
23#4lycosolated 7emo"lobi& less tha& =C
9E. 5B "ms oral "lucose challe&"e test
-est diet 3o&firmative test 25. Strict ,iabetic ,iet
2C. # #alories in diet should consist o% 6;F to L;F
car3oh1drates/ 12F to 2;F ,rotein/ and 2;F
to 5;F %at
NO"$1 Because insulin does not ,ass into the
3reast +il7/ 3reast%eeding is not contraindicated
%or the +other .ith dia3etes. (reastfeedin* is
encoura*edF it decreases the insulin re4uirements
for insulin.independent clients. (reastfeedin* does
not increase the ris8 of maternal infectionF it leads
to an increased caloric demand. Infants of diabetic
mothers often display ?itteriness in response to
hypo*lycemia after birth
-est diet for the disease< 26. %ell-bala&ced 3aloric ?iet
?isease complicatio& 2L. Mate%nal Com$licationsA P)*/ Placental
disorders/ still3irth/ +acroso+ia/ neural tu3e
de%ects.
2M. )etal Diabetic Com$licationsA
8acroso+ia
Pre-ecla+,sia
*1dra+nios
#ongenital ano+alies
N&2E"
2he incidence o% congenital ano+alies
a+ong in%ants o% dia3etic ,regnancies is three to
%our ti+es higher than that in general ,o,ulation
and is related to the high +aternal glucose levels
during the third to si$th gestational .ee7s.
1$=
-est side eKuipme&t 4lucometer
!&suli& .Kuipme&t
J1 Eternal Electronic (etal *eart ate
+onitoring
-est dru"
!&suli& therapy ( don=t use &ral h1,ogl1ce+ics/
the1 are 2eratogenic;
*ature of the dru" !&suli&
*ursi&" ?ia"&osis J1 *igh is7 %or %luid volu+e de%icit related to
,ol1uria and deh1dration
!mbala&ced &utritio& related to imbala&ced of
i&suli&, food a&d physical activity
+ote&tial heath care deficit related to physical
improveme&ts or social factors##
*ursi&" !&terve&tio& (6*64.(.*0
1# Screen clients 3et.een the 2Cth and 2Ath
.ee7s o% ,regnanc1
2# +re&atal visits bimo&thly for 6 mo&ths a&d
wee'ly thereafter#
3# #alories in diet should consist o% 6;F to
L;F car3oh1drates/ 12F to 2;F ,rotein/
and 2;F to 5;F %at
4# )bserve clie&t closely for a& i&suli& si&ce a
precipitous drop i& i&suli& reKuired is usual
5# (o&itor for si"&s of i&fectio& or post
hemorrha"e
6# If a pre*nant diabetic is in labor3 her blood
*lucose should be monitored hourly.
"he preferred method of administration if
insulin is re4uired durin* labor is intra#enous
)07./ !(+)/06*0 (6*64.(.*0<
,ri&e testi&"
-lood "lucose determi&atio&
!&suli& admi&istratio&
?ietary ma&a"eme&t
.1ercise
(etal surveillance"
(N Non-stress test N contraction stress test N
a+niocentesis:
(Pillitteri/ 8aternal and #hild Nursing/ ,.5CK-56L:
). CARDIAC DISEASE
356SS!8!360!)* .88.30S (6*64.(.*0
1$A
Class I 6symptomatic
Class II 6symptomatic at rest9
symptomatic with heavy physical activity
Class III 6symptomatic at rest9
symptomatic with ordi&ary activity
Class I; Symptomatic with all
activity9 symptomatic at rest
a# Class I< &o limitatio& of activities# *o
symptoms of cardiac i&sufficie&cy#
b# Class II< sli"ht limitatio& of activity,
6symptomatic at rest# )rdi&ary activities
causes fati"ue, palpitatio&s a&d dysp&ea
c# Class II< mar'ed limitatio& of activities,
comfortable at rest, less tha& ordi&ary
activities causes discomforts
d# Class I;< u&able to perform a&y physical
activity without discomfort# (ay have
the symptoms duri&" rest#
/etarded "rowth
8etal distress
"o relie#e fetal distress let
the patient lie on her side
+remature labor
5ou don/t ha#e to notify
the physician if the patient
complains of a
Nflutterin*O sensation in
her chest because of
ta8in* terbutaline
0(rethine7 SQ for
premature contractions
because it is a common
side effect unless #ital
si*ns indicate stress
4oal is to reduce wor'load
of heart
+romote rest
+romote a healthy diet
.ducate re"ardi&"
medicatio&
.ducate re"ardi&"
avoida&ce of i&fectio&
+romote reductio& of
physiolo"ic stress
PEGNAN#< )ND!#ED *<PE2ENS)&N (2&?E8)A &( PEGNAN#<:
*6(. )8 07.
?!S.6S.
+/.-.356(+S!6 .356(+S!6
(!5? S.@./.
Sy&o&ym :+/.4*6*3F-!*?,3.? 7F+./0.*S!)*;
+redisposi&" /
3o&tributi&" factors
-Pri+i,aras 1ounger than age 2; 1ears or older than C; 1ears' .o+en %ro+
lo. socioecono+ic 3ac7ground 3ecause o% ,oor nutrition' .o+en o% color'
.o+en .ith heart disease/ dia3etes .ith vessel or renal involve+ent/ and
essential h1,ertension' ,oor calciu+ and +agnesiu+ inta7e (Pillitteri/ A.
1KKK ,. 5K5: h1datidi%or+ +ole' +ulti,le gestation/ ,ol1h1dra+nios/ ,re-
e$isting vascular disease (2he Li,,incott 8anual o% Nursing Practice/ M
th
ed./
2;;1.,.11K;;
-A*e.related concern1 adolescents and primiparas o#er a*e CE are at hi*her
ris8 for preeclampsia.0 Nurses/ C minutes clinical pa*e IID .IIC
Author1 -loria % ,onnelly M.Phd
!&itial Si"& BO1C;@K; ++Gh on at
least t.o occasion O L
hours a,art (2he
5ippi&cott (a&ual of
*ursi&" +ractice $
th

ed#,2BB1#p#11AB;
protei&uria of 1-2V o& a
ra&dom sample9 wei"ht
"ai& over 2 lbs per wee'
i& seco&d trimester a&d
1 l3 ,er .7/ third
trimester9 mild edema i&
upper e1tremities or face
:+illitteri, a#,
1AAA#p#3A5;
BPO1L;@11; ++*g or
diastolic ,ressureO11;
++*g on t.o
occasions at least L
hours a,art .ith the
,atient on 3edrest'
,roteinuria O6 3@2C h
or 5P to CP on
Kualitative assessme&t
:uri&e dipstic'; :0he
5ippi&cott (a&ual of
&ursi&" +ractice $
th

ed#2BB1#p#11AB;
e$tre+e ede+a in
hands and
%ace@D,u%%inessD
:+illitteri,6#,1AAA#p#3A6;
Signs and s1+,to+s o%
severe ,re-ecla+,sia/
,.5K6' te+,erature
rises shar,l1 to 5K.CQ#
or C;R# (1;5R( to
1;CR(: %ro+ increased
cere3ral ede+a'
re%le$es 3eco+e
hyperactive p#3AA,
premo&itio& that
Ssomethi&" is
ha,,eningD' e,igastric
,ain and nausea'
urinar1 out,ut less than
5; +l@h p#4BB :+illitteri,
6#, 1AAA;
1=B
D
u
r
i
n
g

,
r
e
g
n
a
n
c
1
/
3
l
u
r
r
e
d

v
i
s
i
o
n

+
a
1
3
e
a

d
a
n
g
e
r
s
i
g
n

o
%
,
r
e
5ate Si"&
1=1
e
c
l
a
+
,
s
i
a

o
r
e
c
l
a
+
,
s
i
a
/
c
o
+
,
l
i
c
a
t
i
o
n
s
t
h
a
t
r
e
G
u
i
r
e
i
+
+
e
d
i
a
t
e
a
1=2
t
t
e
n
t
i
o
n

3
e
c
a
u
s
e
t
h
e
1
c
a
n

c
a
u
s
e
s
e
v
e
r
e
+
a
t
e
r
n
a
l
a
n
d

%
e
t
a
l
c
o
n
1=3
s
e
G
u
e
n
c
e
s
.
&
l
i
g
a
u
r
i
a

S
C
;
;

t
o

6
;
;

+
l
@
2
C
h
'
c
e
r
e
3
r
a
l
o
r
v
i
s
u
a
l
1=4
d
i
s
t
u
r
3
a
n
c
e
s
(
a
l
t
e
r
e
d

l
e
v
e
l
o
%
c
o
n
s
c
i
o
u
s
n
e
s
s
/
h
e
a
d
a
c
h
e
/
s
c
o
t
o
1=5
+
a
t
a
/
o
r
3
l
u
r
r
e
d

v
i
s
i
o
n
:
'
e
,
i
g
a
s
t
r
i
c
,
a
i
n

o
r

!
T

,
a
i
n
/
,
u
l
+
o
n
1=6
a
r
1
e
d
e
+
a

o
r
c
1
a
n
o
s
i
s
'
i
+
,
a
i
r
e
d

l
i
v
e
r
%
u
n
c
t
i
o
n

o
%
u
n
c
l
e
a
r
e
t
i
1=$
o
l
o
g
1
'
t
h
r
o
+
3
o
c
1
t
o
,
e
n
i
a

(
,
l
a
t
e
l
e
t
c
o
u
n
t
U
1
6
;
/
;
;
;
:
'
d
e
v
e
l
o
,
+
e
1==
n
t
o
%
e
c
l
a
+
,
s
i
a

:
0
h
e
5
i
p
p
i
&
c
o
t
t
(
a
&
u
a
l
o
f
*
u
r
s
i
&
"
+
r
a
c
t
i
c
e
,
$
th
e
d
1=A
#
2
B
B
1
#
p
#
1
1
A
B
;
e
l
e
v
a
t
e
d
s
e
r
u
m

c
r
e
a
t
i
&
i
&
e
P

1
#
2
m
"
/
d
l
9
c
a
r
d
i
a
c
1AB
i
&
v
o
l
v
e
m
e
&
t
9
e
1
t
e
&
s
i
v
e
p
e
r
'
i
p
h
e
r
a
l
e
d
e
m
a
:
+
i
l
l
i
t
t
e
r
i,
a
#,
1
A
A
A
#
p
1A1
#
3
A
5
;
3ardi&al /
+atho"&omo&ic/ (aor
Si"&Si"&s of %orse&i&"
+!7 or !mpe&di&"
Sei2ures
(P 9:B>99B mm +* or
abo#e
$pi*astric pain
,ecreased urinary
output
&isual chan*es
+eadache
*1,ertension and ,roteinuria are the +ost signi%icant. Ede+a is signi%icant
onl1 i% h1,ertension and ,roteinuria or signs o% +ulti-organ s1ste+
involve+ent are ,resent. (Pillitteri/ A./ 1KKK.,.5KC:
*ursi&" ?ia"&osis a&d
*ursi&" !&terve&tio&s
(luid volu+e e$cess
related to
,atho,h1siologic
changes o% P)* and
increased ris7 o% %luid
overload.
6ltered tissue
perfusio&, 8etal
cardiac a&d cereral,
related to altered
place&tal blood flow
caused by vasospasm
a&d thombosis#
/is' for i&ury related
to co&vulsio&s#
?ecreased cardiac
output related to
decreased preload or
a&tihyperte&sive
therapy#
(ai&tai&i&" 8luid -ala&ce
1# 3o&trol !@ i&ta'e usi&" a co&ti&uous i&fusio&
pump#
2# (o&itor i&put a&d output strictly9 &otify health
care provider if uri&e output is Y3B ml/h#
3# (o&itor hematocrit levels to evaluate
i&travascular fluid status#
4# (o&itor vital si"&s every hour#
5# 6uscultate breath sou&ds every 2 hours, a&d
report si"&s of pulmo&ary edema :whee2i&",
crac'les, short&ess of breath, i&creased pulse
rate, i&creased respiratory rate;#
+romoti&" 6deKuate 0issue +erfusio&
1# +ositio& o& side, preferably the left side to
promote place&tal perfusio&#
2# (o&itor fetal activity#
3# .valuate *S0 to determi&e fetal status#
4# !&crease protei& i&ta'e to replace protei& lost
throu"h 'id&eys#
+reve&ti&" !&ury
1# !&struct o& the importa&ce of reporti&"
headaches, visual cha&"es, di22i&ess, a&d
epi"astric pai&#
2# !&struct to lie dow& o& left side if symptoms
are prese&t#
3# Jeep the e&viro&me&t Kuiet a&d as calm as
possible#
4# !f patie&t is hospitali2ed, side rails should be
padded a&d remai& up to preve&t i&ury if
sei2ure occurs#
1A2
*)0.< 2he ,atient .ith a diagnosis o% P)* should
3e close to the nurses= station 3ecause she reGuires
close o3servation. 2he ,atient also should 3e
,laced in a roo+ .ith decreased sti+uli.
(ai&tai&i&" 3ardiac )utput
1# (o&itor !@ i&ta'e usi&" a co&ti&uous i&fusio&
pump#
2# (o&itor i&put a&d output strictly9 &otify
primary care provider if uri&e output is Y 3B
ml/h#
3# (o&itor mater&al vital si"&s9 especially mea&
blood pressure a&d respiratio&s#
4# 6ssess edema status, a&d report pitti&" edema
of Z V 2 to primary care provider#
5# (o&itor o1y"e&atio& saturatio& levels with
pulse o1imetry# /eport o1y"e&atio& saturatio&
rate of YABC to primary care provider#
:0he 5ippi&cott (a&ual of *ursi&" +ractice, $
th
ed#,
2BB1#pp#11A2-11A3;
Scree&i&"/!&itial dia"&ostic test
3o&firmatory
0est-lood pressure
over 14B/AB, or
i&crease of 3B mm
systolic, 15 mm
diastolic over pre-
pre"&a&cy level#
:+illitteri, 6#
1AAA#p#3A5;
2C-hour urine %or ,rotein o% 5;; +g or greater' elevated seru+ B!N and
creatinine' increased dee, tendon re%le$es and clonus' 3lood ,ressure
changes +eeting criteria %or diagnosis :0he 5ippi&cott (a&ual of *ursi&"
+ractice,$
th
ed#, 2BB1#p#11AB;
-est ?iet
Disease
#o+,lications2he
.o+an needs a
+oderate to high-
,rotein/ +oderate-
sodiu+ diet to
co+,ensate %or the
,rotein she is losing#
:+illitterri, 6#,
1AAA#p#3A=;
A3ru,tio ,lacentae (+ypertension in PI+ leads to #asopasm. "his in turn
causes the placenta to tear a)ay from the uterine )all 0abrupto placentae7
0Mosby/s Comprehensi#e e#e) of Nursin* for NCL$'. N p. DD:7
disse+inated intravascular coagulation' *ELLP s1ndro+e' ,re+aturit1'
intrauterine gro.th restriction ()!G: from decreased place&tal perfusio&9
mater&al/fetal death9 hyperte&sive crisis9 acute re&al failure9 hemorrha"e9
cerebrovascular accide&t9 bli&d&ess9 hypo"lycemia9 hepatic rupture :0he
5ippi&cott (a&ual of *ursi&" +ractice,$
th
ed#, 2BB1#pp#11A2;
-est +ositio& SEVEE PE#LA8PS)A" Lateral recu+3ent ,osition
(Pillitteri/A./1KKK.,.5KM: E#LA8PS)A" to ,revent as,iration/ turn the .o+an
on her side to allo. secretions to drain %ro+ her +outh.
(Pillitteri/A./1KKK.,.C;;:
-eside .Kuipme&t !&fusio& pump9 pulse o1imeter :0he 5ippi&cott (a&ual of *ursi&" +ractice,$
th

ed#,2BB1#pp#11A2-11A3;
-est ?ru" ("&esium sulfate< 4-6 loadi&" dose of 5BC "ive !@ over 15-3B mi&s followed
by a mai&te&a&ce dose :seco&dary i&fusio&; of 1-4 "/h or !( i&ectio& or 1B "
:5 " i& each buttoc'; as a loadi&" dose followed by 5 " every 4 hours :0he
1A3
5ippi&cott (a&ual of *ursi&" +ractice,$
th
ed#, 2BB1#pp#11AB;s
Ad+inister antih1,ertensives such as h1drala9ine (A,resoline: as ,rescri3ed/
to ,revent a cere3rovascular accident
*ature of the ?ru" -est tocolytic a"e&t9 a&tihyperte&sive9 a&tico&vulsa&t/eclampsia
J1 #o+,lication o% 8gS&C is " es,irator1 De,ression
RI-RIT= DRUG
ASSESSMENTA
SIDE E))ECT
efle@es3 respiration and urinary output are priority assessments
durin* administration of ma*nesium sulfate therapy in patients )ith
PI+. If the patient/s ma*nesium le#els increase abo#e the therapeutic
ran*e 0C to A +g@dl:/ the absence of refle@es is often the first indication
of to@icity. efle@es often disappear at serum ma*nesium le#els of ; to
9B m*>dl. espiratory depression occurs at le#els of 1; to 16 +g@dl/
and cardiac conduction ,ro3le+s occur at levels o% 16 +g@dl and
higher. !rinar1 out,ut o% less than 5;+l@hour +a1 result in the
accu+ulation o% to$ic levels o% +agnesiu+.
/efere&ces +illitteri, 6# 1AAA# (ater&al a&d 3hild 7ealth *ursi&", 3are of the 3hildbeari&"
& 3hidlreari&" 8amily, 3
rd
ed# 5ippi&cott %illiams & %il'i&s< +hiladelphia,
,S6#
0he 5ippi&cott (a&ual of *ursi&" +ractice, $
th
ed#, 2BB1#5ippi&cott %illiams &
%il'i&s< +hiladelphia, ,S6#
+roper 6ssessme&t of
6b&ormal /efle1es
Assess+ent Patellar e%le$es
osition t!e client <it! le"s &an"lin" o(e% t!e e&"e of t!e e+aminin" table
o% lyin" on bac2 <it! le"s sli"!tly.
St%i2e t!e $atella% ten&on L'st belo< t!e 2neeca$ <it! t!e $e%c'ssion
!amme%.
No%mal Res$onseA )le+ion of t!e a%m at t!e elbo<.
#lonus
osition t!e client <it! le"s &an"lin" o(e% t!e e&"e of t!e e+aminin" table.
S'$$o%t t!e le" <it! one !an& an& s!a%$ly &o%sifle+ t!e clientHs foot <it!
t!e ot!e% !an&.
Maintain t!e &o%sifle+e& $osition fo% a fe< secon&sM t!en %elease t!e foot.
No%mal Res$onseA (Negative #lonus es,onse:
)oot <ill %emain stea&y in t!e &o%sifle+e& $osition.
No %!yt!mic oscillation of Le%2in" of t!e foot <ill be felt.
?!en %elease&D t!e foot <ill &%o$ to a $lanta% fle+e& $osition <it! no
oscillations.
A3nor+al es,onse" (Positive #lonus es,onse:
R!yt!mic oscillations <!en t!e foot is &o%sifle+e&.
Simila% oscillations <ill be note& <!en t!e foot &%o$s to t!e $lanta% fle+e&
$osition.
G. 1LEEDING DIS-RDERS A))ECTING THE LACENTA
1A4
lacentaA co&tai&s 2B cotyledo&s, wei"hs 4BB-6BB "rams# ?evelops o& the 3
rd
mo&th# 8orm from
3horio&ic villi & deciduas basalis# ?eciduas :mea&i&" e&dometrial cha&"es & "rowth;
)'nctionsA (ai& source of &ourishme&t & acts a tra&sfer or"a& for metabolic purposes for the fetus#
lacental %oblem
+lace&tal separatio& is characteri2ed by a sudde& "ush or tric'le of blood from the va"i&a, further protrusio& of
the umbilical cord from the va"i&a, a "lobular-shaped uterus, a&d a& i&crease i& fu&dal hei"ht# %ith cervical or
va"i&al laceratio&, the &urse &otes a co&siste&t flow of bri"ht red blood from the va"i&a# %ith postpartum
hemorrha"e, usually caused by uteri&e ato&y, the uterus is&Lt "lobular# ,teri&e i&volutio& ca&Lt be"i& u&til the
place&ta has bee& delivered#
lacenta %e(ia :lo< im$lantation*
A
b
%
'
$
t
i
o

l
a
c
e
n
t
a
:

%
e
m
a
t
'
%
e
s
e
$
a
%
a
t
i
o
n
*
AssessmentA
. ainless
. Hea(y blee&in"
. SoftD non ten&e%D %ela+e& 'te%'s </ no%mal tone
. S!oc2 in $%o$o%tion to obse%(e& bloo& loss
. Si"ns of fetal &ist%ess 's'ally not $%esent
Asses
smen
tA
%e&is$osin" )acto%sA
D (ultiparityD 6dva&ci&" mater&al a"e, D (ultiple
%e&is$osin" )acto%sA
D 3hro&ic 7yperte&sive diseaseD history of a short
1A5
.
Se
(e
%el
y
$ai
nf
'l
.
He
a(
y
ble
e&i
n"
</
c
ma
yb
e
$a
%ti
all
yNc
om
$le
tel
y
!i
&&
en
.
Ri
"i&
:b
oa
%&
li2
e*D
ten
&e
%
'te
%'
s
$o
ssi
ble
</
co
nt%
act
ion
s
.
"estatio&D 6lteratio& i& the uteri&e structures cord D (ulti"ravida D trauma
1A6
S!
oc
2
see
mi
n"
to
be
o'
t
of
$%
o$
o%t
ion
.
Si"
ns
of
fet
al
&is
t%e
ss
N'%sin" Consi&e%ationsA
. 3lie&t is hospitali2ed a&d put o& bed rest
# 3o&ti&ually mo&itor fetal well- bei&"
# 3aesarea& delivery i&dicate
# (easure blood loss throu"h peri&eal pad cou&ts
# *) va"i&al e1ams
# +rovide emotio&al support
N'%sin" Consi&e%ationsA
# -ed rest i& wed"e positio& too preve&t supi&e
hypote&sio&
.
# 3o&ti&ually mo&itor fetal well- bei&"
# 0reat si"&s of shoc' a&d hemorrha"e
# +rovide emotio&al support
# +repare for delivery
LACENTA RE;IA

8i"ure 2B a
+redisposi&" 8actorP
(ater&al a"eP !mproperly
impla&ted place&ta i& the
lower uteri&e se"me&t &ear
or over the i&ter&al cervical
os
P +arity :&o# )f
pre"&a&cy;
P +revious uteri&e sur"ery
P 0otal< the i&ter&al os
is e&tirely covered
by the place&ta whe&
cervi1 is fully dilated
P (ar"i&al< o&ly a&
+563.*06 +/.@!6
1A$
ed"e of the place&ta
e1te&ds to the
i&ter&al os
P 5ow-lyi&" place&ta<
impla&ted i& the lower
uteri&e se"me&t but does
&ot reach the os
:Sau&ders pa"e 2AA;
3omplicatio&P a&emia,
I1hemorrha*e, I2shoc8,
re&al failure, I3
disseminated intra#ascular
coa*ulation, cerebral
ischemia, mater&al a&d fetal
death :*ursi&" 6lert
p#41=;0herapeutic
!&terve&tio&sP
,ltraso&o"raphy to co&firm
the pressure of place&ta
previa#P +ai&less bleedi&"
as early as $ mo&ths :mild
to hemorrha"e;?efi&itio&
P ?epe&ds o& locatio& of
place&ta, amou&t of
bleedi&" a&d status of the
fetus#
P 7ome mo&itori&" with
repeated ultrasou&ds may
be possible with type !-
low lyi&"
P 3o&trol bleedi&"
P /eplace blood loss if
e1cessive
P 3esarea& birth if
&ecessary
P -etamethaso&e is
i&dicated to i&crease fetal
lu&" maturity# :(osby,
3omprehe&sive p# 2B3;
P Soft uterus
P 6bdomi&al fetal
positio& of breech or
tra&sverse lie
P ,teri&e co&tractio&s
P 6&emic
Best PositionV Le%t lateral
,osition #on%ir+ator1
2estV !ltrasound %or
,lacenta
locali9ation2he ,atient
.ith ,lacenta ,revia
should 3e +aintained
J1 N!S)NG D)AGN&S)S" Potential %luid volu+e de%icit
P (ai&tai& bed rest
P J1 Assess+ent - 8onitor +aternal vital signs/ (*/ and %etal activit1
P 6ssess bleedi&" :amou&t a&d Kuality;
P (o&itor a&d treat si"&s of shoc'
P 6void va"i&al e1ami&atio& if bleedi&" is occurri&"
P +repare for premature birth or cesarea& sectio&
P 6dmi&ister !@ fluids as ordered
P 6dmi&ister iro& suppleme&ts or blood tra&sfusio& as ordered :mai&tai&
1A=
on 3ed rest/ ,re%era3l1
in a side-l1ing
,osition. Additional
,ressure %ro+ an
u,right ,osition +a1
cause %urther tearing
o% the ,lacenta %ro+
the uterine lining.
A+3ulating .ould
there%ore 3e indicated
%or this ,atient.
Per%or+ing a vaginal
e$a+ination and
a,,l1ing internal scal,
electrode could also
cause the ,lacenta to
3e %urther torn %ro+
the uterine
lining.*ursi&"
?ia"&osis with3ardi&al
(a&ifestatio&
*)0.<
Manual pel#ic
e@aminations are
contraindicated )hen
#a*inal bleedin* is
apparent in the third
trimester unit a
dia*nosis is made and
placenta pre#ia is ruled
out. ,i*ital e@amination
of the cer#i@ can lead to
maternal and fetal
hemorrha*e. A dia*nosis
of placenta pre#ia is
made by ultrasound. "he
hemo*lobin and
hematocrit le#els are
monitored and e@ternal
electronic fetal heart
rate monitorin* is
initiated. $lectronic fetal
monitorin* 0e@ternal7 is
crucial in e#aluatin* the
status of the fetus )ho is
at ris8 for se#ere
hypo@ia. 0Saunders
Comprehensi#e DBBD
$dition3 p. CBI7
*ursi&" !&terve&tio&
hematocrit level;
P +repare to admi&ister /h immu&e "lobuli&
1AA
-.S0+)S!0!)*
A1RUTI- LACENTAE
8i"ure 21
A1RUTI- LACENTAE
?efi&itio& +remature separatio& of the place&ta from the uteri&e
wall after the 2B
th
wee' of "estatio& a&d before the
fetus is delivered :Sau&ders pa"e 2AA-3BB;
Sy&o&yms
P
+redisposi&" 8actor
2BB
+
la
c
e
&
ta
l
a
b
r
u
p
ti
o
&
P
+r
e
m
at
ur
e
se
p
ar
at
io
&
of
pl
ac
e
&t
a
Pathophysiolo*yR Maternal a*e
R Parity
R Pre#ious abruptio placentae3 multifetal *estation
R +ypertension
NO"$1
Abruptio placentae is associated )ith conditions
characteri2ed by poor uteroplacental circulation3
such as hypertension3 smo8in* and alcohol or
cocaine abuse. It is also associated )ith physical
and mechanical factors such as o#er distension of
the uterus that occurs )ith multiple *estation or
polyhydranions. In addition3 a short umbilical cord3
physical trauma3 and increased maternal a*e and
parity are ris8 factors. 0Saunders Comprehensi#e
DBBD $dition3 p. CBE7
R Spontaneous rupture of blood #essels at the
placental bed may due to lac8 of resiliency or to
abnormal chan*es in uterine #asculature.
R May be complicated by hypertension or by an
enlar*ed uterus that can/t contract sufficiently to
seal off the torn #essels
R Conse4uently3 bleedin* continues unchec8ed3
possibly shearin* off the placenta partially or
completely. 0Nursin* Alert p.I7
(a&ifestatio&
2B1
3omplicatio&J ainf'l (a"inal blee&in"
P 7yperto&ic to teta&ic, e&lar"ed uterus
P Board-li7e rigidit1 o% a3do+en (#ullen Sign:
P 6b&ormal/abse&t fetal heart to&es
P +allor
P 3ool, moist s'i&
P -loody am&iotic fluid
P /isi&" fu&dal hei"ht from blood trapped behi&d
the place&ta
P Si"&s of shoc'
P (a&ifestatio& of coa"ulopathy
NO"$1
Uterine tenderness accompanies placental
abruption3 especially )ith a central abruption and
trapped blood behind the placenta. "he abdomen
)ill feel hard and boardli8e upon palpation as the
blood penetrates the myometrium and causes
uterine irritability. Obser#ation of the fetal
monitorin* often re#eals increased uterine restin*
tone3 caused by failure of the uterus to rela@ in an
attempt to constrict blood #essels and control
bleedin*. 0Saunders Comprehensi#e DBBD $dition3 p.
CBI7
R +emorrha*e3 shoc83 renal failure3 disseminated
intra#ascular coa*ulation3 maternal death3 fetal
death0Nursin* Alert p.I7
"herapeutic Inter#entions
P
/
e
p
la
c
e
m
e
&
t
o
f
b
l
o
o
d
l
o
s
s#
P
%
it
h
*ursi&" ?ia"&osis with !&terve&tio&
2B2
m
o
d
er
at
e
or
se
v
er
e
se
p
ar
at
io
&
or
m
at
er
&
al
or
fe
ta
l
di
st
re
ss
<
e
m
er
"
e
&
c
y
c
hi
ld
bi
rt
h#
*
)
0
.<
2h
e
go
al
o%
+
2B3
an
ag
e
+
en
t
in
a3
ru
,ti
on
,l
ac
en
ta
e
is
to
co
nt
rol
th
e
he
+
or
rh
ag
e
an
d
de
liv
er
th
e
%et
us
as
so
on
as
,o
ssi
3l
e.
De
liv
er
1
is
th
e
tre
at
+
2B4
en
t
o%
ch
oi
c
i%
th
e
%et
us
is
at
ter
+
ge
sta
tio
n
or
i%
th
e
3l
ee
di
ng
is
+
od
er
at
e
to
se
ve
re
an
d
+
ot
he
r
or
%et
us
is
in
0e
o,
ar
d1.
(S
au
nd
er
2B5
s
#o
+
,r
eh
en
siv
e
2;
;2
Ed
iti
on
/
,.
5;
C:
P
%
it
h
m
il
d
se
p
ar
at
io
&
w
it
h
o
ut
fe
ta
l
di
st
re
ss
a
&
d
i&
th
e
pr
es
e
&
ce
of
so
m
2B6
e
ce
rv
ic
al
ef
fa
ce
m
e
&t
a
&
d
di
la
ta
ti
o
&<
i&
d
u
ct
io
&
of
la
b
or
m
a
y
b
e
at
te
m
pt
e
d
P)1
y
"
e
&
if
&
ec
es
sa
ry
P
(
ai
&t
2B$
e
&
a
&
ce
of
fl
ui
d
a
&
d
el
ec
tr
ol
yt
es
b
al
a
&
ce
#
:
(
os
b
y,
3
o
m
pr
e
h
e
&s
iv
e
p#
2
B
4;
I1 *,/S!*4 ?!64*)S!S< /is' for fluid volume
deficit
V J1 Assess+ent" 8onitor and (*
P 6ssess for va"i&al bleedi&", abdomi&al pai&, a&d
i&crease i& fu&dal hei"ht
P (ai&tai& bed rest
P 6dmi&ister o1y"e& as prescribed
P (o&itor a&d report a&y uteri&e activity
P 6dmi&ister !@ fluid as prescribed
P (o&itor ! & )
P 6dmi&ister blood products as prescribed
P (o&itor blood studies
P +repare for the delivery of the fetus as Kuic'ly as
2B=
possible
P (o&itor for si"&s of dissemi&ated i&travascular
coa"ulatio& i& the post-partum period
3o&firmatory 0est
P ,ltrasou&d detects retro-place&tal bleedi&"
;ENA CA;A S=NDR-ME
?efi&itio&
0
h
e
v
e
&
o
u
s
r
et
u
r
&
t
o
t
h
e
h
e
a
rt
is
i
m
p
ai
r
e
d
b
y
t
h
e
w
ei
"
Sy&o&ym
2BA
h
t
o
f
u
te
r
u
s#
S
u
p
i
&
e
7
y
p
o
te
&
si
v
e
S
y
&
d
r
o
m
e
+redisposi&" factors
0
h
r
o
m
b
o
p
h
le
b
it
is
*
)
!&itial si"&
21B
0
.<
3
o
&
t
r
i
b
u
t
e
t
o
c
l
o
t
f
o
r
m
a
ti
o
&
m
o
ti
o
&
i
&
c
l
u
d
e
i
&
a
c
ti
v
it
y
,r
e
d
u
c
e
d
c
o
r
211
d
i
a
c
o
u
t
p
u
t,
c
o
m
p
r
e
s
s
i
o
&
o
f
t
h
e
v
i
e
&
s
i
&
p
e
l
v
i
s
o
r
l
e
"
s
"
h
e

m
o
s
t

l
212
i
8
e
l
y

c
a
u
s
e

o
f

s
u
p
i
n
e

h
y
p
o
t
e
n
s
i
o
n

i
s

f
e
e
l
i
n
*

d
i
2
2
y
3

s
h
213
o
r
t

o
f

b
r
e
a
t
h

a
n
d

c
l
a
m
m
y

)
h
e
n

l
y
i
n
*

b
a
c
8

f
o
r

l
o
n
*

p
e
r
i
214
o
d
s

o
f

t
i
m
e

i
n

p
a
t
i
e
n
t
s

:
t
h

m
o
n
t
h

o
f

p
r
e
*
n
a
n
c
y
.
"
h
e
c
a
u
215
s
e
o
f
s
u
p
i
n
e
h
y
p
o
t
e
n
s
i
o
n

d
u
r
i
n
*

p
r
e
*
n
a
n
c
y
i
s
t
h
e
)
e
i
*
h
t
o
f
t
h
e
u
t
216
e
r
u
s
c
o
m
p
r
e
s
s
e
s
t
h
e
i
n
f
e
r
i
o
r
#
e
n
a

c
a
#
a
3
d
e
c
r
e
a
s
i
n
*

t
h
e
r
e
t
u
r
n

21$
o
f
b
l
o
o
d

t
o

t
h
e
h
e
a
r
t
3
t
h
u
s
d
e
c
r
e
a
s
i
n
*

c
a
r
d
i
a
c
o
u
t
p
u
t
3
)
h
i
c
h

l
21=
o
)
e
r
s
t
h
e
b
l
o
o
d

p
r
e
s
s
u
r
e
8
at
i
K
u
e,
p
r
o
1
y
m
al
&
o
ct
u
r
&
al
d
y
s
p
&
e
a,
o
5ate Si"&
21A
rt
h
o
p
&
e
a,
h
y
p
o
1
ia
,
c
y
a
&
o
si
s
3ardi&al si"&/educe re&al perfectio&,
?ecrease "lomerular filtratio&
shoc' such as tachycardia
*)0.<
3aused by reduced cardiac output, respiratory distress, fatal
distress
!&itial / Scree&i&" test
3o&firmatory test870 mo&itor
*)0.<
6bove 16B or below 12B beats per mi&utes,
8etal +7 below $#5
6m&iotomy<
*)0.<
6bove 'eepi&" the si"&ifica&t other improved of the
pro"ress of care, the fatal status would he the priority
*ursi&" ?ia"&osis

6lter
*ursi&" !&terve&tio&
22B
/
i
s
'
f
o
r
a
l
t
e
r
e
d
7
e
a
l
t
h
m
a
i
&
t
e
&
a
&
c
e
r
e
l
a
221
t
e
d
t
o
i
&
s
u
f
f
i
c
i
e
&
t
'
&
o
w
l
e
d
"
e
o
f
t
r
e
a
t
m
e
&
t
s
,
d
r
u
"
t
h
e
r
a
p
i
e
s
,
h
o
m
222
e
c
a
r
e
m
a
&
a
"
e
m
e
&
t
a
&
d
p
r
e
v
e
&
t
i
o
&
o
f
f
u
t
u
r
e
i
&
f
e
c
t
i
o
&
6
l
t
e
r
e
d
c
o
m
f
223
o
r
t
r
e
l
a
t
e
d
t
o
m
a
l
a
d
a
p
t
i
v
e
c
o
p
i
&
"

3lose
-est maor Sur"ery
224
(
a
i
&
t
a
i
&
p
a
t
i
e
&
t
o
&
b
e
d
225
r
e
s
t
t
o
r
e
d
u
c
e
)
1
y
"
e
&
d
e
m
a
&
d
s
a
&
d
r
i
s
'
f
o
r
b
l
e
e
d
i
&
"
#
(
o
&
i
t
o
r
p
r
e
s
c
226
r
i
b
e
d
m
e
d
i
c
a
t
i
o
&
"
i
v
e
&
t
o
p
r
e
s
e
r
v
e
r
i
"
h
t
@
e
&
t
r
i
c
u
l
a
r
f
e
l
l
i
&
"
p
r
e
22$
s
s
u
r
e
a
&
d
i
&
c
r
e
a
s
e
b
l
o
o
d
p
r
e
s
s
u
r
e
!
&
s
t
r
u
c
t
p
a
t
i
e
&
t
i
&
s
e
l
f
>
c
a
r
e
a
22=
c
t
i
v
i
t
i
e
s
+
r
o
v
i
d
e
i
&
f
o
r
m
a
t
i
o
&
a
b
o
u
t
a
&
t
i
s
m
o
'
i
&
"
s
t
r
a
t
e
"
i
e
s
a
&
d
22A
a
l
l
o
w

p
a
t
i
e
&
t
t
i
m
e
t
o
r
e
t
u
r
&
d
e
m
o
&
s
t
r
a
t
i
o
&
o
f
t
r
e
a
t
m
e
&
t
t
o
t
h
e
d
o
23B
&
e
a
t
h
o
m
e
6
s
s
e
s
s
p
h
y
s
i
c
a
l
c
o
m
p
l
a
i
&
t
s
m
a
t
t
e
r
s
o
f
f
a
c
t
s
w
i
t
h
o
u
t
e
m
p
231
h
a
s
i
2
i
&
"
c
o
&
c
e
r
&
#
,
s
e
d
e
e
p
>
b
r
e
a
t
h
i
&
"
,
m
u
s
c
l
e
r
e
l
a
1
a
t
i
o
&
,
a
&
d
i
m
232
a
"
e
r
y
t
o
r
e
l
i
e
v
e
d
i
s
c
o
m
f
o
r
t#
.
1
p
r
e
s
s
a
c
a
r
i
&
"
a
t
t
i
t
u
d
e
3
a
e
s
a
ri
a
&
-est dirt for pre-operative
233
S
e
ct
i
o
&
>
&
o
te
if
c
e
r
v
i
1
is
i
&
c
o
m
p
le
te
d
el
et
e
d#
8
o
o
d

a
&
d

f
l
u
i
d

a
r
e

-est diet for ?isease 7ypoaller"e&ic !o&ic diet 3alcium i&creased
234
w
i
t
h
h
e
l
d

b
e
f
o
r
e

i
&
v
a
s
i
v
e

p
r
o
c
e
d
u
r
e

i
s

&
o
t
r
e
s
u
m
e
d

u
&
t
i
l
t
h
235
e

c
l
i
e
&
t
i
s

s
t
a
b
l
e

a
&
d

f
r
e
e

o
f
&
a
u
s
e
a

&

v
o
m
i
t
i
&
"
#
+ossible Sur"ical 3omplicatio&
!
&
te
rr
u
p
3omplicatio& of ?isease
236
ti
o
&
o
f
v
e
&
a
c
a
v
a,
w
h
ic
h
r
e
d
u
c
e
c
h
a
&
&
el
si
2
e#
P
-leedi
&" as
a
result
of
treatm
e&t
*)0.
<
)b
ser
vat
io&
of
the
fet
-est positio& pre-operative Sims +ositio&
*)0.<
2urning to the le%t side to shi%t right o% the %etus o%%
the in%erior vena cava#
23$
al
mo
&it
ori
&"
oft
e&
rev
eal
i&c
rea
se
ute
ri&
e
rus
tli&
"
to&
e,
cau
sed
by
fail
ure
of
the
ute
rus
to
rel
a1
i&
a&
att
em
pt
to
co
&st
rict
blo
od
ves
icl
e
a&
d
co
&tr
ol
ble
edi
&"
P
/espir
23=
atory
failure
#
-ed Side .Kuipme&t )1y"e& obtai& eKuipme&t for e1ter&al electro&ic fetal
heart rate mo&itori&" )1y"e& with 3a&&ula
7istory of ?isease
6&"i&a, myocardial i&farctio&
)verwhelmi&"
i&fectio&s
particularly bacterial
sepsis9 J1 a3ru,tion
,lacenta' ecla+,sia'
am&iotic fluid
embolism9 !,8?:!&tra-
uteri&e fetal death; or
rete&tio& of dead fetus9
bur&9 trauma9 fractures9
maor sur"ery9 fat
embolism9 soc'9
hemolytic tra&sfusio&
reactio&9 mali"&a&cies
particularly of lu&",
colo&, stomach, a&d
pa&creasName of the
,isease
NO"$1
,isseminated
intra#ascular
coa*ulation 0,IC7 is a
state of diffuse clottin*
in )hich clottin* factors
are consumed. "his
leads to )idespread
bleedin*. Platelet are
decreased because they
are consumed by the
process3 coa*ulation
studies sho) no clot
formation 0and are thus
normal to prolon*ed7F
and fibrin plu*s may
clo* the
micro#asculature
diffusely3 oo2in* from
in?ection sites3 and
presence of hematuria
are si*ns associated
)ith the presence of
,IC. S)ellin* and pain
in the calf of one le* are
more li8ely to be
Disse+inated )ntravascular coagulation
23A
associated )ith
thrompophlebitis.
0Saunders
Comprehensi#e DBBD
$dition3 p. CBI7
!&itial
Si"&+redisposi&" /
3o&tributi&" 8actors
3ool&ess a&d mottli&" of e1tremities9 pai&9 dysp&ea9 ab&ormal bleedi&"
5ate Si"& 6ltered me&tal status9 acute re&al failure
*ursi&" ?ia"&osis &
!&terve&tio&
/is' for i&ury
related to bleedi&" due to
thrombocytope&ia
6ltered tissue
perfusio& :all tissues;
related to ischemia due to
microthrombi formatio&
(i&imi2i&" -leedi&"
1# !&stitute -leedi&" precautio&s
2# (o&itor pad cou&t/amou&t of saturatio&
duri&" me&ses9 admi&ister or teach self-
admi&istratio& of hormo&es to suppress
me&struatio& as prescribed#
3# 6dmi&ister blood products as ordered#
(o&itor for si"&s a&d symptoms of aller"ic
reactio&s, a&aphyla1is, a&d volume
overload#
4# 6void dislod"i&" costs# 6pply pressure to
sites of bleedi&" for at least 2B mi&s, use
topical hemostatic a"e&ts# ,se tape
cautiously#
5# (ai&tai& bed rest duri&" bleedi&" episode#
6# !f i&ter&al bleedi&" is suspected, assess
bowel sou&ds a&d abdomi&al "irth#
$# .valuate fluid status a&d bleedi&" by
freKue&t measureme&t fo vital si"&s, ce&tral
ve&ous pressure, i&ta'e a&d output#
+romoti&" 0issue +erfusio&
1# Jeep patie&t warm
2# 6void vasoco&strictive a"e&ts :systemic or
topical;#
3# 3ha&"e patie&tEs positio& freKue&tly a&d
perform /)( e1ercises#
4# (o&itor electrocardio"ram a&d laboratory
test for dysfu&ctio& of vital or"a&s casued by
ischemia > arrhythmias, ab&ormal arterial
blood "ases, i&creased blood urea &itro"e&
a&d creati&i&e#
5# (o&itor for si"&s of vascular occlusio& a&d
report immediately#
a# -rai& > decreased level of co&scious&ess,
se&sory a&d motor deficits,
sei2ures, coma#
b# .yes > @isual deficits#
c# -o&e > +ai&
d# +ulmo&ary vasculature > chest pai&,
short&ess of breath, tachycardia#
e# .1tremities > cold, mottli&", &umb&ess#
f# 3oro&ary arteries > chest pai&,
arrhythmias#
"# -owel > pai&, te&der&ess, decreased
24B
bowel sou&ds#

Scree&i&" or !&itial
?ia"&ostic 0est
+09 +009 +latelet cou&t :Smelt2er, S#3# & -are, -#4#, 1AA2#p#=11;
3o&firmative 0est ?ecreased 8ibri&o"e& level9 i&creased fibri& split products9 decreased a&ti-
thrombi& !!! level
-eside .Kuipme&t .349 3@+
-est ?ru"
7
e
p
a
r
i
&

i
&
h
i
b
i
t
s
c
l
o
t
t
i
&
"

c
o
m
p
o
&
e
&
t
s
o
f
?
*ature of the ?ru"
241
!
3
6
&
t
i
c
o
a
"
u
l
a
&
t
/efere&ces
Smelt3e%DS.C.& 1a%eD 1.G. #77,. 1%'nne% an& S'&&a%t!Hs Te+boo2 of Me&ical9S'%"ical N'%sin"D 5
t!
e&.
>.1. Li$$incott com$anyA !ila&el$!iaD USA.
Hy$e%emesis "%a(i&a%'m
7yperemesis "ravidarum is persiste&t, u&co&trolled vomiti&" that be"i&s i& #the first wee's of pre"&a&cy a&d
may co&ti&ue throu"hout pre"&a&cy# ,&li'e Smor&i&" sic'&ess,T hyperemesis ca& have serious complicatio&s,
i&cludi&" severe wei"ht loss, dehydratio&, a&d electrolyte imbala&ce#
N&2E" 2he de%ining %actor %or h1,ere+esis gravidaru+ should 3e the ti+e o% occurrence E and that is the
2nd tri+ester/ usuall1 the 1C E 1L
th
.ee7. )% this is on the 1
st
tri+ester/ usuall1 this is +orning sic7ness.
Ca'ses
4o&adotropi&e productio&
+sycholo"ical factors
0rophoblastic activity
Assessment )in&in"s
3o&ti&uous, severe &ausea a&d vomiti&"
?ehydratio&
?ry s'i& a&d mucous membra&es
.lectrolyte imbala&ce
(etabolic acidosis
*o&-elastic s'i& tur"or
)li"uria
Dia"nostic Test Res'lt
6rterial blood "as a&d a&alysis reveals al'alosis#
7b level a&d 730 are elevated#
Serum potassium level reveals hypo'alemia
,ri&e 'eto&e levels are elevated#
,ri&e specific "ravity is i&creased#
N'%sin" Dia"noses
8luid volume deficit
6ltered &utritio&9 less tha& body reKuireme&ts
+ai&
242
T%eatment
0otal pare&teral &utritio& :0+*;
/estoratio& of fluid a&d electrolyte bala&ce
D%'" T!e%a$y
6&ti-emetics, as &ecessary for vomiti&", for e1ample +lasil , 7ydro1y2i&e a&d +rochlorpera2i&e
Inte%(ention an& Rationales
(o&itor vital si"&s a&d fluid i&ta'e a&d output to assess for fluid volume deficit#
)btai& blood samples a&d uri&e specime&s for laboratory tests, i&cludi&" 7b level, 730, uri&alysis,
a&d electrolyte levels#
+rovide small freKue&t meals to mai&tai& adeKuate &utritio&#
(ai&tai& !#@# fluid replaceme&t a&d 0+* to reduce fluid deficit a&d p7 imbala&ce#
+rovide emBotio&al support to help the patie&t cope with her co&ditio&#
"eachin* "opics
,si&" salt o& foods to replace sodium lost by vomiti&"#
8rom< Spri&"house, pa"es 4=3-4=4
I;. INTRAARTUM CARE
Int%a$a%t'm $e%io& e1te&ds from the be"i&&i&" of co&tractio&s that cause cervical dilatio& to the first 1-4
hours after delivery of the &ewbor& a&d place&ta#
Int%a$a%t'm ca%e refers to the medical a&d &ursi&" care "ive& to a pre"&a&t woma& a&d her family duri&"
labor a&d delivery#
5abor versus 5abor
1# 5abor< 3oordi&ated seKue&ce of i&volu&tary uteri&e co&tractio&s or a result i& the effaceme&t a&d dilatio&
of the cervi1, followed by e1pulsio& of the products of co&ceptio&#
2# ?elivery< 6ctual eve&t of birth
A. )acto%s Affectin" Labo%
)ACT-RS A))ECTING LA1-R
+6SS64.%6F +6SS.*4./ +)%./S +563.*065
8630)/S
+SF37.
S
4y&ecoid
243
t
r
e
s
s
f
a
c
t
o
r
s
6
b
r
u
p
t
i
o
p
l
a
c
e
&
t
a
+
7
6
S
.
S
5
e
a
d
i
&
"

t
o

h
y
p
o
t
o
6&thropoid
6&droid
+latypelloid
244
&
i
a
+
l
a
c
e
&
t
a
p
r
e
v
i
a
+
l
a
c
e
&
t
a
a
c
r
e
t
a
+
l
a
c
e
&
t
a
m
e
d
i
a
P
P
P
245
6
8
?
!
!
8etal bo&es
I ASSAGE?A=
-refers to the adeKuacy of the pelvis a&d birth ca&al i& allowi&" the fetal desce&t9 factors i&clude<
6# 0ype of pelvis
-# structure of the pelvis :true versus false pelvis;
3# pelvic i&let diameters
?# ability of the uteri&e se"me&t & va"i&al ca&al to diste&d, the cervi1 to dilate
A))ECTED 1= THE )-LL-?ING )ACT-RSA
A . Ty$es of St%'ct'%e

PartsA ischium, iluim, coccy1#
Hoints" Sacroiliac, Sacrococcy"eal, symphysis pubis :all softe& duri&" pre"&a&cy;

#lassi%ications or 21,es o% Pelvis"
a# Gynecoi&A *ormal 8emale +elvis< /ou&ded )val#
8&S2 (AV&ABLE (& S!##ESS(!L LAB& B B)2*.
b# An&%oi&A *ormal (ale +elvis< 8u&&el Shape
c# Ant!%o$oi&A oval
d# laty$elloi&A flatte&ed, tra&sverse oval
e#
B. St%'ct'%e of t!e el(is :<it! $el(ic inlet & o'tlet &iamete%s*
246
)ALSE EL;IS
6bove the li&ea termi&alis, across the top of symphysis pubis# !t supports the e&lar"e uterus i& the
abdomi&al cavity
O Shallow upper basi& of the pelvis
O Supports the e&lar"i&" uterus but &ot importa&t obstetrically
LINEA TERMINALIS
O +la&e dividi&" upper or false pelvis from lower or true pelvis
TRUE EL;IS
5ies below the li&ea termi&alis, the bo&y pelvis throu"h which the baby pass
%idest diameter :tra&sverse;
*arrowest diameter :a&terior > posterior;
O 3o&sists of the pelvic i&let, pelvic cavity, a&d pelvic outlet#
O -o&y ca&al throu"h which the i&fa&t pass#
O (easureme&ts of true pelvis i&flue&ce the co&duct a&d pro"ress of labor a&d delivery#
MIDLANE
+elvic cavity

-UTLET
?i&est &iamete%A 6&terior posterior diameter :reKuires the i&ter&al /elatio&ship of fetal head for e&try;
Na%%o<est &iamete%< 0ra&sverse !&tertuberous ?iameter :facilitates delivery i& )ccipital 6&terior +osterior;
1#. el(ic meas'%ements
a. T%'e conL'"ate o% conL'"ate (e%a
- measured from upper mar"i& of symphysis pubis to sacral promo&tory9 should be at least 11 cm#
- may be obtai&ed by 1-ray or ,/S
b. T'be%9isc!ial &iamete%/ Inte%t'be%o's &iamete%
9 (easures the outlet betwee& the i&&er borders of ischial tuberosities, should be at least =-A cm#
- estimated o& pelvic e1am
c. -bstet%ical ConL'"ate
- ?ista&ce betwee& the i&&er surfaces of the symphysis pubis a&d sacral promo&tory
24$
II. ASSENGER :T!e )et's*
Refe%s to t!e fet's an& its ability to mo(e t!%o'"! t!e $assa"e<ay.
A))ECTED 1= THE )-LL-?ING )ACT-RS<
a# Attit'&e
1# 0he relatio&ship of the fetal body parts to o&e a&other or, a&other word is fetal posture
2# Nor+al intrauterine attitude is %le$ion, i& which the fetal bac' is rou&ded, the head is forward o& the
chest, a&d the arms a&d le"s are folded i& a"ai&st the body
- Lie
- /elatio&ship of the spi&e of the fetus to the spi&e of the mother
2ransverse lie is an indication for cesarean deli#ery. Se#eral maternal and fetal conditions ma8e cesarean
deliver1 necessary ."he commonly accepted indications include complete placenta pre#ia3 trans#erse lie at
term3 cephalopel#ic disproportion3 abruptio placentae3 acti#e *enital herpes3 umbilical cord prolapse3 failure to
pro*ress in labor3 pro#en fetal distress3 beni*n and mali*nant tumors that bloc8 the birth canal3 and cer#ical
cercla*e. Other reasons for a cesarean deli#ery are more contra#ersial3 such as breech presentation3 pre#ious
cesarean birth3 ma?or con*enital anomalies3 and se#ere isoimmuni2ation. ")ins can sometimes be deli#ered
#a*inally3 especially )hen the lo)ermost t)in is in a #erte@ presentation.
1### Lon"it'&inal o% (e%tical
a# 8etal spi&e is parallel to the motherLs spi&e
b# 8etus is either cephalic or breech prese&tatio&
2### T%ans(e%se o% !o%i3ontal
a# 8etal spi&e is at a ri"ht a&"le, or perpe&dicular, to the motherLs spi&e
b# +rese&ti&" part is the shoulder
c# ?elivery by cesarea& sectio&
3... -bliE'e
a# 8etal spi&e is at a sli"ht a&"le from a true hori2o&tal lie
b# ?elivery is by cesarea& sectio& if u&correctable
3 %esentation
9 the relatio&ship of a particular refere&ce poi&t of the prese&ti&" part a&d the mater&al pelvis described with a series
of 3 letters or presentation refers to the part of the fetus at the cer#ical os
+rese&ti&" part< +ortio& of the fetus that e&ters the pelvis first
1# 3ephalic
a# 0he most commo& prese&tatio&
b# 8etal head prese&ts first
2 -reech
a# -uttoc's prese&t first
b# ?elivery by cesarea& sectio& may be reKuired, althou"h it is ofte& possible to deliver va"i&ally
3 Shoulders
a# 8etus is i& a tra&sverse lie, or the arm, bac', abdome&, or side could prese&t
b# !f the fetus does &ot spo&ta&eously rotate or if it is &ot possible to tur& the fetus ma&ually, a cesarea&
sectio& may be performed
*)0.< "he nurse )ould auscultate abo#e the umbilicus if the fetus is in breech presentation has the bac8
abo#e or at the umbilical area. %etal heart tones are ausculated best in the left lo)er abdomen )hen the
fetus is in a left occipitoanterior position. %or the heart tones to be located belo) the umbilicus3 the fetus
24=
)ould be in a cephalic position. %etal heart tones are heard best in the ri*ht lateral abdomen )hen the fetus
is in a ri*ht occipitoposterior position.
?# osition
/elatio&ship of assi"&ed area of the prese&ti&" part or la&dmar' to the mater&al pelvis or the relationship of the
fetusJs presentin* part to the motherJs pel#is
LE--LDOS MANEU;ERS
It is a systematic )ay to e#aluate the presentation3 position and attitude of the fetusF the location of the best
place to auscultate the fetal heart soundsF and the en*a*ement status of the presentin* part. 2he1 don=t
accuratel1 deter+ine ho. large the %etus is/ .hich is 3est deter+ined 31 ultrasound.
+reparatio&
1# 6s' the mother to empty the bladder
2# %arm ha&ds a&d apply them to the abdome& with firm a&d "e&tle pressure
+/)3.?,/.
2he %irst +aneuver deter+ines .hat %etal ,art is in the %undal ,ortion o% the uterus. )n this case/ the so%t/
%ir+ +ass indicated the %etal 3uttoc7s are in the %undus/ re%lecting a verte$ ,resentation. 2he second
+aneuver docu+ents the location o% the %etal 3ac7. 2he side o% the uterus .here the 3ac7 is located is
s+ooth and conve$ to the touch/ and the o,,osite side has areas o% indentation. 2he third +aneuver
con%ir+s that .as .hat ,al,ated in the %undus is correct and also deter+ines .hether the ,resenting ,art is
engaged. )n this case/ the hard/ round/ +ova3le o30ect in the ,u3ic area is the %etal head. 2he %ourth
+aneuver deter+ines id the %etal head is %le$ed or e$tended.
(etal Position
/)6< /i"ht occiput a&terior
5)6< 5eft occiput a&terior (the 3est %etal ,osition:
/)+< /i"ht occiput posterior
/(6< /i"ht me&tum a&terior
/(+< /i"ht me&tum a&terior
5)+< 5eft occiput posterior
5(6< 5eft me&tum a&terior
/)0< /i"ht occiput tra&sverse
5)0< 5eft occiput tra&sverse
/(+< /i"ht me&tum posterior
5S6< 5eft sacrum a&terior
5S+< 5eft sacrum posterior
Se#ere bac8 pain durin* labor maybe related to a fetus in an OCCIPI"O. POS"$IO POSI"ION. "his means
that the fetal head presses a*ainst the client/s sacrum3 )hich causes mar8ed discomfort durin* contractions.
epositionin* the client and pro#idin* sacral bac8 rubs may help alle#iate the discomfort. "rans#erse3 obli4ue
and occiput positions do not cause pressure on the sacrum.
0. )etal Lie 9 refers to the relationship of the fetal lon* a@is to that of the motherJs lon* a@is.
a# CEHALIC P verte1, face, brow
b# 1REECH > fra&', footli&", complete
c# SH-ULDER > tra&sverse lie
N&2E1 Adolescent clients maturation are usually not yet complete3 therefore they are #ery common for
cephalopel#ic disproportion.
24A
N&2EA 5ie :spi&e to spi&e; may be lo&"itudi&al :parallel;, tra&sverse :ri"ht a&"les;, obliKue :sli"ht a&"le off
true tra&sverse lie;#
)ETAL RE)ERENCE -INT :RESENTING ART*
6# )33!+,0 :);
-# S63/,( :S;
3# S36+,56 :Sc;
?# (.*0,( :(;
MATERNAL RE)ERENCE -INT
1# S!?. )8 (60./*65 +.5@!S
6# 5eft :5;
-# /i"ht :/;
3# 0ra&sverse :0;
2# +6/0 )8 07. (60./*65 +.5@!S
6# 6&terior :6;
-# +osterior :+;
1%eec! RESENTATI-NS
25B
)RANG 1REECH )ULL / C-MLETE
1REECH RESENTATI-N
SH-ULDER 1REECH
251
)--TLING RESENTATI-N
III. -?ER
9 /efers to the freKue&cy, duratio&, a&d stre&"th of uteri&e co&tractio&s to cause complete cervical effaceme&t
a&d dilatio&#
0he forces acti&" to e1pel the fetus
1# EffacementA Shorte&i&" a&d thi&&i&" of the cervi1 duri&" the first sta"e of labor
2# Dilation< .&lar"eme&t of cervical os a&d cervical ca&al duri&" first sta"e
LA1-R C-NTRACTI-NS
THREE HASES -) C-NTRACTI-N
1# INCREMENT- steep cresce&t slope from be"i&&i&" of the co&tractio& u&til its pea'#
2# ACME/EAG > stro&"est i&te&sity#
3# DECREMENT > dimi&ishi&" i&te&sity#
CHARACTERISTICS -) C-NTRACTI-NS
)REFUENC= > be"i&&i&" of o&e co&tractio& to be"i&&i&" of o&e co&tractio&# 5ess tha& 2 mi&utes should be
reported#
DURATI-N > be"i&&i&" of o&e co&tractio& u&til its completio&#
(ore tha& AB seco&ds should be reported because of uteri&e rupture or fetal distress#
INTENSIT= > the stre&"th of co&tractio& at its pea' may be mild, moderate or stro&"#
I;. LACENTAL )ACT-RS
- /efers to the site of place&tal i&sertio&#
;. S=CHE
- /efers to the clie&tEs psycholo"ical state, available support systems, preparatio& for birth, e1perie&ces, a&d
copi&" strate"ies#
1. LA1-R
#. Si"ns of im$en&in" labo%
,. Com$a%ison of T%'e Labo% f%om )alse Labo%
3. Sta"es of labo%
3. a. station of t!e $%esentin" $a%t
.. N'%sin" Inte%(entions &'%in" labo% & &eli(e%y
0. Assessin" t!e )etal Hea%t Rate
SIGNS -) IMENDING LA1-R
5i"hte&i&" -ra1to&Es-7ic's co&tractio& 4astroi&testi&al upset -urst of e&er"y -lood show
252
J1 sign o% la3or u,tured 3ag o% .ater
#. REM-NIT-R= SIGNS -) LA1-R
1# 5!470.*!*4
- ?esce&t of the fetus a&d uterus i&to pelvic cavity before labor o&set#
-)ccurs 2-3 wee's earlier i& primipara#
- !& multipara, may &ot occur u&til labor be"i&s#
2# 3./@!365 376*4.S
a# E))ACEMENT
- +ro"ressive softe&i&" Sripe&i&"T a&d thi&&i&" of the cervi1#
- S-5))?F S7)%T :e1pulsio& of mucous plu";
b# DILATI-N
- )pe&i&" of cervical os duri&" labor#
3# /e"ular -ra1to& 7ic'sE co&tractio&s#
4# /upture of am&iotic membra&es#
5# *.S05!*4 -.76@!)/S
6# %ei"ht loss of about 1-3 lbs 2-3 days before labor o&set#
,. C-MARIS-N -) TRUE AND )ALSE LA1-R
376/630./!S0!3S 0/,. 865S.
3o&tractio&s /e"ular
-ecome more freKue&t
4radual i&crease i&
duratio& a&d i&te&sity /
pro"ressive freKue&cy &
i&te&sity
!rre"ular
,&cha&"ed
,&cha&"ed or decrease
i& freKue&cy a&d
i&te&sity
?iscomfort -e"i&s at lower bac' a&d
radiates arou&d abdome&
+rimarily o& the lower
abdome& & "roi&

5es
.ffects of wal'i&"

*o
3ervical cha&"es
+ai& does&Et
disappear+ai&
disappearsShow+ro"ressi
ve dilatio& a&d effaceme&t
+rese&t *ot prese&t
253
376/630./!S0!3S 0/,. 865S.
3o&tractio&s /e"ular
-ecome more freKue&t
4radual i&crease i&
duratio& a&d i&te&sity /
pro"ressive freKue&cy &
i&te&sity
!rre"ular
,&cha&"ed
,&cha&"ed or decrease
i& freKue&cy a&d
i&te&sity
?iscomfort -e"i&s at lower bac' a&d
radiates arou&d abdome&
+rimarily o& the lower
abdome& & "roi&

5es
.ffects of wal'i&"
?uri&" Sedatio&?uri&" sleep 3o&tractio&s does&Et stop 3o&tractio&s stops
3. STAGES -) LA1-R
1
S0
S064. 2
*?
S064. 3
/?
S064. 4
07
S064.
1
st
4
h
o
u
r
s
p
o
s
t
p
a
r
t
u
3o&tractio& to dilatio&
%e$a%ato%y &i(isionA
1; 5ate&t phase B-3 cm
?uratio&< 3B > 45 seco&ds
*)0.< Pushin* durin* the first sta*e of labor )hen the ur*e is felt but the cer#i@ is not yet fully dilated may
produce cer#ical s)ellin* and ma8es labor more difficult. "he client should be encoura*ed to PAN" (LO6 or
(LO6.(LO6 pattern of breathin* to help o#ercome the ur*e to push.
254
m
8
u
l
l
c
e
r
v
i
c
a
l
d
i
l
a
t
a
t
i
o
&
t
o
d
e
l
i
v
e
r
y
?
e
l
i
v
e
r
y
t
o
p
l
a
c
e
&
t
a
l
e
1
p
u
255
l
s
i
o
&
"
h
e

p
r
e
c
a
u
t
i
o
n
s

y
o
u

s
h
o
u
l
d

t
a
8
e

)
h
e
n

a

p
o
s
t
p
a
r
t
u
m

256
c
l
i
e
n
t
s
t
a
r
t
s

a
m
b
u
l
a
t
i
n
*

a
r
e

t
h
e

f
a
l
l
p
r
e
c
a
u
t
i
o
n

a
n
d

c
l
o
s
e
25$

m
o
n
i
t
o
r
i
n
*

s
h
o
u
l
d

b
e

d
o
n
e

d
u
e

t
o

t
h
e

r
i
s
8

o
f
s
y
n
c
o
p
y
3
e
s
p
25=
e
c
i
a
l
l
y

t
h
e

f
i
r
s
t
f
e
)

t
i
m
e
s

o
u
t
o
f
b
e
d
.
"
h
e
n
u
r
s
e
s
h
o
u
l
d

8
n
o
)
25A

i
f
t
h
e
p
l
a
c
e
n
t
a

i
s
*
o
i
n
*

t
o

b
e
d
e
l
i
#
e
r
e
d
3
i
s
t
o

)
a
t
c
h

f
o
r
c
o
r
d
26B

l
e
n
*
t
h
e
n
i
n
*
3
a

s
l
i
*
h
t
*
u
s
h

o
f
d
a
r
8
e
n
e
d

b
l
o
o
d

o
r
a

c
h
a
n
*
e
i
n

261
f
u
n
d
a
l
s
h
a
p
e
.
2; 6ctive phase 4-$ cm
D'%ationA .0948 secon&s
3; 0ra&sitio&al phase =-1B cm
?uratio& < 6B-AB seco&ds
)IRST STAGE -) LA1-R
:-NSET -) REGULAR C-NTRACTI-NS T- )ULL CER;ICAL DILATI-N
2ANS)2)&N P*ASE
0!(.< +/!(!+6/6 :1hour;
(,50!+6/6 :1B > 15 mi&utes;
3./@!Q<
.8863.(.*0 - 1BBC
?!560!)* - =-1B cm
3)*0/630!)*S
8/.X,.*3F - 2-3 mi&utes
?,/60!)* - 6B-AB seco&ds
(6*!8.S060!)*S<
3lie&t may be irritable a&d pa&ic'y9 (ay lose co&trol9 6m&esic betwee& co&tractio&s9 +erspiri&",
&auseous a&d vomiti&" commo&9 0rembli&" of le"s9 +ressure o& bladder a&d rectum9 -ac'ache9 !&creased
show9 3ircumoral pallor
NO"$1 If the client is in acti#e labor and there is no chan*e in dilation after D hours3 the nurse should suspect
cephalopel#ic disproportion. "he client is not e@periencin* a prolon*ed latent phase 0B.C cm73 prolon*ed
transitional phase 0pushin*73 and contraction pattern.
NO"$1&a*inal $@amination
"o determine if the client is fully dilated3 the nurse performs a #a*inal e@amination. "o assess the
suture most readily felt3 the nurse )ould determine the position of the cranial suture termed.SA-I"ALL
SU"U$.
262
STATI-N
/efer to the level of prese&ti&" part of fetus i& relatio& to ima"i&ary li&e betwee& ischial spi&es :2ero
statio&; i& mid pelvis of mother#
- 0he measureme&t of the pro"ress of desce&t i& ce&timeters above or below the midpla&e from the
prese&ti&" part to the ischial spi&e
Min's stationA abo(e isc!ial s$ine
90 to P# in&icates a $%esentin" $a%t abo(e 3e%o station :93)L-ATINGD 9# DIING*
Station 8A at isc!ial s$ine
; +eans ENGAGE8EN2
l's stationA belo< isc!ial s$ine
Q # T- Q 0 in&icates a $%esentin" $a%t belo< 3e%o station
Q3 #&4N)NG
NMy baby is comin*O3 the T9 nursin* inter#ention is to loo8 for perineal bul*in* 0cro)nin*7. If the
perineum is bul*in*3 the patient should be coached to pant )ith her contractions so that she doesn/t
push. %etal heart rate is focus on the labor process or potential fetal cord compression and meconium
stained complications
SEC-ND STAGE -) LA1-R
:C-MLETE CER;ICAL DILATI-N T- 1IRTH -) NE?1-RN*
0!(.
+/!(!+6/6 :3B-5B mi&utes ;
(,50!+6/6 :2B mi&utes;
3)*0/630!)*S
8/.X,.*3F - 2-3 mi&utes
?,/60!)* - 6B-AB seco&ds
!*0.*S!0F
@./F 76/?< 1BB mm 7"
(6*!8.S060!)*S<
?ecrease i& pai& from tra&sitio&al level9 i&creased bloody show9 .1cited ea"er a&d i& co&trol#
263
THIRD STAGE -) LA1-R
:DELI;ER= -) NE?1-RN T- DELI;ER= -) LACENTA*
0!(.< 5-3B mi&utes
3)*0/630!)*S
Stro&" a&d well-co&tracted uterus cha&"i&" to "lobular shape
(6*!8.S060!)*S<
Increased *ush of blood
Uterus becomin* *lobular )ith fundus risin* in the abdomen
Apparent len*thenin* of cord

)-URTH STAGE -) LA1-R
:DELI;ER= -) LACENTA T- H-ME-STASIS*
0!(.
,sually defi&ed as the first hour postpartum# 0his sta"e lasts from 1-4 hours after birth#
,0./,S
0he uterus co&tracts i& the midli&e of the abdome& with the fu&dus midway betwee& the umbilicus a&d
symphysis pubis#
(6*!8.S060!)*S<
5ochia rubra
.1ploratio& of &ewbor&
+are&t-i&fa&t bo&di&" be"i&s
*ewbor& alert a&d respo&sive
8irst period of reactivity
NURSING INTER;ENTI-NS DURING LA1-R AND DELI;ER=
O ?uri&" labor, mo&itor 87/#
O +rovide patie&t comfort#
O 6dmi&ister a&al"esics as i&dicated#
O +repare for delivery#
O !mmediate &ewbor& care at delivery#
- .stablish airway#
- )bserve 6p"ar score at 1 a&d 5 mi&utes i&terval#
- 3lamp umbilical cord#
- (ai&tai& warmth#
- 6ssess the &ewbor&Es "estatio&al a"e#
- 6dmi&ister prophylactic eye drops a&d vitami& J#
- +lace ide&tificatio& ba&d o& baby a&d mother#
NURSING CARE DURING LA1-R
264
6
S
S
.
S
S
(
.
*
0
*
,
/
S
!
*
4

3
)
*
S
!
?
.
/
6
0
!
)
*
S064.
1+76S
.
5ate&t )&set of labor u&til cervical dilatatio&
of 4 cm#
(o&itor freKue&cy, i&te&sity, a&d
patter&s of uteri&e co&tractio&s
(o&itor fetal status duri&" labor by
mo&itori&" fetal heart rate
6ssess bloody show :pi&' or blood
strea'ed mucus;, peri&eal bul"i&",
membra&e status
+eriodic va"i&al e1ams
(o&itor vital si"&s
6ssess clie&tEs ability to cope with
co&tractio&s
+rovide emotio&al support
265
6
S
S
.
S
S
(
.
*
0
*
,
/
S
!
*
4

3
)
*
S
!
?
.
/
6
0
!
)
*
S064.
1+76S
.
5ate&t )&set of labor u&til cervical dilatatio&
of 4 cm#
(o&itor freKue&cy, i&te&sity, a&d
patter&s of uteri&e co&tractio&s
(o&itor fetal status duri&" labor by
mo&itori&" fetal heart rate
6ssess bloody show :pi&' or blood
strea'ed mucus;, peri&eal bul"i&",
membra&e status
+eriodic va"i&al e1ams
(o&itor vital si"&s
6ssess clie&tEs ability to cope with
co&tractio&s
+rovide emotio&al support

+re

!m
2
266
6
S
S
.
S
S
(
.
*
0
*
,
/
S
!
*
4

3
)
*
S
!
?
.
/
6
0
!
)
*
S064.
1+76S
.
5ate&t )&set of labor u&til cervical dilatatio&
of 4 cm#
(o&itor freKue&cy, i&te&sity, a&d
patter&s of uteri&e co&tractio&s
(o&itor fetal status duri&" labor by
mo&itori&" fetal heart rate
6ssess bloody show :pi&' or blood
strea'ed mucus;, peri&eal bul"i&",
membra&e status
+eriodic va"i&al e1ams
(o&itor vital si"&s
6ssess clie&tEs ability to cope with
co&tractio&s
+rovide emotio&al support
3 8rom delivery of the fetus to delivery
of the place&ta, usual withi& 5-2B
mi&s# )f delivery
6ssess umbilical cord for 3 vessels :2
arteries, 1 vei&;
6ssess place&ta for i&tact&ess
0he fu&dus should be midli&e at or two
26$
6
S
S
.
S
S
(
.
*
0
*
,
/
S
!
*
4

3
)
*
S
!
?
.
/
6
0
!
)
*
S064.
1+76S
.
5ate&t )&set of labor u&til cervical dilatatio&
of 4 cm#
(o&itor freKue&cy, i&te&sity, a&d
patter&s of uteri&e co&tractio&s
(o&itor fetal status duri&" labor by
mo&itori&" fetal heart rate
6ssess bloody show :pi&' or blood
strea'ed mucus;, peri&eal bul"i&",
membra&e status
+eriodic va"i&al e1ams
(o&itor vital si"&s
6ssess clie&tEs ability to cope with
co&tractio&s
+rovide emotio&al support
cm# -elow the umbilicus
(y the Dnd postpartum day3 the fundus should
be firm and t)o fin*erbreadths belo) the
umbilicus. "he fundus should be at the le#el of
26=
6
S
S
.
S
S
(
.
*
0
*
,
/
S
!
*
4

3
)
*
S
!
?
.
/
6
0
!
)
*
S064.
1+76S
.
5ate&t )&set of labor u&til cervical dilatatio&
of 4 cm#
(o&itor freKue&cy, i&te&sity, a&d
patter&s of uteri&e co&tractio&s
(o&itor fetal status duri&" labor by
mo&itori&" fetal heart rate
6ssess bloody show :pi&' or blood
strea'ed mucus;, peri&eal bul"i&",
membra&e status
+eriodic va"i&al e1ams
(o&itor vital si"&s
6ssess clie&tEs ability to cope with
co&tractio&s
+rovide emotio&al support
the umbilicus on the day of deli#ery and falls
belo) the umbilicus by appro@imately one
fin*erbreadth 09 cm7 per day3 until it has
contracted into the pel#is by the Uth or 9Bth
day. "he fundus should be firm3 not soft. A soft
26A
6
S
S
.
S
S
(
.
*
0
*
,
/
S
!
*
4

3
)
*
S
!
?
.
/
6
0
!
)
*
S064.
1+76S
.
5ate&t )&set of labor u&til cervical dilatatio&
of 4 cm#
(o&itor freKue&cy, i&te&sity, a&d
patter&s of uteri&e co&tractio&s
(o&itor fetal status duri&" labor by
mo&itori&" fetal heart rate
6ssess bloody show :pi&' or blood
strea'ed mucus;, peri&eal bul"i&",
membra&e status
+eriodic va"i&al e1ams
(o&itor vital si"&s
6ssess clie&tEs ability to cope with
co&tractio&s
+rovide emotio&al support
or bo**y fundus indicates that the uterus isnJt
contractin* properly. "he fundus should be
palpated in the midline of the abdomenF if the
)oman has a full bladder3 ho)e#er3 the fundus
may be de#iated to the ri*ht or left.
2$B
6
S
S
.
S
S
(
.
*
0
*
,
/
S
!
*
4

3
)
*
S
!
?
.
/
6
0
!
)
*
S064.
1+76S
.
5ate&t )&set of labor u&til cervical dilatatio&
of 4 cm#
(o&itor freKue&cy, i&te&sity, a&d
patter&s of uteri&e co&tractio&s
(o&itor fetal status duri&" labor by
mo&itori&" fetal heart rate
6ssess bloody show :pi&' or blood
strea'ed mucus;, peri&eal bul"i&",
membra&e status
+eriodic va"i&al e1ams
(o&itor vital si"&s
6ssess clie&tEs ability to cope with
co&tractio&s
+rovide emotio&al support
0he fu&dus should desce&d
appro1imately 1-2 cm every 24 hours
N-TEA T!e f'n&'s s!o'l& not be
2$1
6
S
S
.
S
S
(
.
*
0
*
,
/
S
!
*
4

3
)
*
S
!
?
.
/
6
0
!
)
*
S064.
1+76S
.
5ate&t )&set of labor u&til cervical dilatatio&
of 4 cm#
(o&itor freKue&cy, i&te&sity, a&d
patter&s of uteri&e co&tractio&s
(o&itor fetal status duri&" labor by
mo&itori&" fetal heart rate
6ssess bloody show :pi&' or blood
strea'ed mucus;, peri&eal bul"i&",
membra&e status
+eriodic va"i&al e1ams
(o&itor vital si"&s
6ssess clie&tEs ability to cope with
co&tractio&s
+rovide emotio&al support
massa"e& 'nless it is %ela+e&. Constant
massa"in" <o'l& ti%e t!e 'te%ine m'scleD
cont%ib'tin" to !emo%%!a"e
2$2
6
S
S
.
S
S
(
.
*
0
*
,
/
S
!
*
4

3
)
*
S
!
?
.
/
6
0
!
)
*
S064.
1+76S
.
5ate&t )&set of labor u&til cervical dilatatio&
of 4 cm#
(o&itor freKue&cy, i&te&sity, a&d
patter&s of uteri&e co&tractio&s
(o&itor fetal status duri&" labor by
mo&itori&" fetal heart rate
6ssess bloody show :pi&' or blood
strea'ed mucus;, peri&eal bul"i&",
membra&e status
+eriodic va"i&al e1ams
(o&itor vital si"&s
6ssess clie&tEs ability to cope with
co&tractio&s
+rovide emotio&al support
2$3
6
S
S
.
S
S
(
.
*
0
*
,
/
S
!
*
4

3
)
*
S
!
?
.
/
6
0
!
)
*
S064.
1+76S
.
5ate&t )&set of labor u&til cervical dilatatio&
of 4 cm#
(o&itor freKue&cy, i&te&sity, a&d
patter&s of uteri&e co&tractio&s
(o&itor fetal status duri&" labor by
mo&itori&" fetal heart rate
6ssess bloody show :pi&' or blood
strea'ed mucus;, peri&eal bul"i&",
membra&e status
+eriodic va"i&al e1ams
(o&itor vital si"&s
6ssess clie&tEs ability to cope with
co&tractio&s
+rovide emotio&al support

+ro

6s
4
2$4
ASSESSING THE )ETAL HEART RATE
;. )ETAL M-NIT-RING
6# ?escriptio&
2# (o&itors uteri&e activity, assesses freKue&cy, duratio&, a&d i&te&sity of co&tractio&s, assesses 87/ i&
relatio& to mater&al co&tractio&s# !t is the baseli&e 87/ measured betwee& co&tractio&s9 the &ormal 87/ is
12B to 16B beats per mi&ute
-# .1ter&al fetal mo&itori&"
1# *o&i&vasive a&d performed by the use of a tocotra&sducer or ?oppler ultraso&ic tra&sducer
2# +erform 5eopoldLs ma&euvers to determi&e o& which side the fetal bac' is located, a&d place the
ultrasou&d tra&sducer over this area :faste& with a belt;
3# +lace the tocotra&sducer over the fu&dus of the uterus where co&tractio&s feel the stro&"est :faste& with a
belt;
4# 6llow the clie&t to assume a comfortable positio&, avoidi&" ve&a cava compressio&
*)0.< 2he e$ternal %etal +onitor records the contractile ,attern and the %etal heart rate res,onse to the
contractions. 2he e$ternal +onitor doesn=t accuratel1 record intensit1 o% the contractions/ and it doesn=t
accuratel1 record %etal heart rate varia3ilit1.
3# !&ter&al fetal mo&itori&"
1# !&vasive a&d reKuires rupturi&" of the membra&es a&d attachi&" a& electrode to the prese&ti&" part of the
fetus#
*)0.< "he patient )ith the fetus in a #erte@ position and meconium.stained fluid )ould ha#e the hi*hest
priority of bein* monitored )ith internal fetal monitorin*. "he patient )ith the meconium.stained amniotic
fluid is at hi*h ris8 for fetal distress. Internal fetal monitorin* re4uires that the patient ha#e ruptured
membranes and be dilated at least 9 cm and that the fetal presentin* part is reachable. In many institutions3
fetal monitorin* is used routinely on all patients. %etal monitorin* is most useful in situations in )hich a hi*h
probability e@ists of maternal contractile problems or fetal distress. %etal monitorin* pro#ides an almost
continuous recordin* of labor e#ents.
*)0.< Internal $%M can be applied only after the clientJs membranes ha#e ruptured3 )hen the fetus is at least
at the H9 station3 and )hen the cer#i@ is dilated at least D cm. Althou*h the client may recei#e anesthesia3 it isnJt
re4uired before application of an internal $%M de#ice.
2# (other must be dilated 2 to 3 cm to perform i&ter&al mo&itori&"
*)0.< "o pre#ent e@posure to human immunodeficiency #irus 0+I&73 in#asi#e procedures3 such as fetal
scalp samplin*3 and #acuum e@traction3 shouldn/t be done unless absolutely indicated. $ach of those
procedures either causes or has the potential to use a brea8 in the fetal s8in3 thereby increasin* the ris8
of transmission of +I& to the fetus. Non.stress test and ultrasono*raphy aren/t nonin#asi#e procedures
and don/t increase the ris8 of transmission of +I& to the fetus. Sterile #a*inal e@aminations are
necessary to monitor the patient/s pro*ress durin* labor and3 if performed appropriately3 shouldn/t pose
additional ris8 of +I& transmission to the fetus
2$5
8.065 7.6/0 /60. +600./* !*?!360!@. )8U## !*0./@.*0!)*
?
e
p
e
&
d
e
&
t
u
p
o
&
c
a
u
s
e
0achycardia :P16B b#p#m#;
-radycardia :Y12B
b#p#m#;(ater&al or fetal i&fectio&
8etal hypo1ia :a& omi&ous
si"&;
8etal hypo1ia or stress
(ater&al hypote&sio& after epidural
i&itiatio&
+lace clie&t o& left side
!&crease fluids :to
cou&teract hypote&sio&;
Stop o1ytoci& :+itoci&; if
i& use
.arly deceleratio&
:?eceleratio& be"i&s a&d e&ds
with uteri&e co&tractio&;
7ead compressio& :&ot omi&ous;
@a"i&al stimulatio&
*ot reKuired
5ate deceleratio&
:7/ decreases after pea' of
co&tractio& a&d recovers after
co&tractio& e&ds;
8etal stress a&d hypo1ia
?eficie&t place&tal perfusio&
Supi&e positio&
(ater&al hypote&sio&
,teri&e hyperstimulatio&
3ha&"e mater&al
positio&
3orrect hypote&sio&
!&crease !#@# fluid rate
as ordered
?isco&ti&ue o1ytoci&
:+itoci&;
N&2E"
Nursing interventions %or
utero,lacental
insu%%icienc1 include
re,ositioning to side-l1ing
2$6
,osition' ad+inistering
o$1gen 31 tight %ace +as7
at 1; to 12 L@+inute'
increasing ).V. %luids'
discontinuing the
o$1tocin/ i% it=s 3eing
in%used' assessing
+aternal vital signs %or
evidence o% h1,otension'
and evaluating the %etal
res,onse to the
interventions.
6dmi&ister o1y"e& as
ordered
.
@ariable deceleratio&
:0ra&sie&t decrease i& 7/
a&ytime duri&" co&tractio&;
3ord compressio&
&ariable decelerations in fetal heart
rate are an ominous si*n3 indicatin*
compression of the umbilical cord.
Chan*in* the clientJs position from
supine to side.lyin* may immediately
correct the problem. An emer*ency
cesarean section is necessary only if
other measures3 such as chan*in*
position and amnioinfusion )ith sterile
saline3 pro#e unsuccessful.
Administerin* o@y*en may be helpful3
but the priority is to chan*e the
)omanJs position and relie#e cord
compression.
"he first action )hen uterine cord
occurs is to relie#e pressure on the
cord by chan*in* the patient/s position.
Once you ha#e chec8ed the cord3 the
rest of the body should be deli#ered
)ith an application of *entle traction
on the anterior shoulder3 ad#isin* the
patient not to push.
If the cord is pulled on before the
placenta has separated there )ill be a
uterine in#ersion or retained placental
fra*ments.
7ypo1ia or hypercarp&ia
3ha&"e mater&al
positio&
:left lateral positio&;
6dmi&ister )2
2$$
?
e
p
e
&
d
e
&
t
u
p
o
&

c
a
u
s
e
?ecreased variability :smooth baseli&e;
8etal sleep cycle
Mec!anisms of Labo% En"a"ement o% Ca%&inal mo(ements by t!e )et's
Definition< (echa&ism by which the fetus &estles i&to the pelvis# 6 co&ti&uous process from the time of
e&"a"eme&t u&til birth, a&d is assessed by the measureme&t called statio&
Descent
6lso termed li"hte&i&" or droppi&" ?esce&t
0he process that the fetal head u&der"oes as be"i&s its our&ey throu"h the pelvis
)le+ion +rocess of the fetal headLs &oddi&" forward toward the fetal chest
Su3occi,oto3reg+aticA the diameter that prese&ts to the mater&al pelvis duri&" COMPL$"$ %L$'ION#
Inte%nal Rotation
!&ter&al rotatio& of the fetus9 most commo&ly from the occipital tra&sverse positio&, assumed at e&"a"eme&t
i&to the pelvis, to the occipital a&terior positio& while co&ti&uously desce&di&"
E+tension
.&ables the head to emer"e whe& the fetus is i& a cephalic positio&
-e"i&s after the head crow&s !s complete whe& the head passes u&der the pubis a&d occipital, a&d the a&terior
fo&ta&el, brow, face, a&d chi& pass over the sacrum a&d coccy1 are over the peri&eum
/estitutio&
/eali"&me&t of the fetal head with the body after that head emer"es
E+te%nal Rotation
0he shoulders e1ter&ally rotate after the head emer"es a&d restitutio& occurs, so that the shoulders are
a&teroposterior diameter of the pelvis
E+$'lsion
0he delivery baby
2$=
CARDINAL M-;EMENTS -) THE )ETUS

?esce&t 8le1io& !&ter&al /otatio& .1te&sio& .1ter&al /otatio&
.1pulsio&

8i"ure 1= 3ardi&al (oveme&ts or (echa&ism of labor
;II. ANESTHESIA
N-TEA Analgesia ad+inistered during the second stage o% la3or includes continuation o% the lu+3ar
e,idural 3loc7/ ,udendal 3loc7/ and local in%iltration o% the ,erineu+. Narcotic analgesics and ,ericervical
3loc7 are ad+inistered during the active ,hase o% la3or. A s,inal 3loc7 is given during the active ,hase o%
the %irst stage o% la3or. Sedative h1,notics/ i% ad+inistered/ are given .hen the ,atient is in earl1 latent la3or
to encourage rest. A s,inal 3loc7 is given during the active ,hase o% the %irst stage o% la3or.
N&2E" *)0.< "he chief concepts of La+a9e teaching include conditioned responses to stimuli throu*h use of
a focal point. An emotionally satisfyin* e@perience is promoted rather than discoura*in* use of anal*esia and
anesthesia.
6# 5ocal a&esthesia
1# ,sed for bloc'i&" pai& duri&" episiotomy
2# 6dmi&istered ust before the birth of baby
3# *o effect o& the fetus
-# +aracervical bloc'
1# ,sed i& the first sta"e of labor
2# +rovides a rapid bloc' of uteri&e pai&
3# *o effect o& the peri&eal area
4# *o effect o& the ability to bear dow&
5# (ay cause fetal bradycardia
3# +ude&dal bloc'
1# 6dmi&istered ust before the birth of the baby
2# !&ectio& site at pude&dal &erve throu"h a tra&sva"i&al route
3# -loc's peri&eal area for episiotomy
2$A
?.S3.*0 85.Q!)* !*0./*65
/)060!)*
.Q0./*65
/)060!)*
.Q0.*S!)*
.Q+,5S!)*
4# .ffect lasts about 3B mi&utes
5# *o effect o& co&tractio&s or fetus
*)0.< Pudendal (loc8 Anesthesia
"he T9 purpose is to relie#e pain primarily in the perineum and #a*ina. It does not relie#e pain primarily
in the perineum and #a*ina. Pudendal bloc8 is ade4uate for episiotomy and its repair.
"he fetus should be assessed for (A,5CA,IA )hich is a potential complication of pudendal bloc8
anesthesia. ,ecrease mo#ements3 increase #ariability and meconium stained are NO" associated.
Maternal Ad#erse effects are the follo)in*1 hypotonia3 reduced responsi#eness and sei2ures.
?# .pidural bloc'
1# !&ectio& site i& epidural space at 53-54
2# 6dmi&istered after labor is established or ust before a scheduled cesarea& birth
3# /elieves pai& from co&tractio&s a&d &umbs va"i&a a&d peri&eum
4# (ay cause hypote&sio&
5# Does not cause headache 3ecause the dura +ater is not ,enetrated
6# 6ssess mater&al blood pressure
$# (ai&tai& the mother i& side-lyi&" positio& or place a rolled bla&'et be&eath the ri"ht hip to displace the
uterus from the ve&a cava
=# 6dmi&ister !@ fluids as prescribed A# !&crease fluids as prescribed if hypote&sio& occurs
A# 0he maor complicatio& of epidural a&esthesia is mater&al hypote&sio&#
N&2E" "o minimi2e the hypertensi#e effects of epidural anesthesia prior to the procedure ade4uately
hydrate the patient and position the patient side lyin* to the left.
After epidural anesthesia the #ital si*ns should be monitored e#ery 9.D minutes for the first 9E minutes.
2he assess+ent should 3e a high ,riorit1 a%ter a ,atient has received an e,idural is 3lood ,ressure
3ecause an e,idural can cause h1,otension and its 3loc7s the autono+ic nervous s1ste+.
A patient )ho is about to recei#e epidural anesthesia should empty her bladder before the procedure
because an epidural )ill lessen the sensation to #oid so #oidin* no) may decrease the need for
catheteri2ation later.
*)0.<
A co++on adverse e%%ect o% e,idural anesthesia is h1,otension/ .hich .ould cause i+,aired gas
e$change in the %etus. 2o ,revent h1,otension/ the ,atient receives a 3olus o% 6;; to 1/;;; +l o% ).V.
%luid 3e%ore the ,rocedure. 2he ,atient isn=t a%%ected 31 these ,ro3le+s 3ecause she didn=t receive the
e,idural anesthesia.
*)0.< "he patient plans to recei#e an epidural anesthetic for pain relief durin* labor3 it )on/t be
administered until the patient is dilated I to E cm.
.# Spi&al bloc'
1# !&ectio& site i& spi&al subarach&oid space at 53-55
2# 6dmi&istered ust before birth
3# /elieves uteri&e a&d peri&eal pai& a&d &umbs va"i&a, peri&eum, a&d lower e1tremities
4# (ay cause mater&al hypote&sio&
5# (ay cause postpartum headache
6# 0he mother must lie flat = to 12 hours followi&" spi&al i&ectio&
$# +lace a rolled bla&'et u&der the ri"ht hip to displace the uterus from the ve&a cava
=# 6dmi&ister !@ fluids as prescribed
2=B
8# 4e&eral a&esthesia
1# (ay be used for some sur"ical i&terve&tio&s
2# 0he mother is &ot awa'e
3# +rese&ts a da&"er of respiratory depressio& vomiti&"
-1STETRICAL R-CEDURES
6# )1ytoci& !&ductio&
1# 6 deliberate i&itiatio& of uteri&e co&tractio&s this stimulates labor
2# .lective i&ductio& may be accomplished ! o1ytoci& :+itoci&; i&fusio&
3# )btai& baseli&e traci&" of uteri&e co&tractio&s a&d 87/
4# !&crease !@ dosa"e of o1ytoci& as prescribed o&ly after assessi&" co&tractio&s, 87/, a&d mater&al blood
pressure a&d pulse
5# ,o not increase rate of o@ytocin once the desired contraction pattern is obtained 0contraction fre4uency
of D to C minutes and lastin* :B seconds7
6# ,iscontinue o@ytocin as prescribed contraction fre4uency is less than D minutes or duration more than UB
seconds3 or if fetal distress is note
*)0.< )1ytoci& :!&ductio& of 5abor;
(efore the induction of Labor3 the nurse should obtain a baseline measurement of the fetal heart rate. If
the fetal heart rate pattern sho)s fetal distress3 the client is not a candidate or if contractions occur less
than D minutes apart or last lon*er than :B seconds
3# Amniotomy
1# 6rtificial rupture of membra&es :6/)(;9 performe by the physicia& to stimulate labor
2# +erformed if the fetus is at [B[ or [V[ statio&
3# !&creases ris' of prolapsed cord a&d i&fectic
4# (o&itor 87/ before a&d after 6/)(
5# /ecord time of 6/)(, 87/, a&d characteristic of fluid
6# (eco&ium-stai&ed am&iotic fluid may be associated with fetal distress
$# -loody am&iotic fluid may i&dicate abrupt place&tae or fetal trauma
=# 6& u&pleasa&t odor to am&iotic fluid is associated with i&fectio&
A# +olyhydram&ios is associated with mater&al diabetes a&d certai& co&"e&ital disorders
1B# )li"ohydram&ios is associated with i&trauteri&e "rowth retardatio& :!,4/; a&d co&"e&ital
disorders
? #E+te%nal (e%sion
1# .1ter&al ma&ipulatio& of the fetus from a& ab&ormal positio& i&to a &ormal prese&tatio&
2# !&dicated for a& ab&ormal prese&tatio& that e1ists after the 34th wee'
3# (o&itor vital si"&s
4# !f the mother is /h-&e"ative, e&sure that /7 immu&e "lobuli& was "ive& at 2= wee's "estatio&
5# +repare for &o&stress test to evaluate fetal well-bei&"
2=1
6# !@ fluids a&d tocolytic therapy may be admi&istered to rela1 the uterus a&d permit easier ma&ipulatio& of
fetus
$# ,ltrasou&d is used duri&" the procedure to evaluate fetal positio& a&d place&tal placeme&t a&d "uide
directio& to the fetus
=# 6bdomi&al wall is ma&ipulated to direct fetus i&to a cephalic prese&tatio& if possible
A# (o&itor blood pressure to ide&tify ve&a cava compressio&
1B# (o&itor for u&usual pai&
11# 8ollowi&" the procedure
a# +erform &o&stress test to evaluate fetal well-bei&"
b# #(o&itor for uteri&e activity, bleedi&", ruptured membra&es, a&d decreased fetal activity
c# %ith /h-&e"ative clie&ts, perform Jleihauer -et'e test as prescribed to detect the prese&ce a&d
amou&t of fetal blood i& the mater&al circulatio& a&d to ide&tify clie&ts who &eed additio&al /h
immu&e "lobuli&
i# E$isiotomy
M "he purpose of episiotomy is to shorten the D
nd
sta*e of labor3 substitutes a clean sur*ical incision for
a tear and decreases undue stretchin* of perineal muscles. An episiotomy helps pre#ent tearin* of the
rectum but does not necessarily relie#e pressure on the rectum. An episiotomy does not pre#ent perineal
edema3 ensure 4uic8 deli#ery of the placenta or cause enlar*in* the pel#ic inlet.
1# !&cisio& made i&to peri&eum to e&lar"e va"i&al outlet a&d facilitate delivery
2# 3hec' episiotomy site
3# !&stitute measures to relieve pai&
4# +rovide ice pac' duri&" the first 24 hours
5# !&struct the clie&t i& the use of sit2 baths
6# 6pply a&al"esic spray or oi&tme&t as prescribed
$# +rovide peri&eal care, usi&" clea& tech&iKue
=# !&struct the clie&t i& the proper care of the i&cisio&
A# !&struct the clie&t to dry the peri&eal area from fro&t to bac' a&d to blot the area rather tha& wipe it
1B# !&struct the clie&t to shower rather tha& bathe i& a tub
11# 6pply a peripad without touchi&" the i&side surface of the pad
12# /eport a&y bleedi&" or dischar"e to the physicia&
13# "he ad#anta*e of an episiotomy is that it facilitates the deli#ery of the fetus3 it pre#ents tearin* of the
perineum3 and it pre#ents undo stretchin* of the perineal muscles.
8# )o%ce$s &eli(e%y
1# 0wo double-crossed, spoo& li'e articulated blades that are used to assist# i& the delivery of the fetal head
2# /eassure the mother a&d e1plai& the &eed for forceps
3# (o&itor mother a&d fetus duri&" delivery possible i&ury
5# 6ssist with repair of a&y laceratio&s
4# ;ac''m e+t%action

1# 6 cap li'e suctio& device is applied to the fetal head to facilitate e1tractio&
2# Suctio& is used to assist i& delivery of the fetal head
3# 0ractio& is applied duri&" uteri&e co&tractio&s u&til desce&t of the fetal head is achieved
2=2
4# 0he suctio& device should &ot be 'ept i& place a&y lo&"er tha& 25 mi&utes
5# (o&itor 87/ every 5 mi&utes if e1ter&al fetal mo&itori&" is &ot used
6# 6ssess &ewbor& i&fa&t at birth a&d throu"hout postpartum period for si"&s of cerebral trauma
$# (o&itor for developi&" cephalohematoma
=# 3aput succeda&eum is &ormal a&d will resolve i& 24 hours
7# Cesa%ean &eli(e%y

1# ?elivery of the fetus usually throu"h a tra&s-abdomi&al, low-se"me&t i&cisio& of the uterus
2# +reoperative
a# !f pla&&ed, prepare the mother a&d part&er
b# !f a& emer"e&cy, Kuic'ly e1plai& the &eed a&d procedure to the mother a&d part&er
c# )btai& i&formed co&se&t
d# (a'e sure that the preoperative dia"&ostic tests are do&e, i&cludi&" the /h factor
e# +repare to i&sert a& !@ li&e a&d a 8oley catheter
f# +repare the abdome& as prescribed
"# (o&itor the mother a&d fetus co&ti&uously for si"&s of labor
h# +rovide emotio&al support
i# 6dmi&ister preoperative medicatio&s as prescribed
3# +ostoperative
a# (o&itor vital si"&s
b# +rovide pai& relief
c# .&coura"e tur&i&", cou"hi&", a&d deep breathi&"
d# .&coura"e ambulatio&
e# (o&itor for si"&s of i&fectio& a&d bleedi&"
f# -ur&i&" a&d pai& o& uri&atio& may i&dicate a bladder i&fectio&
"# 6 te&der uterus a&d foul-smelli&" lochia may i&dicate e&dometritis
h# 6 productive cou"h or chills may i&dicate
p&eumo&ia
C-MLICATI-NS -) LA1-R AND DELI;ER=
%ete%m Labo%
+reterm labor is labor that be"i&s after 2B wee's "estatio& a&d before 3$ wee's "estatio&#
2=3
)btai& thorou"h obstetric
history5ow bac' pai&.0!)5)4F
)btai& specime& for 3-3 & ,/6
?etermi&e freKue&cy, duratio& &
i&te&sity of uteri&e co&tractio&s
?etermi&e cervical dilatatio&s a&d
effaceme&t
6ssess status of membra&es a&d
bloody show
.valuate fetus for distress, si2e
a&d maturity
Suprapubic pressure
@a"i&al pressure
/hythmic uteri&e co&tractio&s :2
uteri&e co&tractio&s lasti&" 3B
seco&ds withi& 15 mi&utes;
3ervical dilatatio& Y4 cm &
effaceme&t 5BC or less
.1pulsio& of cervical mucus plus
-loody dhow
S!4*S /SF(+0)(S 6SS.SS(.*0
+/)(
+erform measures to ma&a"e or stop +reterm
labor(6*64.(.*0
+lace o& 3-/ i& side-lyi&" positio&
+repare fro possible ultrasou&d, am&ioce&tesis,
tocolytic a&d steroid therapy
6dmi&ister meds as prescribed
6ssess S/. such as hypote&sio&, dysp&ea, chest
pai& a&d 87/ e1ceedi&" 1=B b#p#m#

,yspnea on e@ertion and increased #a*inal


mucus are common discomforts caused by
the physiolo*ic chan*es of pre*nancy.
+rovide adeKuate hydratio&
+rovide emotio&al support
*,/S!*4 (6*64.(.*0
4oal< +/.@.*0!)* )8 +/.0./( ?.5!@./F
R-M :%emat'%e R'$t'%e of Memb%ane*
- Spo&ta&eous rupture of am&iotic membra&es prior to o&set of labor, maybe preterm :before 3= wee's
"estatio&; or term
(6*64.(.*0<6SS.SS(.*0
1# %ith i&fectio&< a&tibiotics a&d delivery of i&fa&t
2# %ithout i&fectio&<
34-36 wee's of "estatio&M delay birth,
am&ioce&tesis a&d mo&itor 5S ratio of the
baby
2=-32 wee's of "estatio&M delay birth,
admi&ister steroids to haste& maturity of the
lu&"s a&d decreased /?S

"he *ood indicator of fetal lun* maturity in a


pre*nant diabetic is presence of
(6*64.(.*0
2=4
phosphatid*lycerol in the amniotic fluid.
3)*0/!-,0!*4 8630)/
6# ,mbilical cord prolapse
If the fetus is at HD station and the membranes rupture3 the patient is at ris8 for prolapsed cord.
5ou can determine if a prolapsed cord e@ists if you perform a #a*inal e@am.
R-LASE UM1ILICAL C-RD

8i"ure 23

*e
0
o

c
o
&
t
r
o
l
i
&
t
r
a
v
a
s
c
u
l
a
r
c
o
a
"
u
l
a
t
i
o
&

+/)56+S. ,(-!5!365 3)/?
2=5
i
&

t
h
e
p
u
l
m
o
&
a
r
y

c
i
r
c
u
l
a
t
i
o
&

*a
)
1
y
"
e
&

w
i
t
h

f
a
c
e
-
m
a
s
'
#
S
t
e
2=6
r
i
l
e
h
a
&
d

"
l
o
v
e
0
u
r
&

s
i
d
e
t
o

s
i
d
e
-
7
e
l
p
s
m
a
y

b
e
e
l
e
v
a
t
e
d

t
o

s
h
2=$
i
f
t
t
o

f
e
t
a
l
p
r
e
s
e
&
t
i
&
"

t
o
w
a
r
d

d
i
a
p
h
r
a
"
m
#

)
0
7
.
/
S
<
+
r
e
m
a
2==
t
u
r
i
t
y
,
7
y
p
o
1
i
a
,
(
e
c
o
&
i
u
m

a
s
p
i
r
a
t
i
o
&
,
8
e
t
a
l
d
e
a
t
h

i
f
d
e
l
a
y
e
d
2=A

o
r
u
&
d
i
a
"
&
o
s
e
d
%
a
s
t
#
a
*
i
n
a
l
d
e
l
i
#
e
r
y

)
i
t
h

f
o
r
c
e
p
s
0
h
e
c
o
r
d

m
2AB
a
y

t
h
e
&

p
r
e
s
e
&
t
/
v
i
s
i
b
l
e
\

t
h
e
v
u
l
v
a
#

N
o
t
e
"
D
o

n
o
t
a
t
t
e
+
,
t
t
o
2A1

,
u
s
h

t
h
e

c
o
r
d

i
n
t
o

t
h
e

u
t
e
r
u
s
#
?
y
s
t
o
c
i
a

*
)
0
.
"
%
i
r
s
t
d
i
s
c
2A2
o
v
e
r
e
d

.
h
e
n

t
h
e
r
e

i
s

v
a
r
i
a
3
l
e

d
e
c
e
l
e
r
a
t
e
d

,
a
t
t
e
r
n

8
7
/

2A3
p
a
t
t
e
r
&
v
a
r
i
a
b
l
e
<
?
e
c
e
l
e
r
a
t
i
o
&
s
w
i
t
h
c
o
&
t
r
a
c
t
i
o
&
s
o
r
b
e
t
w
e
e
&
c
o
2A4
&
t
r
a
c
t
i
o
&
o
r
f
e
t
a
l
b
r
a
d
y
c
a
r
d
i
a
p
r
e
s
e
&
t
+
e
r
s
i
s
t
e
&
t
&
o
&

r
e
a
s
s
u
r
2A5
i
&
"

f
e
t
a
l
h
e
a
r
t
r
a
t
e
>

f
e
t
a
l
d
i
s
t
r
e
s
s
6
t
r
o
p
h
y

o
f
t
h
e
u
m
b
i
l
i
c
a
l
2A6
c
o
r
d

&

c
o
r
d

p
r
o
t
r
u
d
i
&
"

f
r
o
m

v
a
"
i
&
a

3
o
r
d
m
a
y
b
e
p
a
l
p
a
t
e
d
i
&
c
2A$
e
r
v
i
1
/
v
a
"
i
&
a

/
e
f
l
e
1

c
o
&
s
t
r
i
c
t
i
o
&

w
h
e
&

c
o
r
d

i
s
e
1
p
o
s
e
d

t
o

2A=
a
i
r

%
e
t
a
l
P
o
s
i
t
i
o
n

o
t
h
e
r

t
h
a
n

c
e
p
h
a
l
i
c

p
r
e
s
e
n
t
a
t
i
o
n
s
+
r
e
m
2AA
a
t
u
r
i
t
y
<

*
)
0
.
<
S
m
a
l
l
f
e
t
u
s
a
l
l
o
w
s
m
o
r
e
s
p
a
c
e
a
r
o
u
&
d

p
r
e
s
e
&
t
i
&
"
3BB

p
a
r
t
#
+
o
l
y
h
y
d
r
a
m
&
i
o
s
M
u
l
t
i
p
l
e

f
e
t
a
l
*
e
s
t
a
t
i
o
n
8
e
t
o
+
e
l
v
i
c
d
i
s
3B1
p
r
o
p
o
r
t
i
o
&
6
b
&
o
r
m
a
l
l
y

l
o
&
"

u
m
b
i
l
i
c
a
l
c
o
r
d
#
P
l
a
c
e
n
t
a

P
r
e
#
i
a
I
3B2
n
t
r
a
u
t
e
r
i
n
e

t
u
m
o
r
s

t
h
a
t
p
r
e
#
e
n
t
t
h
e

p
r
e
s
e
n
t
i
n
*

p
a
r
t
f
r
o
m

e
n
3B3
*
a
*
i
n
*

P
S
m
a
l
l
f
e
t
u
s
CAUSESEa%ly
anal"esia-+ytocin an&
amnionityH=ERT-NIC
LA1-R ATTERNS :%ima%y
ine%tia*H=-T-NIC LA1-R
ATTERNS :Secon&a%y
ine%tia*-CCURRENCELatent
$!ase of labo%Acti(e $!ase of
labo%TREATMENTRest an&
se&ation*ursi&"
!&terve&tio&N&2E" 2he nurse=s
J1 ,riorit1 action to a ,rola,se
cord is to assess the %etal heart
rate. A ,rola,sed cord interru,ts
the o$1gen and nutrient %lo. to
the %etus. )% the %etus doesn=t
receive adeGuate o$1gen/
h1,o$ia develo,s/ .hich can
lead to central nervous s1ste+
da+age in the %etus.*istor1 o%
the DiseaseDe%inition
-owel or bladder diste&tio&
(ultiple "estatio&
5ar"e fetus
7ydram&ios
4ra&dmultiparity
3esarea& sectio& if labor does &ot
resume
8etal mo&itori&"
(Pillitteri/ 8aternal and #hild
Nursing/ ,.5;;:
(Pillitteri/ 8aternal and #hild
Nursing/ ,.6MA-6MK:
). Dystocia
G.
8etal &utrie&ts supply
3ompressio& of the
umbilical cord
*
ur
si
&
"
?
ia
"
&
os
is
3B4
7# - ?ifficult,
pai&ful, ab&ormal
pro"ress of labor of
more tha& 24 hours
!#
1# +owers/ uteri&e
i&ertia/ co&tractio&
2#
3#
2he ,ri+ar1 goal .ith a
,rola,sed o% the u+3ilical cord
is to re+ove the ,ressure %ro+
the cord. #hanging the
+aternal ,osition is the %irst
intervention. Acce,ta3le
,ositions include 7nee-chest/
side-l1ing and elevation o% the
hi,s. 2he nurse +a1 also
,er%or+ a vaginal e$a+ination
and atte+,t to ,ush the
,resenting ,art o%% the cord.
Ad+inistering the o$1gen
3ene%its the %etus onl1 i%
circulation through the cord has
3een reesta3lished.
Start or mai&tai& a& !@ as
prescribed# ,se of lar"e-
"au"e catheter whe& starti&"
the !@ for blood a&d lar"e
Kua&tities of fluid i&ta'e#
6dmi&ister o1y"e& by face >
mas' to provide hi"h o1y"e&
co&ce&tratio& at = >1B5/mi&#
!&struct patie&t to clea&se
from the fro&t to the bac'#
.1plai& the importa&ce of
ha&d washi&" before a&d
after peri&eal care#
)07./ (6*64.(.*0<
/epositio& clie&t to
tre&dele&bur" or '&ee-
chest positio&
)1y"e&
+ush prese&ti&" part
upward
6pply moiste&ed sterile
towels
?elivery as soo& as
possible

8luid volume deficit related to active hemorrha"e


3B5
4# +assa"eway
a# 3o&tracted pelvis
b# ,&favorable pelvic shapes
(a&a"eme&t<
i# .valuate pelvic diameters
ii# 3o&ti&ue labor with careful mo&itori&"
iii# +erform assisted va"i&al or caesarea& delivery
5# +syche
a# 8ear, a&1iety ad te&sio& i&crease stress a&d decrease uteri&e co&tractility
b# Stress i&terferes with the clie&ts ability with her co&tractio&s
c# Stress i&crease fati"ue
(a&a"eme&t<
i# (o&itor clie&ts psycholo"ic respo&se to labor
ii# ?etermi&es clie&ts level of stress
iii# +rovide support
iv# .&coura"ed rela1atio&
D. Infection
"he infant is at ris8 to de#elop thrush if the pre*nant )oman has monillial infection at the time of #a*inal
deli#ery
3lamydia 4o&orrhea Syphilis 6!?S 0)/37
ACFUIRED IMMUN-DE)ICIENC= S=NDR-ME :AIDS*
a# 0ra&smissio&
6# 6cross the place&tal barrier
-# ?uri&" the process of labor a&d delivery
3# @ia breast mil'
-7!@ ca& cross some membra&es such as the place&tal barrier, the blood-brai& barrier, va"i&al mucosa, a&d :i&
the &eo&ate; the walls of the "astroi&testi&al tract
-+re&atal tra&smissio& from i&fected mother to fetus or &ewbor& via tra&splace&tal tra&smissio&, via
co&tami&atio& with mater&al blood duri&" birth, or throu"h breast mil'
b# *ursi&" (a&a"eme&t
3B6
Infection
6void procedures that i&crease the ris' of pre&atal tra&smissio&, such as am&ioce&tesis a&d fetal scalp sampli&"
Note that i% the %etus has not 3een e$,osed to *)V in utero/ the highest ris7 e$ists during deliver1 through
the 3irth canal
*ever use scalp electrodes
6void episiotomy to decrease the amou&t of mater&al blood i& a&d arou&d the birth ca&al
+romptly remove the &eo&ate from the motherLs blood followi&" delivery
*)0.< *)V has 3een %ound to 3e trans+itted through the 3reast +il7 %ro+ +other to 3a31.
2here%ore/ 3reast %eeding isn=t reco++ended %or a +other .ho is *)V-,ositive. 4hile trans+ission rates
o% *)V in%ection %ro+ +other to in%ant range %ro+ 5;F to M6F/ ,ro%essionals esti+ate the actual
trans+ission rate at a3out C;F to 6;F. 2he A)DS virus is ,assed trans,lacentall1/ so cesarean deliver1
.ill not ,revent in%ection o% the neonate. )n o,tions 5/ trans+ission %ro+ +other to %etusWchild can occur
trans,lacentall1 throughout ,regnanc1/ trough contact .ith the +other=s 3lood and vaginal secretions at
deliver1 and through ingestion o% 3rea7 +il7. )n the o,tion C/ a ne.3orn can 3e s1+,to+-%ree at 3irth and
still develo, A)DS. A true diagnostic o% *)V in%ection in neonates cannot actuall1 3e +ade until around 16
+onths o% age.
G# %eci$itate &eli(e%y
9 5abor that is completed withi& 3 hours
A pre*nant patient )ith a 8no)n history of crac8 cocaine use is in labor must be prepared for a precipitous
labor and notify the neonatolo*ist of the infant/s hi*h.ris8 status.
If a patient has a precipitous labor at ris83 the result of the labor process )ould be laceration of the soft
tissues3 uterine rupture3 and e@cessi#e uterine bleedin*.
Ab&ominal $ain &'%in"
cont%actionsINC-MLETES'&&en s!a%$
ab&ominal $ain &'%in"
cont%actionsMana"ementAASSESSMENT
Cont%actions contin'eD b't ce%(i+ fail to &ilate
;a"inal blee&in" may be $%esent
Risin" $'lse %ate an& s2in $allo%
Loss of fetal !ea%t tones
6bdomi&al te&der&ess
3essatio& of co&tractio&s
-leedi&" i&to abdomi&al cavity & sometimes i&to
va"i&a
8etus easily palpated, 870 ceased
Si"&s of shoc'
H. Ute%ine R'$t'%e
I.
H. "he t)o findin*s on physical e@am
indicate uterine rupture is loss of uterine contour
and palpable fetal part.
G.
L. "he number one ris8 factor for
uterine rupture is pre#ious cesarean
*,/S!*4 !*0./@.*0!)*
3B$
section.
M.
N.
-.
.
F.
R.
S.
T. 3)(+5.0.
2# (o&itor clie&t a&d fetus closely
3# +ossibly admi&ister tocolytic a"e&ts
4# +repare for emer"e&cy birth
+redisposi&" 8actors<
H. Amniotic fl'i& embolism
An amniotic fluid embolism is )hen the amniotic fluid lea8s into the maternal bloodstream b"he causes of an
amniotic fluid embolism are difficulty in labor3 or hyperstimulation of the uterus. Polyhydramnios is an
e@cessi#e amniotic fluid.
(6*!8.S060!)* (6*64.(.*0
?ysp&ea
Sharp, chest pai&
+allor or cya&osis
8rothy, blood-ti&"ed mucus
)1y"e&
3+/
!&tubatio&
?elivery
SECTI-N ;. -STARTUM
+7FS!)5)4!365 (60./*65 376*4.S
A. L&#*)A > dischar"e from the uterus duri&" the first 3 wee's after delivery#
)ncreasing Lochia as the da1 ,asses 31 +a1 indicate *e,arin )nto$ication.
L&#*)AL #*ANGES
L&#*)A !BA
O ?ar' red dischar"e occurri&" i& the first 2-3 days#
O 3o&tai&s epithelial cells, erythrocytes a&d decidua#
O 3haracteristic huma& odor#
L&#*)A SE&SA
O +i&'ish to brow&ish dischar"e occurri&" 3-1B days after delivery#
O Serosa&"ui&eous dischar"e co&tai&i&" decidua, erythrocytes, leu'ocytes, cervical mucus a&d
microor"a&isms#
O 7as a stro&" odor#
L&#*)A ALBA
O 6lmost colorless to creamy yellowish dischar"e occurri&" from 1B days to 3 wee's after delivery#
3B=
O 3o&tai&s leu'ocytes, decidua,epithelial cells, fat, cervical mucus, cholesterol crystals, a&d bacteria#
O 7as &o odor#
B. !2E!S
O +rocess of i&volutio& ta'es 4-6 wee's to complete#
O %ei"ht decreases from 2 lbs to 2 o2#
O 8u&dus steadily desce&ds i&to true pelvis9 8u&dal hei"ht decreases about 1 fi&"erbreadth :1 cm;/day9
by 1B-14 days postpartum, ca&&ot be palpated abdomi&ally#
#
#. !terine )nvolution
1# ?escriptio&
a# 0he rapid decrease i& the si2e of the uterus as it retur&s to the &o&pre"&a&t state
b# 3lie&ts who breastfeed may e1perie&ce a more rapid i&volutio&
2# 6ssessme&t
a# %ei"ht of the uterus decreases from 2 pou&ds
to 2 ou&ces i& 6 wee'
b# .&dometrium re"e&erates
c# 8u&dus steadily desce&ds i&to the pelvis
d# 8u&dal hei"ht decreases about 1 fi&"erbreadth :1 cm; per day
3BA
.# -y 1B days postpartum, uterus ca&&ot be palpated abdomi&ally
*)0.< ,e#iation of the fundus to the ri*ht or left and location of the fundus abo#e the umbilical are si*ns that
the bladder is distended
NO"$1 +ei*ht of the Umbilicus on the %irst Postpartum ,ay
"he hei*ht is usually SLI-+"L5 belo) the umbilicus about DI hours after deli#ery. "he top of the umbilicus is
normally MI,6A5 bet)een the umbilicus and the symphysis pubis.
D. Breasts
1# -reasts co&ti&ue to secrete colostrum
2# 6 decrease i& estro"e& a&d pro"estero&e levels after delivery stimulates i&creased prolacti& levels, which
promote breast mil' productio&#
3# -reasts become diste&ded with mil' o& the third day
4# .&"or"eme&t occurs i& 4= to $2 hours i& &o& breast feedi&" mothers#
*)0.<
Brad1cardia is a nor+al ,h1siologic change %or L-1; da1s ,ost,artu+
E. Gastrointestinal tract
1# %ome& are usually very hu&"ry after delivery
2# 3o&stipatio& ca& occur
3# 7emorrhoids are commo&
III. -STARTUM NURSING INTER;ENTI-NS
(o&itor vital si"&s
*)0.< Maternal temperature durin* the first DI hours follo)in* deli#ery may rise to 9BB. IV % 0C;VC7
as a result of dehydration. "he nurse can reassure the ne) mother that these symptoms are normal.
+ostpartum .1ercise
Supine Position )ith the 8nee/s fle@ed3 and then inhale deeply )hile allo)in* the abdomen to e@pand
and e@hale )hile contractin* the abdominal muscles. "he purpose of this e@ercise is to stren*then the
abdominal muscles. $@amples are reachin* for the 8neesF push ups and sits ups on the first postpartum
day.
6ssess hei"ht, co&siste&cy, a&d locatio& of the fu&dus
(o&itor color, amou&t, a&d odor of lochia
6ssess lochia a&d color volume
Give hoGA8 to +other i% ordered. hoGA8 ,ro+otes l1sis o% %etal h (P: B#s.
Ad+inister hoGa+ as ,rescri3ed .ithin M2 hours ,ost,artu+ to the h-negative client .ho has
given 3irth to an h-,ositive neonate.
hoga+ (D: i++une glo3ulin is given 31 intra+uscular in0ection/
3hec' episiotomy a&d peri&eum for si"&s of i&fectio&#
+romote successful feedi&"#
Non-nursing .o+an- tight 3ra %or M2 hours/ ice ,ac7s/ +ini+i9es 3reast
sti+ulation.
31B
Nursing .o+an- success de,ends on in%ant suc7ing and +aternal ,roduction o%
+il7.
IPost,artu+ BluesD 0C.K days7 H Normal occurrence of Nroller coasterO emotions
Se$ual activities. abstain from intercourse until episiotomy is healed and lochia ceased
around C.I )ee8s. emind that Assess hei*ht3 consistency3 and location of the fundus
breastfeedin* does not *i#e ade4uate protection.
6ssess breasts for e&"or"eme&t
(o&itor episiotomy for heali&" : assess dehisce&ce & evisceratio&;
6ssess i&cisio&s or dressi&"s of cesarea& birth clie&t : pro&e to i&fectio&;
(o&itor bowel status : pro&e to co&stipatio&;
(o&itor ! &B
.&coura"e freKue&t voidi&" :preve&t uri&ary rete&tio& which will predispose the mother to uterus
displaceme&t & i&fectio&;
.&coura"e ambulatio& : to preve&t thromboplebitis & paralytic ileus;

6ssess bo&di&" with the &ewbor& i&fa&t : to preve&t failure to thrive;


*)0.<
A ,ositive 3onding e$,erience is indicated .hen the +other turns her %ace to.ard the 3a31 to
initiate e1e-to-e1e contact. &3servation o% ne. +others has sho.n that a %airl1 regular
,attern o% +aternal 3ehaviors is e$hi3ited at %irst contact .ith the ne.3orn. 2he +other
%ollo.s a ,rogression o% touching activities %ro+ %ingerti, e$,loration to.ard ,al+ar contact
to en%olding the in%ant .ith the .hole hand and ar+. 2he +other also increase the ti+e
s,ent in the en %ace ,osition. 2he +other arranges hersel% or the ne.3orn so that her %ace
and e1es are in the sa+e ,lane as in her in%ant.
I;. -STARTUM DISC-M)-RTS
6# Perineal disco+%ort
A,,l1 ice ,ac7s to the ,erineu+ during the %irst 2C hours to reduce s.elling a%ter the %irst 2C hours/ a,,l1
.ar+th 31 sit9 3aths
-# E,isioto+1
1# !&struct the clie&t to admi&ister peri&eal care after each voidi&"
2# .&coura"e the use of a& a&al"esic spray as prescribed
3# 6dmi&ister a&al"esics as prescribed if comfort measures are u&successful
3# Breast disco+%ort
+/.@.*0!)*<
"he ($S" P$&$N"ION "$C+NIQU$ IS "O $MP"5 "+$ ($S" $-ULAL5 AN, %$QU$N"L5 6I"+
%$$,IN-S. "he D
nd
is $'P$SSIN- A LI""L$ MIL= ($%O$ NUSIN-3 MASSA-IN- "+$ ($AS"S
-$N"L5 O "A=IN- A 6AM S+O6$ ($%O$ %$$,IN- MA5 +$LP "O IMPO&$ MIL= %LO6.
Placin* as much of the areola as possible into the neonate/s mouth is one method. Other methods include
chan*in* position )ith each nursin* so that different areas of the nipples recei#e the *reatest stress from
nursin* and a#oidin* breast en*or*ement3 )hich ma8e I difficult for the neonate to *rasp. In addition3 nursin*
more fre4uently3 so that a ra#enous neonate is not suc8in* #i*orously at the be*innin* of the feedin*s3 AN,
%$$,IN- ON ,$MAN, to pre#ent o#er hun*er is helpful. AI,5IN- "+$ NIPPL$S AN, $'POSIN-
311
"+$M "O "+$ LI-+" +A&$ ALSO ($$N $COMM$N,$,. 6arm "ea ba*s3 )hich contain tannic acid
also3 )ill sooth soreness. 6$AIN- A SUPPO"I&$ (ASSI$$ ,O$S NO" P$&$N" ($AS"
$N-O-$M$N". APPL5IN- IC$ and LANOLIN ,O$S NO" $LI$&$ ($AS" $N-O-$M$N". 0Pa*e
9K; .9KU lippincot7
!*0./@.*0!)*<
Measures that help relie#e nipple soreness in a breast.feedin* client include lubricatin* the nipples )ith a fe)
drops of e@pressed mil8 before feedin*s3 applyin* ice compresses ?ust before feedin*s3 lettin* the nipples air dry
after feedin*s3 and a#oidin* the use of soap on the nipples.
*)0.< Specific &ursi&" care for breast .&"or"eme&t
1. Breast%eed %reGuentl1
2. A,,l1 .ar+ ,ac7s 3e%ore %eeding
5. A,,l1 ice ,ac7s 3et.een %eedings
*)0.< Specific *ursi&" 3are for 3rac'ed &ipples
1. E$,ose ni,,les to air %or 1; to 2; +inutes a%ter %eeding
2. otate the ,osition o% the 3a31 %or each %eeding
5. Be sure that the 3a31 is latched on to the areola/ not 0ust the ni,,le
*)0." Do not use soa, on the 3reasts/ as it tends to re+ove natural oils/ .hich increases the chance o%
crac7ed ni,,les
NO"$1 In#erted Nipples
Push the areola tissues a)ay from the nipples3 and then *rasp the nipples to tease them out of the tissue. Usin* a
6oolrich breast shield3 )hich pushes the nipples throu*h openin*s in the shield3 also can help o#ercome in#erted nipples
Phenyl8etonuria
outine Screenin* is done after the neonate has been breast feed for I; hours. "he LA"$AL +$$L 0+$$L
S"IC=7 is the best site because it pre#ents dama*e to the posterior tibial ner#e and artery and plantar artery.

-STARTUM DISCHARGE TEACHINGS
A. Gene%al %inci$les/Consi&e%ations
A. 1%east )ee&in"
0he 6merica& 6cademy of +ediatrics recomme&ds be"i&&i&" breast feedi&" as soo& as possible after
delivery or duri&" the first period of reactivity# 6 &eo&ate that will be breast fed should &ot be "ive& formula by
bottle at this time# 8an1 institutions ,rovide sterile .ater %or the initial %eeding to assess %or eso,hageal
atresia. Because colustru+ is not irritating i% as,irated and is readil1 a3sor3ed 31 the neonate=s res,irator1
s1ste+/ 3reast %eeding can 3e done i++ediatel1 a%ter 3irth. #olustru+ contains anti3odies that the neonate
lac7s/ such as )++unoglo3ulin A. -reast feedi&" stimulates the o1ytoci& secretio&, which causes the uteri&e
muscles to co&tract#
*)0.< &ral contrace,tives containing estrogen are not reco++ended %or 3reast%eeding
+others' ,rogestin-onl1 3irth control ,ills are less li7el1 to inter%ere .ith the +il7 su,,l1 1C.
Ba31 .ill develo, his or her o.n %eeding schedule. *or+onal contrace,tives +a1 cause a
decrease in the +il7 su,,l1 and are 3est avoided during the %irst L .ee7s a%ter 3irth.
312
*)0.< 2he condo+ is the onl1 sa%e/ non ,rescri,tion contrace,tive to use .hile a .o+an
lactating and before there is normal uterine in#olution at this time.
*)0.< LE2 D&4N E(LE? &( 2*E BEAS2
&$1to$in is the J1 %actor that sti+ulates the let do.n re%le$ .hile Prolactin is the one that
sti+ulates the acini cells to ,roduce +il7.
A. )i%st 1%east )ee&in"
0he mother should be e&coura"ed to &urse freKue&tly duri&" the first few days after delivery# ($AS"
%$$,IN- %O A" L$AS" K.9B MINU"$S P$ SI,$ %O "+$ L$" ,O6N $%L$' "O ($-IN#
,
n&
b%east )ee&in"
680./ 07. 8!/S0 -/.6S0 8..?!*4, the mother should breast feed her i&fa&t 2-3 hours u&til her
mil' supply is established#
1%east mil2 contents (e%s's co<Hs mil2
($AS" MIL= is hi*her in fat content than co)/s mil8F CEG . EEG of the calories in breast mil8 are from fat.
Co)/s mil8 is hi*her in iron3 sodium calcium < phosphorus.
3)%S (!5J
6ccordi&" to the 6merica& 6cademy of +ediatrics :66+; recomme&ds that i&fa&ts be "ive& breast mil' of
formula UN"IL 9 5$A O% A-$. "he AAP Committee decreed that co)/s mil8 could be substituted in the
S$CON, : MON"+S O% LI%$3 (U" ONL5 I% "+$ AMOUN" O% MIL= CALOI$S ,O$S NO" $'C$,,
:EG of total calories a&d iro& is replaced by solid foods# "he protein content o co)/s mil8 is too hi*h3 and
therefore is poorly di*ested3 and may cause *astrointestinal tract bleedin*
S,++5.(.*0!*4 -/.6S0 8..?!*4 %!07 -)005.? 8..?!*4
(ottle supplements tend to cause a decrease in the breast mil8 supply and demand for breast feedin*3 AN,
S+OUL, ($ A&OI,$,
*)0.< Breast +il7 Storage
Never store it in clean glass containers 3ecause i++unoglo3ulins tend to stic7 to glass 3ottles and
the containers should BE S2E)LE. 2he client should use S2E)LE PLAS2)# #&N2A)NES
la3eled .ith ti+e/ date and a+ount. Store 3reast +il7 at the re%rigerator %or CA hours or in a %ree9er
%or 2 +onths. (ro9en 3reast +il7 should 3e tha.ed in the re%rigerator %or a %e. hours/ ,laced under
.ar+ ta, .ater/ then sha7e it.
*)0.< S2A2 &( S&L)D (&&D is usuall1 C +onths.
1. 1URING & )EEDING
-,/+!*4
6&other word is bubbli&" the &eo&ate should be do&e after 5 mi&utes of feedi&", i& the middle of the feedi&",
a&d at the e&d o the feedi&"#0he &eo&ate should be held i& a& upri"ht positio& a&d patted o& the bac'#
+)S!0!)* 8)/ 8..?!*4
"he neonate should be placed on the ri*ht side3 placin* the patient on prone position has been associated )ith
SI,S 0Sudden Infant ,eath Syndrome7
313
NO"$1
If the bottle nipple is 8ept full of formula3 the infant )ill suc8 less air3 the infant is less li8ely to spit up and less
li8ely to s)allo) air. S)allo)in* air can lead to colic. A 3ottle should never 3e ,ro,,ed 3ecause o% the
chance o% as,iration. Bur,ing should occur a%ter each 2 o9. (urpin* fre4uently decreased the chance of
spittin* up. "he nipple should be all the )ay in the infant/s mouth so the infant can create a *ood suc8.
NO"$1 Bottle-%ed in%ants are usuall1 %ed .ithin the %irst %e. hours a%ter 3irth. 2he nurse +ust deter+ine i%
the ne.3orn is read1 %or this %eeding. Signs are indicative o% readiness %or %eeding include ,resence o%
rooting and suc7ing re%le$es/ active 3o.el sounds/ a3sence o% a3do+inal dissension/ and a3sence o% signs o%
res,irator1 distress.
*)0.< 7ow to stimulate the !&fa&tEs lips to ta'e the &ipple]
Li*htly brushin* the neonates lips )ith nipple causes the neonate to open the mouth the be*in suc8in*. Such
techni4ues as pullin* do)n on the chin3 s4uee2in* the chee83 or placin* the nipple directly in the mouth force
the mouth open or force the neonates to ta8e the nipple.
C. syc!olo"ical A&a$tation
314
+
o
s
t
p
a
r
t
u
m

b
l
u
e
s
<
o
v
e
r
w
h
e
l
m
i
&
"

s
a
d
&
e
s
s
0a'i&"-i& +hase
+ostpartum
+sychoses0a'i&"-
hold +hase
+ostpartum depressio&
5etti&"-4o
R'binOs ost$a%t'm !ases of Re"ene%ation : -STARTUM S=CH-S-CIAL ADATATI-N*
I2AK)NG )ND P*ASE (DEPENDEN2: )i%st 3 Days
315
#?uri&" this time, food a&d sleep are a maor focus for the clie&t# !& additio&, she wor's throu"h the birth
e1perie&ce to sort out reality from fa&tasy a&d to clarify a&y misu&dersta&di&"s# 0his phase lasts 1 to 3 days
after birth# 0he primary co&cer& is to meet her ow& &eeds#
O 0a'es place 1-2 days postpartum
O 8other is ,assive and de,endent' concerned .ith o.n needs#
O @erbali2es about the delivery e1perie&ce#
O Sleep/food importa&t#
O 8other %ocuses on her o.n ,ri+ar1 needs/ such as slee, and %ood
!mporta&t for the &urse to liste& a&d to help the mother i&terpret the eve&ts of delivery to
ma'e them more mea&i&"ful
*ot a& optimum time to teach the mother about baby care
I2AK)NG *&LDD P*ASE
(DEPENDEN2@)NDEPENDEN2:
0he clie&t is co&cer&ed re"ardi&" her &eed to resume co&trol of all facets of her life i& a compete&t ma&&er# 6t
this time, she is ready to lear& self-care a&d i&fa&t care s'ills#
O 3-1B days postpartum
O (other strives for i&depe&de&ce a&d be"i&s to reassert herself#
O (ood swi&"s occur# (ay cry for &o reaso&#
O (a1imal sta"e of lear&i&" readi&ess#
O (other reKuires reassura&ce that she ca& perform tas's of motherhood#
O -e"i&s to assume the tas's of motheri&"
O 6& optimum time to teach the mother about baby care#
ILE22)NG G&D P*ASE ()N2EDEPEN#E:
O 1B to 6 wee's postpartum
O /ealistic re"ardi&" role tra&sitio&#
O Shows patter& of life-style that i&cludes the &ew baby but still focuses o& e&tire family as a u&it#
O 6ccepts baby as separate perso&#
(other may feel deep loss over separatio& of the baby from part of the body a&d may "rieve
over the loss
(other may be cau"ht i& a depe&de&t/i&depe&de&t role, wa&ti&" to feel safe a&d secure yet
wa&ti&" to ma'e decisio&s
0ee&a"e mothers &eed special co&sideratio& because of the co&flict ta'i&" place withi&
them as part of adolesce&ce
-STARTUM ?ARNING S/S T- RE-RT T- THE H=SICIAN
!&creased bleedi&", clots or passa"e of tissue#
-ri"ht red va"i&al bleedi&" a&ytime after birth#
+ai& "reater tha& e1pected#
0emperature elevatio& to 1BB#4^ 8#
8eeli&" of full bladder accompa&ied by i&ability to void#
.&lar"i&" hematoma#
8eeli&" restless accompa&ied by pallor9 cool, clammy s'i&9 rapid 7/9 di22i&ess9 a&d visual disturba&ce#
+ai&, red&ess, a&d warmth accompa&ied by a firm area i& the calf#
?ifficulty breathi&", rapid heart rate, chest pai&, cou"h, feeli&" of apprehe&sio&, pale, cold, or blue s'i& color
)V. P&S2 PA2!8 #&8PL)#A2)&NS
A. HEM-RRHAGE
316
36,S.S S!4*S )8 7.()//764. (6*64.(.*0
"he T9 cause of POS"PA"UM
+$MO+A-$ IS $"AIN$,
PLAC$N"AL %A-M$N"S.
Uterine atony and #a*inal <
cer#ical tears are associated )ith
early postpartum hemorrha*e
"he TD cause is O&$.
,IS"$N"ION O% "+$ U"$US
from more than 09B7 pounds3
O"+$S A$1 IBBB *ms3
neonate3 e@cessi#e o@ytocin use3
Polyhydramnios and Placental
,isorders.
5ou should assess for uterine
atony after a c.section deli#ery.
"his is more common after a c.
section than after a #a*inal
deli#ery.
-o""y uterus :does &ot
respo&d to massa"e;
A bo**y uterus )ould be
palpable abo#e the umbilicus and
)ould be soft and poorly
contracted.
6b&ormal clots u&usual
pelvic discomfort or headache
.1cessive or bri"ht-red
bleedi&"
Si"&s of shoc'
$arly +emorrha*e starts on the
first DI hours3 or more than EBB
ml of blood on the first DI hrs in
a Normal spontaneous deli#ery..
8luid replaceme&t
.mer"e&cy lay
)1y"e&
@ital si"&s
+eri&eal pad cou&t
+sycholo"ical support
Massa*in* the lo)er abdomen
after deli#ery is done to maintain
a firm uterus3 )hich )ill aid in
the clumpin* do)n of blood
#essels in the uterus3 thereby
pre#entin* any further bleedin*.
N(O--5 U"$US
Uterine atony means that the
uterus is not firm or it is not
contractin*. "he nurse should
*ently massa*e the uterus )hich
)ill contract the uterus and ma8e
it firm. Clients )ho are
predisposed are usually
MUL"IPL$ -$S"A"ION3
POL5+5,AMNIOS3
POLON-$, LA(O and L-A
0LA-$ -$S"A"IONAL A-$
fetus.
1. THR-M1-LE1ITIS
- !&flammatio& of the vei& caused by a clot
"he positi#e +oman/s si*n indicate is possibility of thrombophlebitis or a deep #enous thrombosis that is
present in the lo)er e@tremities.
6hen assessin* for +oman/s si*n as8 the patient to stretch her 8e*s out )ith the 8nee sli*htly fle@ed )hile
dorsifle@ the foot. A positi#e si*n is present )hen pain is felt at the bac8 of the 8nee or calf.
It is normal for a patient on ma*nesium sulfate to feel tired because it acts as a central ner#ous depressant
and often ma8es the patient dro)sy.
+reve&tive!mmobili2e
e1tremity(6*!8.S0
60!)*
6&al"esics
6&ticoa"ula&t
0hrombolytics
(6*64.(.*0 3,/60!@.
.dematous e1tremities
C. IN)ECTI-N
(6*64.(.*0+/.?!S+)S!*4 8630)/S
/upture of membra&es over 8ever 6&tibiotics
31$
(6*64.(.*0+/.?!S+)S!*4 8630)/S
24 hours before
delivery(6*!8.S060!
)*
/etai&ed place&tal
fra"me&ts
!&ter&al fetal mo&itori&"
@a"i&al i&fectio&
3hills
+oor appetite
4e&eral body malaise
6bdomi&al pai&
8oul-smelli&" lochia
Puerperial infection is an
infection of the *enital tract.
$arly si*ns and symptoms of
puerperial infection include
chills3 fe#er3 and flu.li8e
symptoms. It can occur up to one
month after deli#ery.
)1ytoci&
6&al"esics
(ai&tai& hy"ie&e
Semi-fowlers positio&s
@ital si"&s
.arly ambulatio&s
6ssess lochia
(ri*ht red blood is a normal
lochial findin* in the first DI
hours after deli#ery. Lochia
should ne#er contain lar*e
clots3 tissue fra*ments3 or
membranes. A foul odor may
si*nal infection3 as may absence
of lochia.
?# (6S0!0!S
6SS.SS(.*0<
.levated temperature, chills, "e&eral achi&", malaise a&d locali2ed pai&
.&"or"eme&t, hard&ess a&d redde&i&" of the breasts
*ipple sore&ess a&d fissures
!&flammatio& of the breast as a result of i&fectio&
Pri+aril1 seen in 3reast%eeding +others 2 to 5 .ee7s a%ter deliver1 3ut +a1 occur at an1 ti+e
during lactation
*,/S!*4 !(+.(.*060!)*<
!&struct the mother i& "ood ha&d washi&" a&d breast hy"ie&e tech&iKues
6pply heat or cold to site as prescribed
8aintain lactation in 3reast%eeding +others
Encourage +anual e$,ression o% 3reast +il7 or use o% 3reast ,u+, ever1 C hours
.&coura"e mother to support, breasts by weari&" a supportive bra
6dmi&ister a&al"esics & a&tibiotics as prescribed
E. ost$a%t'm Moo& Diso%&e%s
())? ?!S)/?./S
6
S
S
.
+ostpartum blues )&set< 1-1B days postpartum lasti&" 2 wee's or less
8ati"ue
%eepi&" a&1iety
31=
())? ?!S)/?./S
S
S
(
.
*
0
(ood i&stability
+ostpartum depressio&
Normal processes durin*
postpartum include the )ithdra)al
of pro*esterone and estro*en and
lead to the psycholo*ical response
8no)n as Wthe blues.W Postpartum
depression is a psychiatric problem
that occurs later in postpartum and
is characteri2ed by more se#ere
symptoms of inade4uacy. (ecause
the clientJs beha#ior is normal3
notifyin* her physician and
conductin* a home assessment
arenJt necessary.
)&set< 3-5 days lasti&" more tha& 2 wee's
3o&fusio&
8ati"ue
6"itatio&
8eeli&" of hopeless&ess a&d shame Slet dow& feeli&"T
6lteratio&s i& mood Sroller coaster emotio&sT
6ppetite a&d sleep disturba&ce
Accordin* to ubin3 dependence and passi#ity are typical durin* the
ta8in*.in period3 )hich may last up to C days after deli#ery. A client
e@periencin* postpartum depression demonstrates an@iety3 confusion3 or
other si*ns and symptoms consistently. Maternal role attainment occurs
o#er C to 9B months. Attachment also is an on*oin* process that occurs
*radually.
+ostpartum psychosis )&set< 3-5 days postpartum
Symptoms of depressio& plus delusio&s
6uditory halluci&atio&s
7yperactivity
SECTI-N ;II
H=SI-L-GIC STATUS -) NE?1-RN
N-TEA "he best time for physical assessment is mid)ay bet)een feedin*s. "he hun*ry is often fussy3 irritable3
ma8in* physical e@amination difficult. Manipulation after eatin* may cause the neonate to re*ur*itate or #omit.
A. INITIAL H=SICAL E@AMINATI-N & CARE -) THE NE?1-RN
#. Assessment
4e&eral "uideli&es
1# Jeep &ewbor& warm duri&" the e1ami&atio&
2# -e"i& with "e&eral observatio&s9 the& perform assessme&ts that are least disturbi&" to the &ewbor& first
3# !&itiate &ursi&" i&terve&tio&s for ab&ormal fi&di&"s
4# ?ocume&t all ab&ormal fi&di&"s
1# )bserve or assist with i&itiatio& of respiratio&s
2# 6ssess 6p"ar score
3# *ote characteristics of cry
31A
4# (o&itor for &asal flari&", "ru&ti&", retractio&s, ab&ormal respiratio&s
5# )btai& vital si"&s
6# )bserve &ewbor& for si"&s of hypothermia or hyperthermia
$# 6ssess for "ross a&omalies
,. Im$lementation
1# Suctio& mouth, the& &ares, with bulb syri&"e
2# ?ry &ewbor& a&d stimulate cryi&" by rubbi&"
3# (ai&tai& temperature stability9 wrap &ewbor& i& warm bla&'ets a&d place a stoc'i&ette cap o&
&ewbor&Ls head
*)0.< 0emperature ta'i&"
2he 3est site .ithout co+,lications is the ta7ing it in a$illa. )t is not advisa3le to ,ut it in the
+outh/ anus or ear since all o% the+ are sensitive.
4# Jeep &ewbor& with mother to facilitate bo&di&"
5# +lace &ewbor& at motherLs breast if breastfeedi&" is pla&&ed, or place o& motherLs abdome&
6# +lace &ewbor& i& warmer
$# +ositio& &ewbor& o& side or abdome& or i& modified 0re&dele&bur" positio& to facilitate drai&a"e
of mucus
=# .&sure &ewbor&Ls proper ide&tificatio&
A# 8ootpri&t &ewbor& a&d fi&"erpri&t mother o& ide&tificatio& sheet, per a"e&cy policies a&d
procedures
1B# +lace matchi&" ide&tificatio& bracelets o& mother a&d &ewbor&
*)0.< 3o&vectio&, 3o&ductio&, /adiatio& a&d .vaporatio&
Eva,oration occurs .hen .et sur%aces such as neonate=s s7in are e$,osed to air.
#onduction o% heat a.a1 %ro+ the 3od1 +a1 occur .hen the neonate co+es in direct contact .ith cold
sur%aces such as scale or cold stethosco,e.
adiation is the trans%er o heat to cooler o30ects that are not in direct contact .ith the neonate.
#onvection- 7ee,ing a.a1 the neonate %ro+ the air conditioning or cooling ducts ,revents heat loss
3. ;ital si"ns
AGAR CRITERIA
AGAR SC-RE - # ,
3)5)/ +ale -ody pi&', e1tremities blue 0otally pi&'
7.6/0 /60. 6bse&t 5ess tha& 1BB b#p#m# )ver 1BB b#p#m#
/.85.Q !//!06-!5!0F *o respo&se 4rimace @i"orous cry
(,S35. 0)*. 5imp Some fle1io& 6ctively moves
32B
AGAR SC-RE - # ,
3)5)/ +ale -ody pi&', e1tremities blue 0otally pi&'
,(-!5!365 @.!*
a&d ?,30,S
@.*)S,S co&strict
after cord id
clamped/.S+!/60
)/F .88)/0
?,30,S
6/0./!)S,S
co&strict with
establishme&t of
respiratory fu&ctio&
8)/6(.* )@65.
closes fu&ctio&ally
as respiratio&s
established, but
a&atomic or
perma&e&t closure
may ta'e several
mo&ths
7.6/0 /60.
avera"es 14B b#p#m#
-+ $3/55 mm7"
+./!+7./65
3!/3,560!)*
acrocya&osis withi&
24 hours
/-3 hi"h
immediately after
birth9 falls after 1
st

wee'
6-S.*3./ *)/(65
85)/6 !*0.S0!*.
@itami& J
"he components of Ap*ar
scorin* system are tone3
color3 irritability3 respiration
and heart rate.
1# +erform a&d
record the 6p"ar
score at 1 mi&ute
a&d at 5 mi&utes
2# !f the score is less
tha& $ at 5
6bse&t
321
AGAR SC-RE - # ,
3)5)/ +ale -ody pi&', e1tremities blue 0otally pi&'
/.S+!/60)/F
S060,SSlow, irre"ular4ood
cry
6deKuate levels of surfacta&ts :5ecithi& a&d spi&"omyeli&; e&sure mature lu&"
fu&ctio&9 preve&t alveolar collapse a&d respiratory distress sy&drome
// M 3B-=B breaths /mi&utes with short periods of ap&ea :Y 15 seco&ds; M
assess for 1 full mi&ute cha&"e &oted duri&" sleep or activity
*)0.< Periodic apnea is common in preterm infants. Usually3 *entle
stimulation is sufficient to *et the infant to breathe
322
AGAR SC-RE - # ,
3)5)/ +ale -ody pi&', e1tremities blue 0otally pi&'
!((60,/.
36/?!63
S+7!*30./ > may
allow reflu1 of food,
burped,
/.4,/4!060.-
placed *- ri"ht side
after feedi&"/.*65
SFS0.(
*ewbor& ca&Et move
food from lips to
phary&1# !&sert
&ipple well to mouth
8..?!*4 +600./S
vary
- *ewbor&s may
&urse vi"orously
immediately
afterbirth or may
&eed as lo&" as
several days to suc'
effectively
- +rovide support a&d
e&coura"eme&t to
&ew mothers duri&"
this time as i&fa&t
feedi&" is very
emotio&al doe most
mothers
NO"$1 ,istin*uishin*
Neonatal &omitin* from
e*ur*itation
&omitin* is usually sour3
loo8s li8e curdled mil8
due to +CL3 )ith a sour
odor3 )hile re*ur*itation
has no sour odor or
curdlin* of mil83 or
occurs durin* or
immediately after
feedin*.
,ri&e prese&t i& the bladder at birth but *- may &ot void doe 1
st
12-24 hours
5ater patter& is 6-1B voidi&"s/ day > i&dicative of sufficie&t fluid i&ta'e
,ri&e is pale a&d straw colored > i&itial voidi&"s may leave bric'-red spots o&
diaper : d/t passa"e of uric acid crystals i& uri&e;
!&fa&t u&able to co&ce&trate uri&e for the 1
st
3 mo&ths

323
AGAR SC-RE - # ,
3)5)/ +ale -ody pi&', e1tremities blue 0otally pi&'
?
!
4
.
S
0
!
@
.

S
F
S
0
.
(
7.+60!3
0.(+./60,/.5iver
respo&sible for cha&"i&"
7"b i&to co&u"ated
bilirubi&, which is
further cha&"ed i&to
co&u"ated :water
soluble; bilirubi& that
ca& be e1creted
.1cess u&co&u"ated
bilirubi& ca&
permeate the sclera
a&d the s'i&, "ivi&"
a au&diced or
yellow appeara&ce to
these tissues
HEAT R-DUCTI-N i& &ewbor& accomplished by<
a# (etabolism of S -/)%* 860T
- 6 special structure i& *- is a source of heat
- !&creased metabolic rate a&d activity
61illary temperature< A6#= to AA8
*ewbor& ca&Et shiver as a& adult does to release heat
*ewbor&s are u&able to mai&tai& a stable body temperature because they have a&
immature vasomotor ce&ter, a&d u&able to shiver to i&crease body heat#
*-Es body temperature drops Kuic'ly after birth > after stress occurs
easily
-ody stabili2es temperature i& =-1B hours if u&stressed
Cold stress increases oD consumption H may lead to metabolic acidosis
and respiratory distress
!((,*)5)4!3 N( de#elops o)n antibodies durin* 9
st
C months but at ris8 for infection
durin* the first : )ee8s
6bility to develop a&tibodies develops seKue&tially
Neonatal !ysical Assessment
-irth wei"htM25BB-4BB "rams :5 lbs# =o2# > = lbs# 13 o2#;
5e&"thM 45#$ > 55#A cm# :1=-22 i&ches;
324
7.6? 7ead circumfere&ceM33-35 cm :2-3 cm# 4reater tha& chest circumfere&ce;
6&terior fo&ta&el :diamo&d shape; M closes 12-1= mo&ths
+osterior fo&ta&el :tria&"le shape;M closes 2-3 mo&ths
*)0.< 0he $oste%io% fontanel is located at the intersection of the sa*ittal and
lambdoid suture is the space bet)een the pariental bonesF the lambdoid suture
separates the t)o parietal bones and the occipital bone
(oldi&"- asymmetry of head as a result of pressure i& birth ca&al
#e,halohe+ato+as donJt cross the suture lines and are the result of blood #essels
rupturin* in the babyJs scalp durin* labor. (lood outside the #asculature in a ne)born
increases the possibility of ?aundice as the ne)bornJs body tries to reabsorb the blood
. #a,ut succedaneu+3 )hich is simply soft tissue edema of the scalp3 can occur in any
labor and isnJt limited to a prolon*ed second sta*e of labor.
.F.S
*)S.-lue/ "ray
d/t scleral
thi&&ess9
perma&e&t color
established w/i&
3-12 mos#
5acrimal "la&ds
immature at birth9
tearless cry up to
2 mo&ths
Absence of tears
is common
because the
neonate/s tear
*lands are not yet
fully de#eloped
0ra&sie&t
strabismus
?ollEs eye refle1
persist for about
te& days
ed efle@1 A red
circle on the
pupils seen )hen
an
ophthalmoscope/s
li*ht is shinin*
onto the retina is
a normal findin*.
"his indicates that
the li*ht is shinin*
onto the retina
*ose breathers for first few mo&ths of life
325
7.6? 7ead circumfere&ceM33-35 cm :2-3 cm# 4reater tha& chest circumfere&ce;
6&terior fo&ta&el :diamo&d shape; M closes 12-1= mo&ths
+osterior fo&ta&el :tria&"le shape;M closes 2-3 mo&ths
*)0.< 0he $oste%io% fontanel is located at the intersection of the sa*ittal and
lambdoid suture is the space bet)een the pariental bonesF the lambdoid suture
separates the t)o parietal bones and the occipital bone
(oldi&"- asymmetry of head as a result of pressure i& birth ca&al
#e,halohe+ato+as donJt cross the suture lines and are the result of blood #essels
rupturin* in the babyJs scalp durin* labor. (lood outside the #asculature in a ne)born
increases the possibility of ?aundice as the ne)bornJs body tries to reabsorb the blood
. #a,ut succedaneu+3 )hich is simply soft tissue edema of the scalp3 can occur in any
labor and isnJt limited to a prolon*ed second sta*e of labor.
.
CON&$-$N"
S"A(ISMUS
0COSS $5$,7
It is common
durin* infancy
until a*e : months
because of poor
oculomotor
coordination
NO"$ 1
Con*enital
-laucoma
Une4ual si2e
should be
reported
immediately.
(),07 Sca&t saliva with pi&' lips
$pstein/s Pearls . small shiny )hite spec8s on the neonate/s *ums and hard palate )hich
are normal
Short a&d wea'
with deep fold of
s'i&.6/S
!&curvi&" of pi&&a a&d cartila"e depositio&
37.S0*.3J 3haracteri2ed by cyli&drical thora1 a&d fle1ible ribs
*)0.<
appears circular si&ce a&teroposterior a&d lateral diameters are about eKual
/espiratio&s appear diaphra"matic
*ipples promi&e&t a&d ofte& edematous
8il71 secretion (.itchXs +il7: co++on ( e%%ect o% estrogen:
6-?)(.* 3yli&drical with some protrusio&9 scaphoid appeara&ce i&dicates diaphra"matic her&ia
,mbilical cord is white a&d "elati&ous with two arteries a&d o&e vei& a&d be"i&s to dry
withi& 1-2 hours after delivery
326
7.6? 7ead circumfere&ceM33-35 cm :2-3 cm# 4reater tha& chest circumfere&ce;
6&terior fo&ta&el :diamo&d shape; M closes 12-1= mo&ths
+osterior fo&ta&el :tria&"le shape;M closes 2-3 mo&ths
*)0.< 0he $oste%io% fontanel is located at the intersection of the sa*ittal and
lambdoid suture is the space bet)een the pariental bonesF the lambdoid suture
separates the t)o parietal bones and the occipital bone
(oldi&"- asymmetry of head as a result of pressure i& birth ca&al
#e,halohe+ato+as donJt cross the suture lines and are the result of blood #essels
rupturin* in the babyJs scalp durin* labor. (lood outside the #asculature in a ne)born
increases the possibility of ?aundice as the ne)bornJs body tries to reabsorb the blood
. #a,ut succedaneu+3 )hich is simply soft tissue edema of the scalp3 can occur in any
labor and isnJt limited to a prolon*ed second sta*e of labor.
*)0.< ,mbilical cord
2hree vessels/ t.o arteries and one vein/ in cord' i% %e.er than three vessels are
noted noti%1 the ,h1sician
Small, thi& cord may be associated with poor fetal "rowth
6ssess for i&tact cord, a&d e&sure that damp is cured
#ord should 3e cla+,ed %or at least the %irst C hours a%ter 3irth' cla+, can 3e
re+oved hen the cord is dried and occluded
!+3ilical cla+, can 3e re+oved a%ter 2C hours
4.*!065!6
(
6
5
.
<
i
&
c
l
u
d
e
s
r
u
"
a
e
o
&

.Q0/.(!0!.S
32$
7.6? 7ead circumfere&ceM33-35 cm :2-3 cm# 4reater tha& chest circumfere&ce;
6&terior fo&ta&el :diamo&d shape; M closes 12-1= mo&ths
+osterior fo&ta&el :tria&"le shape;M closes 2-3 mo&ths
*)0.< 0he $oste%io% fontanel is located at the intersection of the sa*ittal and
lambdoid suture is the space bet)een the pariental bonesF the lambdoid suture
separates the t)o parietal bones and the occipital bone
(oldi&"- asymmetry of head as a result of pressure i& birth ca&al
#e,halohe+ato+as donJt cross the suture lines and are the result of blood #essels
rupturin* in the babyJs scalp durin* labor. (lood outside the #asculature in a ne)born
increases the possibility of ?aundice as the ne)bornJs body tries to reabsorb the blood
. #a,ut succedaneu+3 )hich is simply soft tissue edema of the scalp3 can occur in any
labor and isnJt limited to a prolon*ed second sta*e of labor.
t
h
e
s
c
r
o
t
u
m

a
&
d

t
e
s
t
e
s
d
e
s
c
e
&
d
e
d

i
32=
7.6? 7ead circumfere&ceM33-35 cm :2-3 cm# 4reater tha& chest circumfere&ce;
6&terior fo&ta&el :diamo&d shape; M closes 12-1= mo&ths
+osterior fo&ta&el :tria&"le shape;M closes 2-3 mo&ths
*)0.< 0he $oste%io% fontanel is located at the intersection of the sa*ittal and
lambdoid suture is the space bet)een the pariental bonesF the lambdoid suture
separates the t)o parietal bones and the occipital bone
(oldi&"- asymmetry of head as a result of pressure i& birth ca&al
#e,halohe+ato+as donJt cross the suture lines and are the result of blood #essels
rupturin* in the babyJs scalp durin* labor. (lood outside the #asculature in a ne)born
increases the possibility of ?aundice as the ne)bornJs body tries to reabsorb the blood
. #a,ut succedaneu+3 )hich is simply soft tissue edema of the scalp3 can occur in any
labor and isnJt limited to a prolon*ed second sta*e of labor.
&
t
o

t
h
e
s
c
r
o
t
u
m
,
r
i
&
a
r
y
m
e
a
t
u
s
<

7y

.pi
32A
7.6? 7ead circumfere&ceM33-35 cm :2-3 cm# 4reater tha& chest circumfere&ce;
6&terior fo&ta&el :diamo&d shape; M closes 12-1= mo&ths
+osterior fo&ta&el :tria&"le shape;M closes 2-3 mo&ths
*)0.< 0he $oste%io% fontanel is located at the intersection of the sa*ittal and
lambdoid suture is the space bet)een the pariental bonesF the lambdoid suture
separates the t)o parietal bones and the occipital bone
(oldi&"- asymmetry of head as a result of pressure i& birth ca&al
#e,halohe+ato+as donJt cross the suture lines and are the result of blood #essels
rupturin* in the babyJs scalp durin* labor. (lood outside the #asculature in a ne)born
increases the possibility of ?aundice as the ne)bornJs body tries to reabsorb the blood
. #a,ut succedaneu+3 )hich is simply soft tissue edema of the scalp3 can occur in any
labor and isnJt limited to a prolon*ed second sta*e of labor.
*)0
.
<
8eatus at
0estes
6ssess for
8irst
8.(65.<
33B
7.6? 7ead circumfere&ceM33-35 cm :2-3 cm# 4reater tha& chest circumfere&ce;
6&terior fo&ta&el :diamo&d shape; M closes 12-1= mo&ths
+osterior fo&ta&el :tria&"le shape;M closes 2-3 mo&ths
*)0.< 0he $oste%io% fontanel is located at the intersection of the sa*ittal and
lambdoid suture is the space bet)een the pariental bonesF the lambdoid suture
separates the t)o parietal bones and the occipital bone
(oldi&"- asymmetry of head as a result of pressure i& birth ca&al
#e,halohe+ato+as donJt cross the suture lines and are the result of blood #essels
rupturin* in the babyJs scalp durin* labor. (lood outside the #asculature in a ne)born
increases the possibility of ?aundice as the ne)bornJs body tries to reabsorb the blood
. #a,ut succedaneu+3 )hich is simply soft tissue edema of the scalp3 can occur in any
labor and isnJt limited to a prolon*ed second sta*e of labor.
Pseudo
8irst
331
7.6? 7ead circumfere&ceM33-35 cm :2-3 cm# 4reater tha& chest circumfere&ce;
6&terior fo&ta&el :diamo&d shape; M closes 12-1= mo&ths
+osterior fo&ta&el :tria&"le shape;M closes 2-3 mo&ths
*)0.< 0he $oste%io% fontanel is located at the intersection of the sa*ittal and
lambdoid suture is the space bet)een the pariental bonesF the lambdoid suture
separates the t)o parietal bones and the occipital bone
(oldi&"- asymmetry of head as a result of pressure i& birth ca&al
#e,halohe+ato+as donJt cross the suture lines and are the result of blood #essels
rupturin* in the babyJs scalp durin* labor. (lood outside the #asculature in a ne)born
increases the possibility of ?aundice as the ne)bornJs body tries to reabsorb the blood
. #a,ut succedaneu+3 )hich is simply soft tissue edema of the scalp3 can occur in any
labor and isnJt limited to a prolon*ed second sta*e of labor.
6
l
l
&
e
o
&
a
t
e
s
h
a
v
e
b
o
w
l
e
"
"
e
d

a
&
S+!*.
332
7.6? 7ead circumfere&ceM33-35 cm :2-3 cm# 4reater tha& chest circumfere&ce;
6&terior fo&ta&el :diamo&d shape; M closes 12-1= mo&ths
+osterior fo&ta&el :tria&"le shape;M closes 2-3 mo&ths
*)0.< 0he $oste%io% fontanel is located at the intersection of the sa*ittal and
lambdoid suture is the space bet)een the pariental bonesF the lambdoid suture
separates the t)o parietal bones and the occipital bone
(oldi&"- asymmetry of head as a result of pressure i& birth ca&al
#e,halohe+ato+as donJt cross the suture lines and are the result of blood #essels
rupturin* in the babyJs scalp durin* labor. (lood outside the #asculature in a ne)born
increases the possibility of ?aundice as the ne)bornJs body tries to reabsorb the blood
. #a,ut succedaneu+3 )hich is simply soft tissue edema of the scalp3 can occur in any
labor and isnJt limited to a prolon*ed second sta*e of labor.
d

f
l
a
t
f
e
e
t
*
)
0
.

*
)
/
(
6
5

8
.
6
0
,
/
.
S
<
8
a0
o
r
gl
ut
333
7.6? 7ead circumfere&ceM33-35 cm :2-3 cm# 4reater tha& chest circumfere&ce;
6&terior fo&ta&el :diamo&d shape; M closes 12-1= mo&ths
+osterior fo&ta&el :tria&"le shape;M closes 2-3 mo&ths
*)0.< 0he $oste%io% fontanel is located at the intersection of the sa*ittal and
lambdoid suture is the space bet)een the pariental bonesF the lambdoid suture
separates the t)o parietal bones and the occipital bone
(oldi&"- asymmetry of head as a result of pressure i& birth ca&al
#e,halohe+ato+as donJt cross the suture lines and are the result of blood #essels
rupturin* in the babyJs scalp durin* labor. (lood outside the #asculature in a ne)born
increases the possibility of ?aundice as the ne)bornJs body tries to reabsorb the blood
. #a,ut succedaneu+3 )hich is simply soft tissue edema of the scalp3 can occur in any
labor and isnJt limited to a prolon*ed second sta*e of labor.
e
al
%o
ld
s
e
v
e
n
3
re
as
es
o
&
so
le
s
of
fe
et
A
334
7.6? 7ead circumfere&ceM33-35 cm :2-3 cm# 4reater tha& chest circumfere&ce;
6&terior fo&ta&el :diamo&d shape; M closes 12-1= mo&ths
+osterior fo&ta&el :tria&"le shape;M closes 2-3 mo&ths
*)0.< 0he $oste%io% fontanel is located at the intersection of the sa*ittal and
lambdoid suture is the space bet)een the pariental bonesF the lambdoid suture
separates the t)o parietal bones and the occipital bone
(oldi&"- asymmetry of head as a result of pressure i& birth ca&al
#e,halohe+ato+as donJt cross the suture lines and are the result of blood #essels
rupturin* in the babyJs scalp durin* labor. (lood outside the #asculature in a ne)born
increases the possibility of ?aundice as the ne)bornJs body tries to reabsorb the blood
. #a,ut succedaneu+3 )hich is simply soft tissue edema of the scalp3 can occur in any
labor and isnJt limited to a prolon*ed second sta*e of labor.
ss
es
s
%o
r
hi
,
d
1s
,l
a
si
a'
.
h
e
n
th
ig
h
s
a
re
r
ot
at
e
d
o
ut
.
a
r
d/
n
o
cl
ic
7
335
7.6? 7ead circumfere&ceM33-35 cm :2-3 cm# 4reater tha& chest circumfere&ce;
6&terior fo&ta&el :diamo&d shape; M closes 12-1= mo&ths
+osterior fo&ta&el :tria&"le shape;M closes 2-3 mo&ths
*)0.< 0he $oste%io% fontanel is located at the intersection of the sa*ittal and
lambdoid suture is the space bet)een the pariental bonesF the lambdoid suture
separates the t)o parietal bones and the occipital bone
(oldi&"- asymmetry of head as a result of pressure i& birth ca&al
#e,halohe+ato+as donJt cross the suture lines and are the result of blood #essels
rupturin* in the babyJs scalp durin* labor. (lood outside the #asculature in a ne)born
increases the possibility of ?aundice as the ne)bornJs body tries to reabsorb the blood
. #a,ut succedaneu+3 )hich is simply soft tissue edema of the scalp3 can occur in any
labor and isnJt limited to a prolon*ed second sta*e of labor.
s
s
h
o
ul
d
3
e
h
e
a
r
d
S
o
m
e
&
e
o
&
a
t
e
s
m
a
y
h
a
v
e
a
3
n
o
r
+
336
7.6? 7ead circumfere&ceM33-35 cm :2-3 cm# 4reater tha& chest circumfere&ce;
6&terior fo&ta&el :diamo&d shape; M closes 12-1= mo&ths
+osterior fo&ta&el :tria&"le shape;M closes 2-3 mo&ths
*)0.< 0he $oste%io% fontanel is located at the intersection of the sa*ittal and
lambdoid suture is the space bet)een the pariental bonesF the lambdoid suture
separates the t)o parietal bones and the occipital bone
(oldi&"- asymmetry of head as a result of pressure i& birth ca&al
#e,halohe+ato+as donJt cross the suture lines and are the result of blood #essels
rupturin* in the babyJs scalp durin* labor. (lood outside the #asculature in a ne)born
increases the possibility of ?aundice as the ne)bornJs body tries to reabsorb the blood
. #a,ut succedaneu+3 )hich is simply soft tissue edema of the scalp3 can occur in any
labor and isnJt limited to a prolon*ed second sta*e of labor.
a
l
e
$
t
r
e
+
i
t
i
e
s
<

+ol

Sy

8i"

SJ!*Should be
strai"ht a&d flat
6&us should be
pate&t without a&y
fissure
?impli&" at the
base is associated
with spi&a bifida
A degree o%
h1,otonicit1 or
Assess+ent %or Haundice
2he J1 techniGue is to 3lanch the s7in over the 3on1 ,ro+inence such as the %orehead/
chest or ti, o% the nose.
N-TEA !aundice starts at the head first3 spreads to the chest3 then the abdomen3 then the
arms and le*s3 follo)ed by the hands and feet3 )hich are the last to be ?aundiced.
!aundice in the first DI hours after the birth is a cause for concern that re4uires further
assess+ent. Possi3le causes o% earl1 0aundice are 3lood inco+,ati3ilit1/ o$1tocin
induction/ and severe he+ol1tic ,rocess. Acrocyanosis of the hands and feet is normal3
33$
7.6? 7ead circumfere&ceM33-35 cm :2-3 cm# 4reater tha& chest circumfere&ce;
6&terior fo&ta&el :diamo&d shape; M closes 12-1= mo&ths
+osterior fo&ta&el :tria&"le shape;M closes 2-3 mo&ths
*)0.< 0he $oste%io% fontanel is located at the intersection of the sa*ittal and
lambdoid suture is the space bet)een the pariental bonesF the lambdoid suture
separates the t)o parietal bones and the occipital bone
(oldi&"- asymmetry of head as a result of pressure i& birth ca&al
#e,halohe+ato+as donJt cross the suture lines and are the result of blood #essels
rupturin* in the babyJs scalp durin* labor. (lood outside the #asculature in a ne)born
increases the possibility of ?aundice as the ne)bornJs body tries to reabsorb the blood
. #a,ut succedaneu+3 )hich is simply soft tissue edema of the scalp3 can occur in any
labor and isnJt limited to a prolon*ed second sta*e of labor.
h1,ertonicit1 is
indicative o%
central nervous
s1ste+ (#NS
da+age
resultin* from slu**ish peripheral circulation
Mon*olian Spots
-ary3 blue or blac8 mar8s that are fre4uently found on the sacral area3 buttoc8s3 arms
shoulders or other areas.
+arle4uins Si*n
Occurs on one side of the body turns deep red color. It occurs )hen blood #essels on one
side constrict3 )hile those on the other side of the body dilate.
Acrocyanosis #ersus Central Cyanosis
Acrocyanosis in#ol#es the e@tremities of the neonate3 for e@ample bluish hands and feet
due to neonates bein* cold or poor perfusion of the blood to the periphery of the body.
6hile central cyanosis3 )hich in#ol#es the lips3 ton*ue and trun8 indicatin* +5PO'IA
)hich needs further assessment by the nurse.
.
E,stein=s ,earls are small3 )hite cysts on the hard palate or *ums of the ne)born. "hey
are nor abnormal and )ill disappear shortly after birth.
8ilia are bloc8ed sebaceous *lands located on the chin and the nose of the infant.
+eman*iomas > &ascular "umors
Ne#i flammeus or port )ine stains
&$NI' CAS$OASA
Should not be remo#ed by oil or hand lotion3 because it is a protecti#e layer of the neonate
after birth3 and it disappears after birth 0 pa*e 9UU lippincot7 Ne#er remo#e it )ith
alcohol or cotton balls3 unless meconium s8inned.
*)0.<
@er&i1 3aseosa
.rythema to1icum &eo&aturum
0ela&"iectasia
+ort wi&e stai& :&evus flamus;
Strawberry hema&"ioma
+eman*ioma is beni*n #ascular tumor that may be present on the ne)born
33=
7.6? 7ead circumfere&ceM33-35 cm :2-3 cm# 4reater tha& chest circumfere&ce;
6&terior fo&ta&el :diamo&d shape; M closes 12-1= mo&ths
+osterior fo&ta&el :tria&"le shape;M closes 2-3 mo&ths
*)0.< 0he $oste%io% fontanel is located at the intersection of the sa*ittal and
lambdoid suture is the space bet)een the pariental bonesF the lambdoid suture
separates the t)o parietal bones and the occipital bone
(oldi&"- asymmetry of head as a result of pressure i& birth ca&al
#e,halohe+ato+as donJt cross the suture lines and are the result of blood #essels
rupturin* in the babyJs scalp durin* labor. (lood outside the #asculature in a ne)born
increases the possibility of ?aundice as the ne)bornJs body tries to reabsorb the blood
. #a,ut succedaneu+3 )hich is simply soft tissue edema of the scalp3 can occur in any
labor and isnJt limited to a prolon*ed second sta*e of labor.

8i"ure 24 7ema&"ioma

8i"ure 25 .rythema to1icum &eo&aturum a&d (ilia
C.GESTATI-NAL ASSESSMENT
+6/6(.0./ *,/S!*4
630!)*
WTERMH bo%n bet<een
359., <ee2s "estation
XP$"$M/ born before CK )ee8s
*estation
Sy&o&ymLo) birth
)ei*ht+/. 0./(
!*86*0-)*?!*4.
&coura"e pare&t to tal'
to, hold, a&d si&" to
i&fa&t3!/3,(3!S!)
* 36/.)bserve for
bleedi&", first
uri&atio&3lea&se the
cord with alcohol a&d
sometimes triple dye
o&ce a
day3/6%5!*4a#
+lace the &ewbor& o&
the abdome&0)*!3
*.3J /.85.Q%hile
the &ewbor& is falli&"
asleep or sleepi&",
"e&tly a&d Kuic'ly
tur& the head to o&e
side0he &ewbor&
simulates wal'i&",
8old the pi&&a :auricle; forward
33A
alter&ately fle1i&" a&d
e1te&di&" the
feetS0.++!*4 )/
%65J!*4
/.85.Q-6-!*SJ!E
S!4*(e*innin* at the
heel of the foot3 *ently
stro8e up)ard alon*
the lateral aspect of
the soleF then the
e@aminer mo#es the
fin*ers alon* the ball
of the footOO"IN-
$%L$'ootin* and
suc8in* refle@ usually
disappears after C.I
months but may
persists for up to 9
yearMOO
$%L$'Symmetric <
bilateral abduction <
e@tension of arms and
handsNe)born/s
fin*ers curl around
the e@aminer/s fin*ers
and the ne)born/s toes
curl
do)n)ard.($AS"
"ISSU$Measure itC
mmLess than C
mm%$MAL$
-$NI"ALIAObser#eL
abia ma?ora co#er
labia minoraLabia
minora are more
prominentF #a*inal
openin* can be
seenMAL$
-$NI"ALIAObser#eSc
rotal sac #ery
)rin8led%e)er
shallo) ru*ae on the
scrotum+$$L
C$AS$SObser#e$@t
end D>C of the )ay
from the toes to the
34B
heelSoles are
smoother3 creases
e@tend less than D>C of
the )ay from the toes
to the heel$A
0Mosby/s Comprehensi#e e#ie) of
Nursin* for NCL$'.N pa*e D9E7
6 &eo&ate bor& before
3= wee's a"e of
"estatio&


8i"ure 2$ +remature
i&fa&ts sole creases,
earlobe a&d premature
female "e&italia

RE TERM IN)ANT

?efi&itio&
+romotes s'i&-to-s'i& co&tact
betwee& pare&t a&d i&fa&t
8eedi&"s are opportu&ities for
pare&t-i&fa&t bo&di&"
*otify physicia& for si"&s of
i&fectio&
N&2E" Sense o% 2ouch
2he +ost highl1 develo,ed
sense at 3irth that is .h1/
neonates res,onds .ell to
touch.

6pply diaper loosely to


preve&t irritatio&
341
*otify physicia& for si"&s of
i&fectio&
Jeep the area clea& a&d dry
Jeep the &ewbor&Es diaper
below the cord to preve&t
irritatio&
Si"&s of i&fectio&< red&ess,
drai&a"e, swelli&", odor
*otify physicia& for si"&s of
i&fectio&
*)0.<
Note any bleedin* or
draina*e from the cord
"riple dye may be applied
for initial cord care
because it minimi2es
microor*anisms and
promotes dryin*F use a
cotton.tipped applicator to
paint the dye3 one time3 on
the cord on 9 inch of
surroundin* s8in
Application of KBG
isopropyl alcohol to the
cord )ith each diaper
chan*e and at least t)o r
three times a day to
minimi2e microor*anisms
and promote dryin*.
*)0.< "he s8in is
surrounded )ith alcohol
)hich promotes dryin* and
cleans the area. "he
umbilical cord dries and
falls off about 9I days.
Pero@ide and lanolin
promote moisture3 )hich
can inhibit dryin* and
allo) *ro)th of bacteria.
6ater doesn/t promote
dryin*.

It is best to care for the
neonate/s umbilical cord
area by cleanin* it )ith
cotton pled*ets moistened
)ith alcohol. "he alcohol
promotes dryin* and helps
decrease the ris8 of
infection. An antibiotic
ointment maybe used
342
instead of alcohol3 because
there are a lot of bacteria
)hich is resistant a*ainst
some bacteria. Other
a*ents such as )ipes3
sterile )ater and soap <
)ater are not as effecti#e
as alcohol.

E. 1ASIC TEACHING
NEEDS -) NE?
ARENTS

3)/? 36/.
b# 0he &ewbor&
be"i&s ma'i&"
crawli&" moveme&ts
with the arms a&d
le"s
c# 2he re%le$ usuall1
disa,,ears a%ter
a3out L .ee7s

6s the &ewbor& faces the left


side, the left arm & le" e1te&d
outward while the ri"ht arm &
le" fle1
%he& the head is tur&ed to the
ri"ht side, the ri"ht arm & le"
e1te&d outward while the left
arm & le" fle1
,sually disappears withi& 3-4
mo&ths
0he refle1 is usually prese&t 3-
4 mo&ths

2he ne.3orn=s toes


h1,ere$tend .hile the 3ig toe
dorsi%le$es
e%le$ disa,,ears a%ter the
ne.3orn is 1 1ear old
6bse&ce of this refle1
i&dicates the &eed for a
&eurolo"ical e1ami&atio&
0humb & forefi&"er form a 3
S.(-/63.T refle1
+rese&t at birth, complete
respo&se may occur up to =
343
wee's
6 persiste&t respo&se lasti&"
more tha& 6 mo&ths may
i&dicate the occurre&ce of brai&
dama"e duri&" pre"&a&cy

A normal refle@ in a youn*


infant caused by a sudden
loud noise. It results in
dra)in* up the le*s3 an
embracin* position of the
arms3 and usually a short
cry.


8i"ure 26 (oro /efle1
or .mbrace /efle1
+almar respo&se lesse&s withi&
3-4 mo&ths
+almar respo&se lesse&s withi&
= mo&ths
D.NE?1-RN RE)LE@ES
0he rooti&" refle1 is elicited by
stro'i&" the &eo&ateLs chee' or
stro'i&" &ear the cor&er of the
&eo&ateLs mouth# 0he &eo&ate tur&s
the head i& the directio& of the
stro'i&", loo'i&" for food# 0his
refle1 disappears by 6 wee's# )ther
optio&s refer to other refle1es see&
i& &eo&ates< 0he palmar "rasp refle1
is elicited by placi&" a& obect i& the
palm of a &eo&ate9 the &eo&ateLs
fi&"ers close arou&d it# 0his refle1
disappears betwee& a"es 6 a&d A
mo&ths# 0he -abi&s'i refle1 is
elicited by stro'i&" the &eo&ateLs
foot, o& the side of the sole, from
the heel toward the toes# 6 &eo&ate
will fa& his toes, produci&" a
positive -abi&s'i si"&, u&til about
a"e 3 mo&ths# 0he suc'i&" refle1 is
see& whe& the &eo&ateLs lips are
touched a&d lasts for about 6
mo&ths#
+65(6/ 4/6S+ /.85.Q
9. *ursi&"
dia"&osisImpaired
*as e@chan*e
related to
immature
5ow socioeco&omic level
+oor &utritio&al status
5ac' of pre &atal care
(ultiple pre"&a&cy
344
pulmonary
functionin*(est
procedureesuscit
ation Abnormal
laboratory
#alues,ecreased
(C/sPinna
recoils 0sprin*s
bac87Pinna opens
slo)ly or stays
folded in #ery
premature infants
D. is8 for fluid #olume
deficit related to
insensible )ater loss at
birth and small stomach
capacity
C. is8 for aspiration related
to )ea8 or absent *a*
refle@ a nd>or
administration of tube
feedin*s
I. +ypothermia related to
lac8 of subcutaneous and
bro)n fat deposits3
inade4uate shi#er
response3 immature
thermore*ulation center3
lar*e body surface area in
relation to body )ei*ht3
and>or lac8 of fle@ion of
e@tremities to)ard the
body.
E. is8 for infection related
to immature immune
response3 stasis of
respiratory secretions3
and> or aspiration
:. Imbalanced nutrition1 less
than body re4uirements
related to lac8 of ener*y
to suc8 and>or )ea8 or
absent suc8in* refle@.
0 Mosby/s Comprehensi#e
e#ie) of Nursin* for
NCL$'.N pa*e D9:7
1# *alo1o&e :*arca&;
2# *ature of the dru"<
*arcotic a&ta"o&ist
Side effects<
7yperte&sio&,
irritability,
tachycardia

5. Sur%actan ( Survanta:
+rior previous early birth
/ace :&o& whites have a hi"her i&cide&ce of prematurity tha&
whites;
3i"arette smo'i&"
0he a"e of the mother : the hi"hest i&cide&ce is i& motherEs
you&"er tha& a"e 2B#;
)rder of birth : early termi&atio& is hi"hest i& first pre"&a&cies
a&d i& those beyo&d the forth ;
3losely spaced pre"&a&cies
6b&ormalities of the reproductive system such as i&trauteri&e
septum
!&fectio&s : specially uri&ary tract i&fectio&s;
)bstetric complicatio&s such as premature rupture of membra&es
or premature separatio& of the place&ta
.arly i&ductio& of labor
.lective cesaria& birth
345
C. Nature o% the drug"
Lung sur%actant to
i+,rove lung
co+,liance
Side e%%ect"
2ransient
3rad1cardia/ rales

5# @itami& J
:6Kuamephyto&;
,se for prophyla1is to
treat hemorrha"ic
disease of the
&ewbor&#
Side effects<
7yperbilirubi&uria

6# .ye prophyla1is
$# :.rythromyci& B#5C
!lotyci&, 0etracycli&e 1C
=# Silver Nitrate 1F ( not
alread1 used E causes
che+ical con0unctivitis;
+rophylactic measure
to protect a"ai&st
*eisseria "o&orrhoeae
a&d 3hlamydia
trachomatis
Side effects<
Silver &itrate ca&
cause chemical
co&uctivitis

?ru" study
+reterm si2e lary&"oscope
.0 tube
Suctio& catheter with
sy&thetic surfacta&t
!solettes :i&cubator;
-edside eKuipme&t
6&emia of prematurity
7yperbilirubi&emia/
'er&icterus
+ersiste&t pate&t ductus
arteriosus
+erive&tricular /
i&trave&tricular
hemorrha"e
/espiratory distress
sy&drome
etino,ath1 o%
,re+aturit1
etrolental fibroplasias are
346
a complication that occurs
if the infant is o#ere@posed
to hi*h o@y*en le#els.
Necroti9ing enterocolitis
3omplicatio&s
+ositio&i&" the i&fa&t o&
the bac' with the head of
the mattress elevated
appro1imately 15 de"rees
to allow abdomi&al
co&te&ts to fall away from
the diaphra"m affordi&"
optimal breathi&" space#

-est positio& for


suctio&i&"<
!&fa&t o& the bac' a&d slide
a folded towel or pad u&der
shoulders to rise, head is i&
&eutral positio&#
-est positio&
*)0.<
resuscitation
3eco+es i+,ortant
%or in%ant .ho
%ails to ta7e %irst
3reath or di%%icult1
+aintaining
adeGuate
res,irator1
+ove+ents on his
o.n.

Suctio&i&"
*)0.< allo.s
re+oving +ucus
and ,revents
as,iration o% an1
+ucus and
a+niotic %luid
,resent in the
+outh and nose o%
the ne.3orn to
esta3lish clear
air.a1.

!&tubatio&s
*)0.< head o%
the in%ant in
neutral ,osition
.ith to.el under
shoulder.

34$
?ecreased serum "lucose
!&creased co&ce&tratio& of
i&direct bilirubi&
?ecreased serum albumi&
*)0.< 2he nor+al
range o% urine out,ut %or
a ,reter+ 3a31 is 1 to
2+l@7g@da1. 2he nor+al
s,eci%ic gravit1 %or a
,reter+ 3a31 is 1.;2;.
2he nor+al range %or
3lood glucose level in a
,reter+ 3a31 is C; to L;
+g@dl.

6ppears small a&d


u&derdeveloped
0he head is
disproportio&ately
lar"e : 3 cm or more
"reater tha& chest si2e;
S'i& is thi& with
visible blood vessel
a&d mi&imal
subcuta&eous fat pads
@er&i1 caseosa is
abse&t
-oth a&terior a&d
posterior fo&ta&elles
are small
3o&tributi&" factors
*ursi&" i&terve&tio&
3ardi&al si"&s
2he nurse=s %irst ,riorit1 in ,re,aring a sa%e environ+ent %or a
,reter+ ne.3orn .ith lo. A,gar scores is to ,re,are
res,irator1 resuscitation eGui,+ent. Air.a1 +aintenance is the
%irst ,riorit1.
-i#e the mother o@y*en by mas8 durin* the birth to pro#ide the
preterm infant )ith optimal o@y*en saturation at birth 0 ;E.UBG7.
=eepin* maternal anal*esia and anesthesia to a minimum also
offers the infant the best chance of initiatin* effecti#e respiration.
(edside larn*yoscope3 endotracheal tube3 suction catethers and
synthetic surfactant to be administered by the endotracheal tube.
Infant must be 8ept )arm durin* resuscitation procedures so he
or she is not e@pendin* e@tra ener*y to increase the metabolic
rate to maintain body temperature.
Obser#e for chan*es in respirations3 color and #ital si*ns
Chec8 efficacy of Isolette1 maintain heat3 humidity and o@y*en
concentration3 administer o@y*en only if necessary
Maintain aseptic techni4ue to pre#ent infection
Adhere to the techni4ues of *a#a*e feedin* for safety of infant
Obser#e )ei*ht.*ain patterns
,etermine blood *ases fre4uently to pre#ent acidosis. Institute
34=
phototherapy )hen hyperbilirubinemia occurs
Support parents by lettin* them #erbali2e and as8 4uestions to
relie#e an@iety.
Pro#ide liberal #isitin* hours for parents3 allo) them to
participate in care.
Arran*e follo).up before and after dischar*e by a #isitin* nurse.
0Mosby/s Comprehensi#e e#ie) of Nursin* for NCL$'.N pa*e
D9:7
-ST TERM IN)ANT

8i"ure 2=
?efi&itio&
+)S0 0./( !*86*0
6 &eo&ate bor& after 42 wee's a"e of "estatio&
3o&tributi&" factors
5ow socioeco&omic level
+oor &utritio&al status
5ac' of pre &atal care
(ultiparous motherEs
3i"arette smo'i&"
0he a"e of the mother :the hi"hest i&cide&ce is i& motherEs you&"er tha& a"e 2B#;
(otherEs with diabetes mellitus
3o&"e&ital ab&ormalities such as omphalocele#
-ody is covered with la&u"o
)ld ma& facies
3lassic si"&sIntrauterine )ei*ht loss3 dehydrations and chronic hypo@ia Nold man faces/
Lon* < thin )ith crac8ed s8in )hich is loose3 )rin8led and strained *reenish yello)3 )ith no #erni@
nor lanu*o
Lon* nails )ith firm s8ull
6ide eyed alertness of one month old baby
(ater&al & child &ursi&"9 a developme&tal approach to comprehe&sive c"f&s a&d &cle1 review9 5
th
ed#
+a"e 1316b&ormal laboratory values!&creased total &o# of /-3Es
!&creased hematocrit level
?ecreased serum "lucose
Scree&i&" test
So&o"ram
-est procedure
/esuscitatio&
34A
*)0.< resuscitatio& becomes importa&t for i&fa&t who fails to ta'e first breath or difficulty mai&tai&i&"
adeKuate respiratory moveme&ts o& his ow&#
Suctio&i&"
*)0.< allows removi&" mucus a&d preve&ts aspiratio& of a&y mucus a&d am&iotic fluid prese&t i& the
mouth a&d &ose of the &ewbor&#
0o establish clear airway#
!&tubatio&s
*)0.< head of the i&fa&t i& &eutral positio& with towel u&der shoulder#
-est positio&+ositio&i&" the i&fa&t o& the bac' with the head of the mattress elevated appro1imately 15
de"rees to allow abdomi&al co&te&ts
3omplicatio&s
(eco&ium aspiratio& sy&drome
/espiratory distress sy&drome
*)0.< Post mature neonates ha#e difficulty maintainin* *lucose reser#es. Other common problems
include Meconium aspiration syndrome3 polycythemia3 con*enital anomalies3 sei2ure acti#ity and cold
stress.
NO"$1 "he infant )ho are e@posed to hi*h blood.*lucose le#els in utero may e@perience rapid and
profound h1,ogl1ce+ia after birth because of the cessation of a hi*h in.utero *lucose load. "he s+all-%or-
gestational-age infant has use up *lyco*en stores as a result of intrauterine malnutrition and has blunted
hepatic en2ymatic response )ith )hich to carry out *luconeo*enesis.
*)0.< "he patient )ith post.term pre*nancy is at hi*h ris8 for decreased placental functionin*3 therefore
increasin* the ris8 of inade4uate o@y*en circulation to the fetus
-edside eKuipme&t
.0 tube
Suctio& catheter
?ru" study
1# @itami& J :6Kuamephyto&;
,se for prophyla1is to treat hemorrha"ic disease of the &ewbor&
Side effects<
7yperbilirubi&uria
2# .ye prophyla1is
:.rythromyci& B#5C !lotyci&, 0etracycli&e 1C Silver *itrate 1C
+rophylactic measure to protect a"ai&st *eisseria "o&orrhoeae a&d 3hlamydia trachomatis
Side effects<
Silver &itrate ca& cause chemical co&uctivitis
*ursi&" dia"&oses1# !&effective airway breathi&"
2# /is' for fluid volume deficit related to i&se&sible water loss at birth
3# !&effective i&fa&t feedi&" patter&*ursi&" i&terve&tio&s1#
6ssess &ewbor&Es respiratory rate, depth a&d rhythm# 6uscultate lu&" sou&d#

*ote< Meconium stained syndrome of POS" MA"U$ neonates Aspiration of meconium is best
pre#ented by suctionin* the neonate/s nasopharyn@ immediatelt after the head is deli#ered and before
35B
the shoulders and chest are deli#ered. As lon* as the chest is compressed in the #a*ina3 the infant )ill
not inhale and aspirate meconium in the upper respiratory tract. Meconium aspiration bloc8s the air
flo) to the al#eoli3 leadin* to potentially life threatenin* respiratory complications.
Suctio& every 2 hours or more ofte& as &ecessary
+ositio& &ewbor& o& side or bac' with the &ec' sli"htly e1te&ded
6dmi&ister )2, a&ticipate the &eed for 3+6+ or +..+
3o&ti&ue to assess the &ewbor&Es respiratory status closely#
.&coura"e as much pare&tal participatio& i& the &ewbor&Es care as co&ditio& allows
2#
6dmi&ister !@ fluids after birth to provide 4lucose to preve&t hypo"lycemia, mo&itor closely the
i&fusio& rate#
Jept the i&fa&t u&der a radia&t heat warmer to preserve e&er"y
(o&itor babyEs wei"ht, serum electrolytes a&d e&sure adeKuate fluid i&ta'e
(easure uri&e output by wei"hi&" diapers
3hec' for blood stools to evaluate for possible bleedi&" from i&testi&al tract#
Jeep a restful e&viro&me&t#
3#
6&ticipate the i&fa&ts &eed to be breastfeed
?emo&strate tech&iKue for feedi&" to mother, &ote proper positio&i&" of the i&fa&t, Slatchi&" o&T
tech&iKue, rate of delivery of feedi&" a&d freKue&cy of burpi&"
+rovide a rela1ed e&viro&me&t duri&" feedi&"
6dust freKue&cy a&d amou&t of feedi&" accordi&" to i&fa&ts respo&se
6lter&ate feedi&" procedure :&ipple a&d "ava"e feedi&"; accordi&" to i&fa&ts ability#
(o&itor motherEs effort, provide feedbac' a&d assista&ce as &eeded
Su""est mother to mo&itor i&fa&ts wei"ht periodically
-THER NE? 1-RN A1N-RMALiTIES
A. RESIRAT-R= DISTRESS S=NDR-ME
9 ?elay i& lu&" maturatio& a&d deficie&cy i& surfacta&t
9 3ommo& amo&" cesarea& birth a&d low birth wei"ht
9 6 serious lu&" disease immaturity a&d i&ability to pre-resulti&" i& hypo1ia a&d acidosis
*)0.1 More common in neonates deli#ered by cesarean section than in those deli#ered
#a*inally.
C-MM-N SIGNS
9 3ya&osis, dysp&ea, ster&al a&d/or costal retractio&s, tachyp&ea, "ru&ti&", a&d &asal
flari&", %laring nares/ E$,irator1 grunting
MANAGEMENT
9 (ai&tai& a pate&t airway, place the i&fa&t i& a warm isollete with o1y"e&, admi&ister
a&tibiotics as prescribed a&d correct acidosis
1. HEM-L=TIC DISEASE
9 6-) or /h i&compatibility
351
C-MM-N SIGNS
9 Haundice in 2C hours o% li%e/ signs o% ane+ia (restlessness/ %atigue/ anore$ia: enlarge+ent
o% liver and s,leen and increase in 3iliru3in levels
RE;ENTI-N INDIRECT C--M1HS TEST
9 0ests for a&ti-/h :V; 6b i& motherEs circulatio&
9 +erformed duri&" pre"&a&cy at first visit & a"ai& about 2= wee'Es "estatio&
RESULTSA
9 !f :-; at 2= wee's, a small dose of :(icro/ho"am; is "ive& prophylactically to preve&t
se&siti2atio& i& the 3
rd
trimester#
9 /ho"am may also be "ive& after 2
&d
trimester am&ioce&tesis
9 !f :V;, levels are titrated to determi&e pote&tial effects o& the fetus
DIRECT C--M1HS TEST
9 0ests do&e o& the cord blood at delivery to determi&e prese&ce of :V; 6b o& fetal /-3Es
RESULTS
9 !f both i&direct & direct 3oombEs test is *.460!@. & i&fa&t is /h :V;
9 *.460!@.< *o formatio& of 6&ti-/h :V; 6b
9 /ho"am :/ho _?` huma& immu&e "lobuli& is "ive& to the /h :-; mother to preve&t
developme&t of a&ti-/h :V; 6b as the rest of se&siti2atio& from prese&t/ust termi&ated
pre"&a&cy#
C. H=ER1ILIRU1INEMIA
9 Serum bilirubi& "reater tha& 15 m"/dl withi& first 24-36 hours of life are alarmi&"
9 6t a&y serum bilirubi& level, au&dice duri&" the first day of patholo"ical process
.valuatio& is i&dicated whe& serum< over 12 +g@dL in the ter+ ne.3orn
0herapy is aimed at preve&ti&" results i& perma&e&t &eurolo"ical dama"i&" from the
depositio& of bilirubi& i& cells
TREATMENTA !otot!e%a$y
0he "oal of phototherapy is to decrease the serum u&co&u"ated bilirubi& level because a hi"h level may lead to
bilirubi& e&cephalopathy :'er&icterus;# +hototherapy does&Lt preve&t hypothermia or promote respiratory
stability# !t has &o effect o& co&u"ated bilirubi&, a water-soluble substa&ce easily e1creted i& uri&e a&d stool#
+hototherapy i&creases "astric motility, causi&" the i&fa&t to have ma&y "ree&, watery stools# 0he i&creased
"astric motility also causes the i&fa&t to be irritable# 0here is &o evide&ce that the &ewbor& has a lactose
i&tolera&ce or malabsorptio& problem, &or is there evide&ce that the &ewbor&Ls bilirubi& levels are risi&" to
da&"erous levels#
*)0.<
2he ,hotothera,1 lights +ust 3e turned o%% .hen seru+ 3iliru3in levels are dra.n 3ecause the light
decreases the 3iliru3in levels in the test tu3e/ and the result re,orted .ouldn=t 3e accurate. 2he in%ant
should 3e re,ositionec at least ever1 2 hours to ,er+it the light to reach all s7in sur%aces. 2he in%ant=s
inta7e should 3e increased to co+,ensate %or the %luid loss through the s7in and the loose stools. 2he e1e
,atches are re+oved .ith ever1 %eeding/ and the e1es are assessed %or con0unctivitis ever1 A hours.
*,/S!*4 !(+5.(.*060!)*<
.1pose as much of the &ewbor&Ls s'i& as possible
352
#over the genital area/ and +onitor genital area %or s7in irritation or 3rea7do.n
( ,ria,is+ +a1 occur:
#over the ne.3ornXs e1es .ith e1e shields or ,atches' +a7e sure e1elids are closed
.hen shields or ,atches are a,,lied
e+ove the shields or ,atches at least once ,er shi%t to ins,ect the e1es %or
in%ection or irritation and to allo. e1e contact
# (easure the Kua&tity of li"ht every = hours
(o&itor s'i& temperature closely
)ncrease %luids to co+,ensate %or .ater loss
E$,ect loose green stools and green urine
8onitor the ne.3ornXs s7in color .ith the %lorescent light turned o%%/ ever1 C to A
hours
8onitor the s7in %or 3ron9e 3a31 s1ndro+e/ a gra1ish 3ro.n discoloration o% the
s7in
e,osition ne.3orn ever1 2 hours
ER=THR-1LAST-SIS )ETALIS
/h a&ti"e&s from the babyLs blood e&ter the mater&al bloodstream ?estructio& of /-3s those results
from a& a&ti"e& a&tibody reactio&
.1cha&"e of fetal a&d mater&al blood ta'es place primarily whe& the place&ta separates at birth
0he mother produces a&ti-/h a&tibodies a"ai&st the fetal blood cells
6&tibodies are harmless to the mother but attach to the erythrocytes i& the fetus a&d cause hemolysis
Se&siti2atio& is rare with the first pre"&a&cy
A(O incompatibility is usually less se#ere
-# 6ssessme&t
1# hyperbilirubi&emia & hemolytic a&emia
2# Haundice that develo,s ra,idl1 a%ter 3irth and 3e%ore 2C hours
(PA2*&L&G)#AL HA!ND)#E:
3# !mpleme&tatio&
1# 6dmi&ister /ho:?; immu&e "lobuli& to the mother duri&" the first $2 hours after delivery if the /h-
&e"ative mother delivers a& /h-positive fetus but remai&s u&se&siti2ed
2# 0he babyLs blood is replaced with /h-&e"ative blood to stop the destructio& of the babyLs red blood cells9
the /h-&e"ative blood is replaced with the babyLs ow& blood "radually
*)0.< "he + ne*ati#e mother )ho has no titer 0ne*ati#e Coombs/ test results3 non sensiti2ed7 and )ho
has deli#ered an + positi#e fetus is *i#en an intra +uscular in0ection o% anti-* (D: (*oGA87.
Paternal blood type mi*ht be determined for the pre*nant + ne*ati#e )oman in order to help determine
fetal blood type..
+o-AM bloc8s antibody production by attachin* to fetal + positi#e blood cells in the maternal
circulation before an immunolo*ical response is initiated.
+o-AM must be administered to unsensiti2ed postpartum )omen after the birth of each + positi#e
infant to pre#ent production of antibodies. If the father of future fetuses is + positi#e hetero2y*ous3 there
is a EBG chance of an + ne*ati#e infantF if he is + positi#e homo2y*ous3 all infants )ill be + positi#e.
353
THE ADDICTED NE?1-RN

*)0.? 8.60,/.S<
Short ,al,e3ral %issures/ *1,o,lastic ,hiltru+, short/ u,turnednose/ 8lat midface
0hi& upper lip, 5ow nasal 3ridge/ 6b&ormal palmar creases, /espiratory distress _ap&ea, cya&osis;, 3o&"e&ital
heart disorders, )rrita3ilit1/ h1,ersensitivit1 to sti+uli/ 2re+ors
Poor %eeding/ Sei2ures#
N&2E" "hese are si*ns of +eroine )ithdra)al usually occurs .ithin 2C to CA hours o% 3irth. "he
ne)born may be 0itter1 and h1,eractive. "he cry is often shrill and persistent )ith ya)nin* and
snee2in*. "endon refle@es are increased3 and 8oro=s re%le$ is decreased.
*)0.< *eroin .ithdra.al neonates
*igh ,itch cr1/ increase )#P/ h1,ogl1ce+ia/ loud and lust1 cr1
*,/S!*4 !*0./@.*0!)*<
1# 8onitor %or res,irator1 distress
2# +ositio& &ewbor& o& side to facilitate drai&a"e of secretio&s
3# Jeep resuscitatio& eKuipme&t at the bedside
4# 8onitor %or h1,ogl1ce+ia
5# 6ssess suc' a&d swallow refle1
6# 6dmi&ister small feedi&"s a&d burp well
$# Suctio& as &ecessary
=# (o&itor ! & B
A# 8onitor .eight and head circu+%erence (#hec7 %or )ncrease )#P:
1B# ?ecrease e&viro&me&tal stimuli
11# 2he use o% narcotic antagonists to reverse res,irator1 de,ression in the drug addicted neonate is
contraindicated 3ecause these drugs +a1 ,reci,itate acute .ithdra.al in the neonate.
NE? 1-RN -) DIA1ETIC M-THER
6# ?escriptio&
*eo&ate bor& to a& i&suli&-depe&de&t mother or "estatio&al diabetic mother a&d with hi"h i&cide&ce of
co&"e&ital a&omalies#
3)(+5!360!)*S< *igh incidences o% h1,ogl1ce+ia/ res,irator1 distress/ h1,ocalce+ia/ and
h1,er3iliru3ine+ia
-# 6ssessme&t
8A#&S&8)A B LGA as a result o% e$cess %at and gl1cogen in tissues
.dema or puffi&ess i& the face a&d chee's
Signs o% h1,ogl1ce+ia/ such as t.itching/ di%%icult1 in %eeding/ letharg1/ a,nea/ sei9ures/ and
c1anosis
7yperbilirubi&emia
Signs o% res,irator1 distress/ such as tach1,nea/ c1anosis/ retractions/ grunting/ and nasal %laring
N-TE )-R CHARACTERISTICS -) H=-GL=CEMIAA
354
6b&ormally low level of "lucose :less tha& 3B m"/d5 i& the first $2 hour 45 m"/d5 after the first 3 days of life
* 2# *ormal blood "lucose level is 4B to a 1-day-old &eo&ate a&d 5B to AB &eo&ate older tha& 1 day
)ncreased res,irator1 rate
2.itching/ nervousness/ or tre+ors
!nsta3le te+,erature
#1anosis
*,/S!*4 !*0./@.*0!)*<
1# (o&itor for si"&s of respiratory distress
2# (o&itor bilirubi& a&d blood "lucose levels
3# (o&itor wei"ht
4# (eed earl1/ .ith 1;F glucose in .ater/ 3reast +il7/ or %or+ula as ,rescri3ed
5# Ad+inister )V glucose to treat necessar1 and as ,rescri3ed
6# (o&itor for edema
$# (o&itor for tremors & sei2ures
SMALL )-R GESTATI-NAL AGE
6# ?escriptio&< 6 &eo&ate who is plotted at or below the 1)th perce&tile o& the i&trauteri&e "rowth curve
NO"$1 T9 Predisposin* factor is Maternal Smo8in*
-# 6ssessme&t
1# 8etal distress
2# 4estatio&al a"e a&d physical maturity
3# 5owered or elevated body temperature
4# +hysical ab&ormalities
5# 7ypo"lycemia
6# Si"&s of polycythemia<
a# /uddy appeara&ce
b# 3ya&osis
c# Nau&dice
$# Si"&s of i&fectio&
=# Si"&s of aspiratio& of meco&ium
NO"$1 Obtainin* a blood sample to determine *lucose le#el )ould ha#e the hi*hest priority to on S-A. A
common complication of the S-A ne)born immediately after birth is hypo*lycemia because of the increased
metabolic rate in response to heat loss and poor hepatic *lyco*en stores. "he S-A ne)born may also ha#e
suffered intrauterine hypo@ia3 )hich depletes *lucose.
3# !mpleme&tatio&
1# (ai&tai& airway
2# (ai&tai& body temperature
3# )bserve for si"&s of respiratory distress
4# (o&itor for i&fectio& a&d i&itiate measures to preve&t sepsis
5# (o&itor blood "lucose levels a&d for si"&s of hypo"lycemia
6# !&itiate early feedi&"s a&d mo&itor for si"&s of aspiratio&
$# +rovide stimulatio&, such as touch a&d cuddli&"
355
A. NER;-US S=STEM AN-MALIES
9 (yelome&i&"ocele type of spi&a bifida, i& which the spi&al cord a&d associated membra&es
protrude throu"h a "ap i& the lami&ae of the vertebrae#

SINA 1I)IDA
DefinitionRefe%s to malfo%mation of s$ine in <!ic! t!e $oste%io% $o%tion of t!e laminae of t!e
(e%teb%ae fails to close.
SynonymsS$inal Dys%a$!ia
Ty$esS$ina bifi&a occ'lta
Menin"ocele
Myelomenin"ocele
S$ina bifi&a occ'ltaMenin"oceleMyelomenin"ocele
: menin"omyelocele*?escriptio&)s an o,ening in the verte3ral colu+n .ith no a,,arent reason. ( ,.
AKA/ 2e$t3oo7 o% Basic Nursing Li,,incott L
th
ed.:
8ost co++on site o% in0ur1 E lu+3osacral area ( 8os31=s #o+,rehensive revie. o% Nursing %or
N#LE?-N ,. 52L:&ne o% the +eninges (the S,inal cord covering: ,rotrudes or herniated through
o,ening in verte3ral colu+n. (,. AKA/ 2e$t3oo7 o% Basic Nursing Li,,incott L
th
ed.:
Menin"es o% $%otecti(e co(e%in" a%o'n& t!e s$inal co%& !as $'s!e& o't t!%o'"! t!e o$enin" in
t!e (e%teb%ae in a sac.
S$inal co%& intact
Ne'%olo"ical &eficit a%e 's'ally N-T RESENT
Can be %e$ai%e& </ little o% no &ama"e to t!e ne%(e $at!<ays.8ost severe %or+ o% s,ina 3i%ida .
( ,. AKA/ 2e$t3oo7 o% Basic Nursing Li,,incott L
th
ed.:
%ot%'sion of t!e s$inal co%& $%ot%'&es t!%o'"! t!e bac2.
Sacs a%e co(e%e& by t!in memb%ane & ne%(e a%e e+$ose&
Ne'%olo"ical &eficits a%e e(i&ent%e&is$osin" )acto%#hild undergoes a gro.th s,urt during
,u3ert1. (,. AKA/ 2e$t3oo7 o% Basic Nursing Li,,incott L
th
ed.:
Un2no<n b't "ene%ally t!o'"!t to %es'lt f%om t%i""e%e& en(i%onment.
;al$%oic aci&92no<n to ca'se ne'%al t'be &efect if a&ministe%e& &'%in" $%e"nancy.
Genetic
Malfo%mation of t!e (e%teb%al a%c! & s$inal co%& &'%in" emb%io"enesis on t!e .
t!
94
t!
<ee2s.N-TEA same
</ meni"ocele
Clinical ManifestationDi+,le is ,resent over the 3ac73one. (,. AKA/ 2e$t3oo7 o% Basic Nursing
Li,,incott L
th
ed.:
E+te%nal cyst &efect in t!e s$inal co%& 's'ally at t!e mi&line
?ea2ness of t!e le"s & lac2 of s$!incte% cont%ol
are ,aral1sis (,. AKA/ 2e$t3oo7 o% Basic Nursing Li,,incott L
th
ed.:
8eningitis E in%la++ation o% the +eninges covering the s,inal cord.
Ence,halitis (,. AKA/ 2e$t3oo7 o% Basic Nursing Li,,incott L
th
ed.:
Ro'n&D %aise& $oo%ly e$it!eliali3e& a%ea a%e at t!e le(el of t!e s$inal col'mnD commonly at t!e
l'mbosac%al
Loss of moto% an& sensation belo< t!e le(el of t!e lesion.
Cont%act'%e in t!e an2lesD 2neesD o% !i$s may occ'%.
Cl'bfeet9 %/t t!e $a%a$le"ic feet in t!e 'te%'s.
1la&&e% &ysf'nction
356
)ecal incontinence & consti$ation9 ca'se& by $oo% inne%(ations of t!e anal s$!incte% & bo<el
m'sc'lat'%e
Sei3'%eD b%ain &ama"eD blin&ness can be a late si"n.Sc%eenin" / Dia"nostic Test?-ra1 (s+all tu%t o%
hair or ,ort urine strain is so+eti+es ,resent in the verte3ral are: ( ,. AKA/ 2e$t3oo7 o% Basic
Nursing Li,,incott L
th
ed.:
Ne'%olo"ic E+amination9in&icate loss of ne'%olo"ic f'nctions belo< t!e &efect.
%enatal Sc%eenin":#
st
T%imeste%*
1loo& test It%i$le sc%eenK
- Inc%ease& se%'m al$!a $%otein.
%enatal 'lt%aso'n&
Amniocentesis
Elective a3ortion ( ,. AKA/ 2e$t3oo7 o% Basic Nursing Li,,incott L
th
ed.:
Afte% bi%t!
S$ine @9%ay %e(eals t!at e+act e+tent & location of t!e &efect.
S$ine Ult%aso'n& to &ete%mine s$inal co%& abno%malities.
CT scan/ MRI
*ote< same with me&i&"oceleMaLo% S'%"e%ySurger1 i% necessar1 (,. AKA/ 2e$t3oo7 o% Basic
Nursing Li,,incott L
th
ed.:
Laminectomy
- clos'%e of t!e o$en lesion
%emo(al of sacLaminectomy
- Clos'%e of t!e o$en lesion
Remo(al of sac '%$ose of S'%"e%yTo $%e(ent f'%t!e% &ete%io%ation of ne'%al f'nction.
To minimi3e t!e &an"e% of %'$t'%e & infection
To im$%o(e cosmetic effect.
To facilitate !an&lin" of infants.To $%e(ent f'%t!e% com$lications.
To $%e(ent ne'%al &ete%io%ation
To facilitate !an&lin" of infant.
ost9-$e%ati(e N'%sin" ca%eMeas'%e !ea& si3e to &ete%mine if !y%oce$!al's is &e(elo$in"
Monito% fo% si"n of inc%ease int%ac%annial $%ess'%e
Loo7 %or sign o% in%ection ( ,. AKA/ 2e$t3oo7 o% Basic Nursing Li,,incott L
th
ed.:
Meas'%e !ea& si3e to &ete%mine if !y%oce$!al's is &e(elo$in"
Monito% fo% si"n of inc%ease int%ac%annial $%ess'%e
A(oi& s$inal co%& &ama"e
ange o% +otion (,assive and active: ( ,. AKA/ 2e$t3oo7 o% Basic Nursing Li,,incott L
th
ed.:
ossible s'%"ical com$licationHy&%oce$!al's
)n%ection (,atient is o,en catheteri9ed:. (,. AKA/ 2e$t3oo7 o% Basic Nursing Li,,incott L
th
ed.:
Hy&%oce$!al's
Paral1sis/ hi, destruction/ 7nee %le$ion contracture/ sensor1 loss (,. 2LL/ Ph1sical 8edicine B eha3ilitation
Basic/ Garrison:1est osition )o% %e9o$/ost9o$ & &se
%one9 to minimi3e t!e tension on t!e sac/%is2 fo% t%a'ma :RationaleATo $%e(ent $%ess'%e on t!e
incision*
*i, slightl1 %le$ed and a3ducted
(eet hanging/ %ree o% +attress and slight trendelen3urg ( reduce s,inal %luid: (,.52L/ 8os31=s
#o+,rehensive evie. %or Nursing N#LE? N:NoteA Same </ Menin"cele
Disease Com$licationMenin"itis9if sac <ill %'$t'%e& t!en infection <ill occ'%
ScoliosisD Cont%act'%e & Loint &islocation
S2in b%ea2&o<n in senso%y &ene%(ate& a%eas & 'n&e% b%aces.D%'"sA
Antibiotics9to $%e(ent infection
Antic!oline%"ic9to im$%o(e t!e '%ina%y incontinence
La+ati(e9 to ac!ie(e bo<el continence in t!e c!il&
35$
Anti$asmo&ics9to cont%ol bla&&e% s$asm
NoteA Same </ Menin"cele
N'%sin" Dia"nosis an& Inte%(entionIm$ai%e& s2in Inte"%ity %elate& to im$ai%e& moto% & senso%y
f'nction.
Ris2 fo% Infection %elate& to contamination
N'%sin" Inte%(entionsA %otectin" t!e s2in inte"%ity
1# A#oid positionin* on the infantJs bac8 to pre#ent pressure on the sac#
,. Do not $lace any co(e%in" &i%ectly o(e% t!e sac.
3. -bse%(e sac fo% e(i&ence of i%%itation o% lea2a"e of CS)
.. !se ,rone ,osition .@ hi,s slightl1 %le$ed to decrease tension on the sac.
0. Place a %oa+ ru33er ,ad@ s+all ,illo. or roll dia,er 3et.een the in%ant=s legs to +aintain hi,s in
a3duction B to ,revent or counteract su3lu$ation.
4. %o(i&e s2in ca%e es$ecially an2lesD 2neesD ti$ of noseD c!ee2s & c!in.
5. %o(i&e $assi(e %an"e of motion e+e%cise.
6. Use foam o% fleece $a& to %e&'ce $%ess'%e of t!e matt%ess a"ainst t!e s2in.
7. A(oi& to'c!in" t!e sac.
%e(entin" Infection
#. Gee$ a%ea clean f%om '%ine an& feces
,. Gee$ t!e infant clean es$. b'ttoc2s & "enitalia
3. A$$ly ste%ile "a'3e /moistene& to<el an& <atc! fo% any si"ns of infection.: fe(e%D i%%itabilityD let!a%"yD
oo3in" of fl'i& o% $'s f%om t!e sac*
NoteA Same </ Menin"cele
Nursing alert"
1. Prevent %urther da+age.
2. 8ost co++on ,ro3le+ is loss o% sensation in the legs (,rotect child against ,ossi3le leg in0ur1.
5. S7in e$a+ination" ,ressure areas and tight clothing.
C. #hange dia,ers i% necessar1 a%ter voiding and de%ecating.
6. Patient is e$tre+el1 sensitive to late$. 2he nurse +ust +a7e sure the1 do not co+e in contact .ith
ite+s such as tourniGuets/ catheters/ ru33er 3ands/ gloves/ 3alloons/ various tu3es +ade o% late$.
L. (olic acid (%olate: ta7es during ,regnanc1 to reduce the severit1.
(,. AKA/ 2e$t3oo7 o% Basic Nursing Li,,incott L
th
ed.:
35=
#
#
8ever with chills
+ai& a&d red&ess i& affected area
+ositive 7oma&Es si"&
2# (ultiparity
3# 7istory of rapid labor
4# +remature or small fetus
5# 5ar"e bo&y pelvis
6#
$# /is's<
2# +eri&eal laceratio&s & 7emorrha"e
C. 6hen deli#erin* the neonate3 you should deli#er the head bet)een contractions. "his )ill pre#ent the
head from bein* deli#ered too suddenly3 thuds pre#entin* a possible tearin* of the perineum.
I.
5# 3# 8etal 3erebral trauma
6ltered tissue perfusio& related to mater&al vital or"a& a&d fetal related to hypovolemia
/is' for i&fectio& related traumati2e tissue

1# !&compete&t cervi1
2# 0rauma
3# !&fectio&
4#
5# S!4*S 6*? SF(+0)(S
1# 5ea'a"e of am&iotic fluid
2# p7 hi"her tha& 6#5
3# *itra2i&e paper reactio& M blue
4#
5# /!SJ 8)/<
1# +rolapsed cord
2# !&fectio&
3# /?S
4#
3o&servative 0reatme&t<
-ed rest i& lateral positio&
7ydratio& w/ !@8 a&d co&ti&uous fetal a&d uteri&e co&tractio& mo&itori&"
0ocolytic 0herapy<
-eta mimetic a"e&ts< /itodri&e :Futopar;
Use of ritodrine can lead to pulmonary edema. "herefore3 the nurse should assess for crac8les and dyspnea.
(lood *lucose le#els may temporarily rise3 not fall3 )ith ritodrine. itodrine may cause tachycardia3 not
bradycardia. itodrine may also cause hypo8alemia3 not hyper8alemia.
itodrine 05utopar7 can cause tremor and ?ittery feelin*s3 so it must be assessed )hether the feelin*s are from
the medication or from the Preterm labor Steroid therapy
!&compete&t cervi1
(ultiple "estatio&
+revious history of +reterm labor
35A
?.S e1posure
.motio&al stress
7ydram&ios
+lace&ta previa
6bruptio place&ta
(ater&al a"e Y1= or P35
?epressa&t dru"s
7ypo1ia
3*S a&omalies
Suture li&es
8o&ta&els head measureme&ts
8etal lie
8etal attitude
8etal prese&tatio&
8etal positio&
8etal statio&
0he 5ippi&cott (a&ual of *ursi&" +ractice, $
th
ed#, 2BB1# 5ippi&cott %illiams & wil'i&s<
+hiladelphia, ,S6# +p#==$-===#
-+elvic i&fectio&
-.&dometriosis
-Smo'i&"
.+istory of IU, usa*e
.
+ills must be ta'e& each day a&d preferably same time each day to achieve ma1imal effective&ess
0hi&s a&d atrophy e&dometrium a&d thic'e&s cervical mucous
6?@6*064.< ca& be use immediately postpartum if clie&t is &ot breastfeedi&" a&d 6 wee's if breastfeedi&"
6omen ta8in* the minipill ha#e a hi*her incidence of tubal and ectopic pre*nancies3 possibly because
pro*estin slo)s o#um transport throu*h the fallopian tubes. $ndometriosis3 female hypo*onadism3 and
premenstrual syndrome arenJt associated )ith pro*estin.only oral contracepti#es.
D ?rops before ovulatio& a&d rises B#2 8-B#= 8
In (asal body temperature method the patient should ta8e her temperature e#ery mornin* upon a)a8enin*
and prior to any acti#ity to a#oid the temperature bein* influenced by other factors.
36B

You might also like